You are on page 1of 351

‫بسم الله الرحمن الرحيم‬

‫هذه الئسئلة تم جمعها و حلها من قبل مجموعة من خريجي كلية الطب بجامعة القصيم ‪ .‬وهم )‬
‫‪ -1‬محمد الحربي ‪ -2 ,‬عبدال فيصل الخميس ‪ -3 ,‬عبدال المنيفي ‪ -4 ,‬مهند الحربي ‪-5 ,‬‬
‫عبدال صالح الخميس ‪ -6 ,‬هاني الجهني ‪ -7 ,‬يائسر الوابلي ‪ -8 ,‬ماجد الحربي ‪ -9 ,‬لؤي‬
‫) العبدي ‪ -10 ,‬منصور الصغير‬

‫أتمنى أن تكون هذه الئسئلة جيده من حيث الترتيب و التنسيق ‪ ,‬والجابات الصحيحة ‪ ,‬علم ًا بأن‬
‫الجابات هي عبارة عن اجتهاد قد تحتمل الخطأ ‪ .‬وهذا الملف يحتوي على قرابة اللف ئسؤال هذه‬
‫الئسئله تم جمعها من خلل موقع الكاديميه الطبيه و من بعض الذين ئسبق لهم الختبار من‬
‫‪ .‬مختلف المناطق ‪ .‬فشكر ُا لكل من قام بكتابة الئسئله‬
‫تمنياتي للجميع بالتوفيق‬

‫أخوكم محمد الحربي‬

1) 25 year teacher have fear attack and worry before enter the
class ( I forgot all the scenario) what is the initial treatment:

a. Selective serotonin reuptake inhibitor

b. Tricyclic depressant

c. Beta blocker

d. -----

e. ------

-This is a case of social phobia and the initial treatment is


answer a (SSRIs).

………………………………………………………………………………………………………
…………………………………………

2) The best initial TTT for depression is:

a. SSRIs

b. Tricyclic depressant

c. MAO inhibitors

d. Beta blocker

e. -----------
-The correct answer is a (SSRIs).

………………………………………………………………………………………………………
…………………………………………

3) Using the following classification :

Risk factor Case (disease) Non case total


Present A B A+b
Absent C D C+d
Total A+C B+D
Relative risk of those with risk factor to those without risk
factor is:

a. A/A+B , C/C+D

b. A/A+B

c. C/C+D

-The correct answer is a.

Positive predictive value (PPV) : a/ (a+b)

Negative predictive value (NPV) : d / (C+d)

Sensitivity : proportion of people with disease who have a positive test:


a / (a+c)

Specificity : proportion of people without disease who have a negative


test : d/ (b+d)

Relative risk (RR) : incidence in expose ÷ incidence in unexposed: ( a /


(a+b) ) / (c/ (c+d) )

Attributable risk : incidence in expose ‫ ـــ‬incidence in unexposed: ( a /


(a+b) ) ‫( ــ‬c/ (c+d) )

Odd ratio (OR) : Odds that a disease person is exposed ÷ Odds that a
nondisease person is exposed : ad / bc
4) 50 year old Man presented to ER with sudden headache,
blurred of vision, and eye pain. The diagnosis is:

a. Acute glaucoma

b. Acute conjunctivitis

c. Corneal ulcer

d. -----

-The correct answer is a.

………………………………………………………………………………………………………
…………………………………………

5) Which heart condition is tolerable during pregnancy:

a. Eisenmenger syndrome

b. Aortic stenosis

c. Severe mitral regurge

d. Dilated cardiomyopathy with EF 20%

e. Mitral stenosis and the mitral area is 1 cm (or mm).

-the correct answer is c.

………………………………………………………………………………………………………
…………………………………………

6) Diffuse abdominal pain “in wave like” and vomiting. The


diagnosis is:

a. Pancreatitis

b. Appendicitis

c. Bowel obstruction

d. Cholelithiasis
The correct answer is c. (colicky pain: is the pain that comes in
waves “wave-like” and it is associated with bowel obstruction,
cholelithiasis, nephrolithiasis)

………………………………………………………………………………………………………
…………………………………………

7) Which type of contraceptive is contraindicative in lactation:

a. OCPs

b. Mini pills (progesterone-only birth control pills)

c. IUD

d. Condom

e. Depo-Provera (progesterone-only birth control injection)

-The correct answer is a. because combined OCPs decrease the


quantity of milk. Progesterone –only contraceptive are not
believed to decrease the quantity of milk.

8) A long scenario about patient with polydipsia ad polyuria. I


don’t remember the scenario but they mention osmolality in
urine and serum, measurement of Na.

I encourage you to read about central VS nephrogenic diabetes


insipidus.

- Q IS NOT COMPLETE .

(Diabetes Insipidus) : failure to concentration of urine as a result of


central or nephrogenic ADH. Present with polyurea, polydepsia and
hyponatremia.
Diagnosis central or nephrogenic by administration of Desmopressin
acetate (DDAVP) , synthetic analog of ADH, can be used to distinguish
central from nephrogenic DI.

1- Central DI : DDAVP challenge will decrease urine output and


increase urine osmalirity

2- Nephrogenic DI : DDAVP change will not significant decrease urine


output

………………………………………………………………………………………………………
…………………………………………

9) A 5 year old child came with earache on examination there is


fluid in middle ear and adenoid hypertrophy. Beside
adenoidectomy on management, which also you should do:

a. Myringotomy

b. Grommet tube insertion

c. Mastidectomy

d. Tonsillectomy

e. -----

-The correct answer is a.

- Myringotomy (is used for bulging acute otitis media)

-Grommet tube insertion (is used for recurrent acute otitis


media)

………………………………………………………………………………………………………
…………………………………………

10) How the randomized control study become strong or of


good validity: ( sorry I forgot the answers)

Q IS NOT COMPLETE

-By doing blinding and allocation concealment. The best way to


increase the power of the study is to increase the sample size.
………………………………………………………………………………………………………
…………………………………………

11) Woman with postpartum depression, beside the medical


TTT, whch should be include in therapy:

a. Family therapy or support

b. -------

-Nonpharmacological treatment strategies are:cognitive


behavioral therapy,psycho educational or support
group,increase partner support,additional help with child care.

12) Classic Scenario of stroke on diabetic and hypertensive


patient. What is the pathopysiology of stroke:

a. Atherosclerosis

b. Anyresm

c. -----

-The correct answer is a.

………………………………………………………………………………………………………
…………………………………………

13) Middle aged patient with an acyanotic congenital heart


disease the X-ray show ventrical enlargement and pulmonary
hypertension:

a. VSD

b. ASD

c. Trancus arteriosus

d. Pulmonary stenosis

-The correct answer is b, bcause the pt. in middle age and


acynotic.
………………………………………………………………………………………………………
…………………………………………

14) Best food in travelling is:

a. Boiling water

b. Water

c. Ice

d. Partial cocked fish and meat

e. ---

-The correct answer is a.

………………………………………………………………………………………………………
…………………………………………

15) Mechanism of vitamin C in wound healing :

a. Epithiliazation

b. Aerobic fibroblast synthesis

c. Collagen synthesis

d. Enhance vascularization

e. ----

-The correct answer is c.

………………………………………………………………………………………………………
…………………………………………

16) Open globe injury . TTT is:

a. Contineuos antibiotic drops

b. Continuous wate and NS drops

c. Continuous steroids drops

d. Sterile cover and the refered


e. -----

-The correct answer is d.

17) Adolescent female counseling on fast food. What you


should give her:

a. Ca + folic acid

b. Vit C + folic acid

c. Zinc + folic acid

d. Zinc + Vit C

-The correct answer is a.

………………………………………………………………………………………………………
…………………………………………

18) What is the TRUE about backache with osteoporosis:

a. Normal x ray vertebra exclude the diagnosis

b. Steroid is beneficial TTT

c. Vit D defiency is the cause

d. Another TTT I don’t remember

-The correct answer is c , because vit. D deficiency is cause


osteomacia. Osteomalcia is considering secondary causes of
osteoporosis. Choice d not mention may be more accurate
answer.

………………………………………………………………………………………………………
…………………………………………

19) Adolescent female with eating disorder and osteoporosis

a. Weight gain

b. Vit D

c. Bisphosphonates

d. --------

-The correct answer is a.


………………………………………………………………………………………………………
…………………………………………

20) Fourth degree hemorrhoids, Management is:

a. Hemoridectomy

b. band ligation

c. sclerotherapy

d. fiber diet

-The correct answer is a.

- Classification of internal hemorrhoid :

First degree: hemorrhoid do not prolapsed

Second degree: hemorrhoid prolapsed upon defecation but


spontaneously reduce

Third degree: hemorrhoid prolapsed upon defecation, but must be


manually reduce

Fourth degree: hemorrhoids are prolapsed and cannot be manually


reduce

-treatment:

First & Second: life style modification (fiber dite)

Third: life style modification with (band ligation, sclerotherapy or


cryotherapy) , if

Failed go to surgery.

Fourth: surgery (hemorrhiodectomy)

21) nulligravida at 8 weak gestational age, follow up for


genetic screening, she refused the invasive procedure but she
agree for once screening , what is the appropriate action now:

a. do ultrasound

b. 1st screening
c. 2nd screening

d. 3rd screening

e. Amncentesis

-The answer is correct B

………………………………………………………………………………………………………
…………………………………………

22) 70 year old male with chronic Hepatitis B virus antigen


carrier. The screening of choice is:

a. Alfaprotien + liver ultrasound

b. Alfaprotien + another tumor marker

c. Abdominal CT + abdominal ultrasound

d. ----

-The correct answer is a.

………………………………………………………………………………………………………
…………………………………………

23) 35 year old smoker , on examination sown white patch on


the tongue, management:

a. Antibiotics

b. No ttt

c. Close observation

d. ---------

-This is a case of leukoplakia and the management includes:ask


the pt. to stop smoking, do a biopsy for the lesion; if there is pre-
cancerous changes or cancer in the biopsy ; surgical excision
should be done.

………………………………………………………………………………………………………
…………………………………………

24) Regarding GDM:


a. Screening for GDM at 24 t0 28 weeks

b. Diet control is always successful TTT

c. Screening at 8 weeks

d. ---------

-The correct answer is a.

………………………………………………………………………………………………………
…………………………………………

25) After doing CPR on child and the showing asystole:

a. Atropine

b. Adrenaline

c. Lidocane

d. ------

-The correct answer is b.

26) Scenario of trauma , on face examination there is shifted


mouth angle, loss of sensation of ant. Third of tongue, which
CN is affected:

a. Facial nerve

b. Trigeminal nerve

c. ----

-The correct answer is b , facial nerve for taste ant. Third of


tongue, but trigeminal nerve for sensation .

………………………………………………………………………………………………………
…………………………………………

27) On examination of newborne the skin show papules or


(pastules) over erythema base:

a. transient neonatal pustular melanosis

b. erythema toxicum neonatorum


c. ----

-The correct answer is b.

………………………………………………………………………………………………………
…………………………………………

28) The most common causes of precocious puberty:

a. Idiopathic

b. Functional ovary cysts

c. Ovary tumor

d. Brain tumor

e. Adenoma

-The correct answer is a.

………………………………………………………………………………………………………
…………………………………………

29) Mother worry about radiation from microwave if exposed


to her child. What you tell her:

a. Not all radiation are dangerous and microwave one of


them

b. Microwave is dangerous on children

c. Microwave is dangerous on adult

d. ----

- In international study : all microwave is dangerous on children


( very vague Q )

………………………………………………………………………………………………………
…………………………………………

30) Earlier sign of puberty in male is:


a. Appearance of pubic hair

b. Increase testicular size

c. Increase penis size

d. Increase prostate size

e. ---

-The correct answer is b.

31) Question about nutritional marasmus on definition.


(protein energy malnutrition).

Marasmus is a form of severe protein-energy malnutrition


characterized by energy deficiency. A child with marasmus looks
emaciated. Body weight may be reduced to less than 80% of the
average weight that corresponds to the height. Marasmus
occurrence increases prior to age 1, whereas Kawasaki
occurrence increases after 18 months. It can be distinguished
from Kawasaki in that Kawasaki is protein wasting with the
presence of edema. The prognosis is better than it is for
Kawasaki.

………………………………………………………………………………………………………
…………………………………………

32) What is the most risk of antihypertensive drugs on


elderly patient:

a. Hypotension

b. Hypokalemia

c. CNS side effect

d. –

-The correct answer is a. because orthostatic hypotension will


increase the risk of fall and fractures in elderly.

………………………………………………………………………………………………………
…………………………………………
33) Asymptomatic woman with trichomonas:

a. Treat if symptomatic

b. Treat if she is pregnant

c. Treat her anyway

d. –

-The correct answer is c.

Single dose of metradinazole for symptomatic and asymptomatic pt.


and treatment partner (because it is sexual transmitted disease).
But, not treatment of asymptomatic trichomniasis in first trimester
of pregnancy. (Contraindication in pregnancy)

………………………………………………………………………………………………………
…………………………………………

34) What is the risk of GDM on her life later:

a. DM type 1

b. DM type 2

c. Impaired fasting glucose

-The correct answer is b.

35) What is the major sign that can tell you that patient have
polycythemia vera rather than secondary polysythemia:

a. Hepatomegaly

b. Splenomegaly

c. Venous engorgement

d. Hypertension
e. ----

-The correct answer is b.

in polycythemia vera there is clonal proliferation of a pluripotent


marrow stem cell. Also, level of erythropietine may be useful
distinguish PCV, level are low, from other causes of polycythemia.
Secondary polycythemia is associated with excessive dirusis,
severe gastroenteritis and burn.

………………………………………………………………………………………………………
…………………………………………

36) What is the deficient vitamin in infantile beri beri :

a. B1(thiamine)

b. C

c. E

d. Niacine

-The correct answer is a.

* VITAMINES DEFICIENCY :

Vit. A : night blindness & dry skin

Vit, B1 (thiamine): Beriberi (polyneuritis, dilated


cardiomayopathy, edema)

Vit. B2 (riboflavin): angular stomatitis, cheilosis. Corneal


vascuarization.

Vit. B3 (nacine) : pellagra ( Diarrhea, Dermatitis, Dementia )

Vit. B5 (pantothenate): dermatitis, enteritis, alopecia, adrenal


insufficiency

Vit. B6 (pyridoxine): convulsion, hyperirritability

Vit B12 (cobalamin): macrocytic megaloblastic anemia,


neurologic symptoms

FOLIC ACID : macrocytic megaloblastic anemia without


neurologic symptoms.
Vit C : scurvy ( swollen gums, brusing, anemia, poor wound
healing)

Vit D : rickets in children , osteomalasia in adult

Vit. E : increase fragility of RBC

Vit. K : neonatal hemorrhage, increase PT & PTT, normal BT

………………………………………………………………………………………………………
………………………………………

37) 17 year boy admit to involve in recurrent illegal drug


injection , what the screening test to do:

a. HIV

b. Hepatitis B

c. Hepatitis C

d. -----

-All of the above choices are correct.?

38) 20 year old male found to have hepatitis b surface


antibodies :

a. Previous vaccination

b. Previous infection

c. Active infection

d. ----

-The correct answer is a.

HBsAg : indicate carrier state.

HBsAb : indicate provide immunity to HBV

HBcAg: associated with core of HBV

HBcAB: during widow period, HBcAb-IgM indicate recent disease


………………………………………………………………………………………………………
…………………………………………

39) Most common cause of secondary hypertension in female


adolescent is:

a. Cushing syndrome

b. Hyperthyroidism

c. Renal disease

d. Essential HTN

e. Polycystic ovary disease

-The correct answer is c.

Most common causes of secondary HTN in young female is


renal artery stenosis ( caused by fibromuscular dysplasia )

40) clomiphene citrate:

a. induce ovulation
b. ------

-The correct answer is a.

………………………………………………………………………………………………………
…………………………………………

41) Scenario about perimenstrual depression syndrome ????

Premenestral syndrome : define as a symptoms complex of


physiological emotional symptoms sever enough to interfere with
everyday life and occur cyclical during luteal phase of menses

Premenestral dysphoric disorder : is a sever form of pf


premenestral syndrome characterized by severe recurrent depressive
and anxity symptoms with premenestral ( luteal phase) onest that
remit a few days after the start of menses.

………………………………………………………………………………………………………
…………………………………………

42) Child present with stiffing neck, fever, headach. You


suspect meningitis what is your initial TTT should be:

a. Tobramycin

b. Levoflaxicine

c. Peneciline(ampicillin)

d. Doxycycline

• I remember there is no vancomycine, ceftriaxone or


dexamethasone on the choices.

-The correct answer is c.

AGE Causative organism Treatment


< 1 MONTH GBS, E coli Ampicillin + cefotaxime or
gentamicin

1-3 MONTHS S.pneumonia, Vancomycin + cefotaxime or


H.infulenza cefriaxone

meningocci

3 MONTHS - Pneumoccoci, Vancomycin + cefotaxime or


ADULT meningococci ceftiaxone

>60 Pneumococi, Ampicillin + vancomycin +


YEAR\acoholism meningococci ceotaxime or ceftiaxone

Gram –ve bacilli

43) A patient have tender, redness nodule on lacriminal duct


site. Before referred him to ophthalmologist what you will do:

a. Topical steroid

b. Topical antibiotics

c. Oral antibiotics

d. Nothing

e. –

The correct answer is c.(this is a case of dacrocystitis).

………………………………………………………………………………………………………
…………………………………………

44) About DM in KSA:


a. about < 10 %
b. Most of the pt of insulin dependant type
c. female more affected with type 2 DM
d. most of NIDDM are obese

-The correct answer is d.


………………………………………………………………………………………………………
…………………………………………

45) Major aim of PHC in Saudi Arabia :

a. To provide comprehensive maternal & child health

b. ------------

-other aims are:community participation, immunization, referral,


chronic disease management, prescribing, health education,
management of communicable disease and environmental
health.

………………………………………………………………………………………………………
…………………………………………

46) 17 y.o,she missed her second dose of varecila


vaccine,the first one about 1 y ago what you'll do:

a. give her double dose vaccine

b. give her the second dose only

c. revaccinate from start

d. see if she has antibody and act accordingly

-The correct answer is b.

………………………………………………………………………………………………………
…………………………………………

47) there is outbreak of difteria and tetanus in community ,


regaring to pregnant woman:

a. contraindication to give DT vaccine

b. if exposed , terminate pregnancy immediately

c. if exposed , terminate after 72 hour

d. give DT vaccine anyway


-The correct answer is d.

48)All of the following are live vaccine except:

a. MMR

b. Oral plio

c. Varicella

d. Hepatitis B vaccine

e. BCG

-The correct answer is d.

………………………………………………………………………………………………………
…………………………………………

49) Pt with scoliosis, you need to refer him to the ortho when
the degree is:

a. 5

b. 10

c. 15

d. 20

-The correct answer is d.

………………………………………………………………………………………………………
…………………………………………

50) 87 year old who brought by his daughter, she said he is


forgettable, doing mess thing in room , do not maintain
attension , neurological examination and the investigation
are normal

a. Alzheimer disease

b. Multi-Infarct Dementia
c. ---

-The correct answer is a.

………………………………………………………………………………………………………
…………………………………………

51) Mechanical intestinal obstruction

a. Nasogastric tube decompression

………………………………………………………………………………………………………
…………………………………………

52) Patient with cancer. You want to break bad news, which
of the following is the answer:

a. Inform his family

b. Find out how much the patient know about it

c. Let social service inform him

d. Don’t tell him

-The correct answer is b.

53) HIV patient has hemorrhagic lesion in the mouth and


papules in the face. Skin biopsy show spindle cells and
vascular structures:

a. Kaposi sarcoma

b. ………………….

Kaposi sarcoma : malignant multifocal neoplasm characterized by


vascular tumors of skin and viscera , caused by Human herpesvirus
8 (HHV8) , most commom associated with AIDS patient. Lesion
usally papules . in biobsy show spindle shape of cell.

………………………………………………………………………………………………………
…………………………………………

54) Patient with retrosternal chest pain , barium swallow


show corkscrew appearance

a. Achalasia

b. Esophagitis

c. GERD

d. Diffuse esophageal spasm

-The correct answer is d.

- Alchalasia : show esophagus dilation with (bird beak) tapering of


distal esophagus

-Diffuse esophagus spasm: show corkscrew appearance.

………………………………………………………………………………………………………
…………………………………………

55) Most common cause of intra cerebral hemorrhage:

a. ruptured aneurysm
b. Hypertension

c. Trauma

d. –

-The correct answer is b.

56) Rt upper qudrent pain and tenderness , fever, high WBC ,


jaundice, normal hepatic marker

a. Acute cholecyctitis

b. Pancreatitis

c. Acute hepatitis

d. –

-The correct answer is a.

………………………………………………………………………………………………………
…………………………………………

57) 5 y.o child with h.o fever and swelling of the face ant to
the both ears (parotid gland enlargement) what is the most
common complication:

a. Labrynthitis

b. meningitis

c. encephalitis

d. orchitis.

-The correct answer is b.

Orchitis: (this one is the most common complication in


postpubertal males-adolscents and adults-). This pt. is
prepuberty .
………………………………………………………………………………………………………
…………………………………………

58) classic symptoms of tension headache .

Tension headach : is the most common type of headach


diagnosed in adult. Present with tight, band-like pain . may be
generalized or most intense in the frontal, occipital, and neck
region. Usually occurs at the end of day. Treatmet avoidance of
exacerbating factors, NSAID and acetaminophen.

………………………………………………………………………………………………………
…………………………………………

59) prophylaxis of Asthma

a. oral steroid

b. inhaler steroids

c. inhaler bronchodilator B agonists

-The correct answer is b.

………………………………………………………………………………………………………
…………………………………………

60) 30 year woman with dysmenorrhea, menorrhagea,


infertility, and on examination found immobile mass on
uterosacral ligaments :

a. uterine fibroid

b. endometriosis

c. ---

-The correct answer is b

61) Classic symptoms of primary dysmenorrheal


Primary dysmenorrhea: menestral pain associated with ovulatory
cycle in abcense of pathological finding.

Secondary dysmenorrhea: Menestral pain wich organic causes


exist, most common cused by endometriosis, adenomyosis, tumor
and fibroid.

………………………………………………………………………………………………………
…………………………………………

62) Newly married woman complain of no pregnancy for 3


month with unprotective sexual intercourse :

a. Try more ( infertility is defined as no pregnancy for


one year)

………………………………………………………………………………………………………
…………………………………………

63) Younger diabetic patient came with abdominal pain,


vomiting and ketones smelled from his mouth. What is
frequent cause:

a. Insulin mismanagement

b. Diet mismanagement

c. ----

-The correct answer is a.. But if the infection is one of the


answers , you should choose it because the most common cause
of DKA is infection.

………………………………………………………………………………………………………
…………………………………………

64) Cause of syncope in aortic stenosis

a. Systemic hypotension
………………………………………………………………………………………………………
………………………………………

65) On stroke management: except

a. IV fluid not include dextrose

b. Diazepam for convulsion

c. No indication of anticonvulsive drugs

d. ----

-The correct answer is c.

………………………………………………………………………………………………………
…………………………………………

66) Patient use illegal drug abuse and the blood show RNA
virus. Which hepatitis

a. A

b. B

c. C

d. D

-The correct answer is c.

67) Classic symptoms and signs of infectious mononucleosis


(EBV) ?

Most common occur in young adult, usually cused by EBV ,


transmission through exchange of body fluid include saliva. Present
with triad ( fever, sorethroat, lymphadenopathy) , also, may present
tonsillar exudates, splenomegaly, maculopapular rash and bilateral
upper eyelid edema. Management is mostly supportive and
corticosteroids are indicated for airway compromise due to tonsillar
enlargement.
………………………………………………………………………………………………………
…………………………………………

68) Treatment of EBV ( in scenario there patent with tonsiller


exudates, lymphadenopathy, splenomegaly) :

a. Oral acyclovir

b. Oral antibiotic

c. IM or IV acyclovir

d. Supportive TTT

e. Observation

-The correct answer is d.

………………………………………………………………………………………………………
…………………………………………

69) 25 year old woman with weight loss, heat intolerance,


irritable ….

a. Hyperthyroidism

………………………………………………………………………………………………………
…………………………………………

70) Patient with coloured pastules around his mouth,


organism show herpes simplex type 1, what is the
treatment:

a. Oral antiviral

b. Iv antiviral

c. Supportive

d. –
-The correct answer is a.

………………………………………………………………………………………………………
…………………………………………

71) Acute otitis media , initial TTT:

a. Amoxicilline

72) Man with sudden onset of scrotal pain , also had Hx of


vomiting, on examination tender scrotom and there is
tender 4 cm mass over right groin, what you will do:

a. Consult surgeon

b. Consult urologist

c. Do sonogram

d. Elective surgery

-The correct answer is b , if expected tortion of tests , don't


wast time on radiological study

………………………………………………………………………………………………………
…………………………………………

73) Most common symptoms of renal cell carcinoma is

a. Hematuria

b. Abdominal mass

c. Flank pain

-The correct answer is a.

………………………………………………………………………………………………………
…………………………………………
74) Link the ttt with organism:

a. Shegella  metronidazole

b. Salmonella  erythromycin

c. Combylobacter  amoxacilline

d. Gardia  I foregut ( but I remember it is antibiotic)

-All of the above answers are wrong.?

………………………………………………………………………………………………………
…………………………………………

75) Medical student had RTA systolic pressure is 70 mmhg,


what you will do next in management:

a. IV fluid therapy

b. ECG

c. Abdominal U/S

d. ---

-The correct answer is a.

………………………………………………………………………………………………………
…………………………………………

76) 20 year old male had been stabbed on midtriceps , ‫المهم لفاها‬
‫وبعد أئسبوع فتحوا الجرح وطلع ئسائل أخضر‬.

On microscopic examination of this greenish fluid show


gram positive cocci in chains:

a. Streptococcal gangrene

b. Chlostrideal gangrene

c. Fournier’s gangrene

d. meningocemia

e. –

-The correct answer is a.


77) Patient around his nose there are pastules, papules and
telangiectasia lesions. The diagnosis is:

a. Rosacea

………………………………………………………………………………………………………
…………………………………………

78) Child with radial head dislocation, what is the next in


management:

a. Reduction

b. x ray

c. MRI

d. –

-The correct answer is a.

………………………………………………………………………………………………………
…………………………………………

79) In cervical LNs there are well differentiated thyroid cells,


during operation you find no lesion on thyroid what will you
do next

a. Total thyroidectomy

b. Total thyoidectomy + radical cervical LNs dissection

c. Total thyroidectomy + specific LNs dissection

d. Thyoid lobectomy with -----

-The correct answer is b.

………………………………………………………………………………………………………
…………………………………………
80) Irritable bowel syndrome , ( they mention a specific
mechanism” I do not remember” that produce which
symptom

a. Vomiting

b. Diarrhea

c. Constipation

d. Abdominal pain

???????????? Q IS NOT CLEAR

………………………………………………………………………………………………………
…………………………………………

81) Free fluid accumulate in abdominal cavity cause:

a. Hypovolemic shock

b. Cardigenic shock

c. Sepsis

d. Emesis

e. ---

-The correct answer is c.

82) Woman with cyclic bilateral nodularity in her breast since


6 month, on examination there is 3 cm tender mobile mass
in her breast : what you will do next

a. FNA with cytology

b. Mammogram

c. Biopsy

d. Follow up for next cycle

e. Observation
-The correct answer is d. ( I'm not sure )

………………………………………………………………………………………………………
…………………………………………

83) Most common symptoms of soft tissue sarcoma :

a. Paralysis

b. Ongrowing mass

c. Pain

d. ---

-The correct answer is b.

Ongrowing mass: painless and slow-growing.

………………………………………………………………………………………………………
…………………………………………

84) the most common symptom in placenta abruption is


a. Vaginal bleeding
b. Uterine tenderness
c. Uterine contractions
d. Fetal distress

The correct answer is a

Placental abruption is mainly a clinical diagnosis with all the


above findings. the most common symptom is dark red vaginal
bleeding with pain during the third trimester of pregnancy (80%)
and abdominal or uterine tenderness (70%). Bleeding may occur at
various times in pregnancy Bleeding in the first trimester of
pregnancy is quite common and may be due to the following:
miscarriage (pregnancy loss) ectopic pregnancy (pregnancy in the
fallopian tube) . Bleeding in late pregnancy (after about 20 weeks)
may be due to the following: placenta previa or placental
abruption.
85) Female presented with vaginal discharge, itching, and
on microscope showed mycoleous cells and spores. This
medical condition is most likely to be associated with:
a. TB
b. Diabetes
c. Rheumatoid Arthritis

The correct answer is b

Vaginal thrush is a common infection caused by a yeast called


Candida albicans. Vulvovaginal candidiasis is usually secondary to
overgrowth of normal flora Candida species in the vagina. Conditions
that interrupt the balance of normal vaginal flora include: antibiotic
use, oral contraceptives, contraceptive devices, high estrogen levels,
and immunocompromised states such as diabetes mellitus and HIV.
Women are prone to vaginal thrush between puberty and the
menopause because, under the influence of the hormone estrogen, the
cells lining the vagina produce a sugar and yeasts which Candida
albicans are attracted to. That is why thrush is rare before puberty.
…………………………………………………………………………………………………………
………………………………………

86) Primigravida in her 8th week of gestation, presented to


your clinic wanting to do genetic screening, she declined
invasive procedure . the best in this situation is

a. Amniocentesis
b. 1st trimester screening
c. 2nd trimester screening
d. Ultrasound

The correct answer is b


…………………………………………………………………………………………………………
………………………………………

87) Trichomoniasis is classically have:


a. Clue cells
b. Greenish frothy discharge

The correct answer is b

Trichomoniasis is caused by the flagellated protozoan


Trichomonas vaginalis; it's the most common curable sexually
transmitted disease in the world. It usually presents with frothy
yellow-green vaginal discharge, strong-unpleasant odor, pain
during urination and sexual intercourse.
…………………………………………………………………………………………………………
………………………………………

87) Obsessive neurosis:


a. Treatment is east
b. Clomipramine doesn’t not work
c. Mostly associated with severe depression
d. Can be cured spontaneously

the correct aswer is c


88) Patient came to you complaining of hearing voices, later
he started to complain of thought get into his mind and can
be taken out
a. SCZ
b. Mood
c. Mania
d. Agoraphobia

The correct answer a


…………………………………………………………………………………………………………
………………………………………

89) Female had history of severe depression, many episodes,


she got her remission for three months with Paroxitine
( SSRIs) .. now she is pregnant .. your advise
a. Stop SSRi's because it cause fetal malformation
b. Stop SSRi's because it cause premature labor
c. Continue and monitor her depression
d. Stop SSRIs

The correct answer is c or a

In general, SSRI have the least side effects then other classes of
antidepressants. Some SE: Sleep disturbance, dizziness, sexual
dysfunction, Nervousness, and diarrhea
…………………………………………………………………………………………………………
…………………………………

90) Hallucinations and Paranoia:

a. SCZ
b. Mood
c. Mania
d. Phobia

The correct answer is a

………………………………………………………………………………………………………
…………………………………………

91) Female presented with thirst and polyurea.. all medical


history is negative and she is not know to have medical
issues.. .she gave history of being diagnosed as Bipolar and
on Lithium but her Cr and BUN is normal. What is the cause
of her presentation
a. Adverse affect of lithium
b. Nephrogenic DI
c. Central DI

The correct answer is a

Nephrogenic diabetes insipidus is characterized by a decrease


in the ability to concentrate urine due to a resistance to ADH
action in the kidney. Nephrogenic diabetes insipidus can be
observed in chronic renal insufficiency, lithium toxicity,
hypercalcemia, hypokalemia.

92) Most common medical problems encountered in Primary


care is:
a. Coryza
b. UTI
c. HTN

The correct answer is a

…………………………………………………………………………………………………………
………………………………………

93) Regarding diabetes mellitus in pregnancy :


a. Prevelance of diabetes mellitus in pregnancy is 10%
b. Glucose screeing is best in 24-28 week
c. Diabetic and non diabetic have same perinatal
mortality
d. Gestational diabeted can be diagnosed by abnormal
FGS test

The correct answer is b

………………………………………………………………………………………………………
…………………………………………

94) Left red eye, watery discharge, photo phobia, peri-


auricular non-tender lymph nodes .. Dx
a. Bacterial conjctvitis
b. Viral conjctvitis

The correct answer is b

………………………………………………………………………………………………………
…………………………………………
95) One of the Anti-psychotics causes ECG changes ,
Leukopenia, drooling :

a. Respiredone
b. Colzapine
c. Amisulpride

The correct answer is b


…………………………………………………………………………………………………………
………………………………………

96) One of the following decrease chance of colon cancer :

a. Zinc
b. Vit. E
c. Vit C
d. Folic acid

????????

Both C, E, Beta carotene all are Antioxidants, they are correct


but C is more likely. A big dose of vitamin C fights the big "C."
some others: Fiber ,Vitamin C, Calcium, Vitamin E, Selenium.

97) Best sentence to describe specifity of screening test,is


the population of people who :
a. Are negative of disease, and test is negative
b. Are positive of disease, and test is negative
c. Are positive comparing to total other people
d. Negative disease , positive test
e. Positive disease , negative test

The correct answer is a

Specificity measures the proportion of negatives which are


correctly identified (e.g. the percentage of healthy people who
are correctly identified as not having the condition).
Sensitivity (also recall rate) measures the proportion of actual
positives which are correctly identified (e.g. the percentage of
sick people correctly identified as having the condition).
…………………………………………………………………………………………………………
………………………………………

98) Heavy smoker came to you asking about other cancer,


not Lung cancer, that smoking increase its risk:
a. Colon
b. Bladder
c. Liver
The correct answer is b

smoking-associated cancers are lung, head &neck (like


esophagus and larynx), bladder and kidney, pancreatic, cervical
and stomach.

………………………………………………………………………………………………………
…………………………………………

99) Mid clavicle fracture :


a. Surgery is always indicated if fracture is displaced
b. Figure-8-dressing has better outcomes than simple
sling
c. Figure-8-dressing is strongly indicated in patient with
un-union risk
d. Both figure-8 and simple sling has similar outcomes

the correct answer is d


- simple sling has been to give the same result as a figure-8
(more comfort and fewer skin problem).
………………………………………………………………………………………………………
…………………………………………

100) patient complains of "can't breathe air in one nostril "; on


examination showed edematoud mucosa structure, best to
give initially :
a. Corticosteroids
b. Decongestants
c. Alfa-adrenergic blockers

The correct answer is a


101) Pediatric came to you in ER with wheezing, dyspnea,
muscle contraction ( most probably asthma), best to give
initially is :
a. theophillin
b. Albuterol neubelizors
c. oral steroids

the correct answer is b

…………………………………………………………………………………………………………
……………………………………

102) Female presented with complain of neck pain and


occipital headache , no other symptoms , on X-ray has
cervical spine osteophyes and narrow disks :
a. cervical sponylosis ?

??????
………………………………………………………………………………………………………
…………………………………………

103) Patient complaining of pain at night when he elevated


his arm, tingeling on lateral arm side and lateral three
fingers , Dx
a. brachial plexus neuropathy
b. shoulder impengment syndrom
c. brachial artery thrombophebitis
d. thoracic outlet problem

the correct answer is d

Branchial plexus neuropathy is characterized by acute onset of


intense pain in the shoulder or arm followed shortly by focal muscle
weakness.

Impingement syndrome, swimmer's shoulder, and thrower's


shoulder, is a clinical syndrome which occurs when the tendons of
the rotator cuff muscles become irritated and inflamed. This can
result in pain, weakness and loss of movement at the shoulder. The
pain is often worsened by shoulder overhead movement and may
occur at night, especially if the patient is lying on the affected
shoulder.

Thoracic outlet syndrome: numbness and tingling in the fingers;


pain in the neck, shoulder, and arm; weakness of the arm and
dropping things from the hand; worsening of the symptoms when
elevating the arm to do such things as comb or blow dry one's hair
or drive a car; and coldness and color changes in the hand. The
symptoms are often worse at night or when using the arm for work
or other activities.

104) Young adult presented with pain on lateral elbow,


tingeling of lateral arm, he plays Squash:
a. carbel tunnel
b. tennis elbow

the correct answer is b


-Lateral epicondylitis( inflammation of common extensor
tendon ) also known as (tennis elbow, shooter's elbow and
archer's elbow is a condition where the outer part of the elbow
becomes sore and tender. It is commonly associated with
playing tennis and racquet sports.
-Medial epicondylitis (inflammation of common flexor elbow)
also know (golfer elbow)
………………………………………………………………………………………………………
…………………………………………

105) male came to you complaining of sudden progressive


decreasing in vision of left eye over last two/three days,
also pain on the same eye, on fundoscopy optic disk
swelling was sees , Dx :
a.central retinal artery occlusion
b.central retinal vein occlusion
c.optic neuritis
d.macular degeneration

they said the correct answer is c

• Ask about symptoms of temporal arteritis in the older population.


Patients complain of sudden, painless, nonprogressive vision loss in
one eye. History of headaches, jaw claudication, scalp tenderness,
proximal muscle and joint aches, anorexia, weight loss, or fever may
be elicited.
• Some patients may reveal a history of amaurosis fugax involving
transient loss of vision lasting seconds to minutes but which may last
up to 2 hours. The vision usually returns to baseline after an episode of
amaurosis fugax.

• Central retinal Artery occlusion: presenting complaint is an acute


sudden painless unilateral loss of vision. Shows a classic cherry red
spot

• Central retinal vein occlusion: usually sudden painless variable


visual loss; the fundus may show retinal hemorrhages, dilated tortuous
retinal veins, cotton-wool spots, macular edema, and optic disc edema.

• Optic neuritis: Major symptoms are sudden loss of vision (partial or


complete), or sudden blurred or "foggy or washed out" vision, and pain
on movement of the affected eye. Many patients with optic neuritis
may lose some of their color vision in the affected eye (especially red).
The optic disk becomes swollen

• Macular degeneration is a leading cause of permanent painless


irreversible vision loss in the elderly.

Retinal Detachment; Symptoms are decreased peripheral or central


vision, often described as a curtain or dark cloud coming across the
field of vision. Associated symptoms can include painless vision
disturbances, including flashing lights and excessive floaters.
106) unilateral headache, exaggerated by excersice and light ,
Dx :
a. migraine
b. cluster headach
c. stress headache

the correct answer is a


………………………………………………………………………………………………………
…………………………………………

107) 70 years old with progressive demntia , no personality


changes , neurological examination was normal but there is
visuodeficit , on brain CT shower cortex atrophy and
ventricular dilatations :
a. multi micro infract dementia
b. alzehimer demenita
c. parkinsonism dementia

the correct answer is b

 alzehimer dementia :
most common cause of dementia. age and family history are risk
factors for AD. Etiology unknown but toxic b-amyloid deposit in
brain. Present with amnesia for newly acquired information is
usually the first presentation, followed by language deficit ,
acaluia, depression, agitation and finally apraxia(inability to
perform skilled movement). Diagnosis by exclusion that can be
definitive diagnosis only on autopsy: suggested by clinical feature
and by progressive cognitive course without substantial motor
impairment. MRI & CT may show atrophy , venticule enlargement
and can rule out other causes. On brain microscopy amyloid
plaques and neurofibrially tangle. Death usually occurring
secondary to aspiration pneumonia . treatment by supportive
therapy for Pt. and family , and cholinesterase inhibitor .

 multi mico infarct dementia ( vascular demensia )


dementia associated with history of strok. Criteria for vascular
dementia include presence of dementia and 2 or more of the
following:
1- focal neurological signs
2- symptoms onset that was abrupt , stepwise, or related to strok
3- brain imaging showing evidence of fold infarction or extensive
deep white matter changes secondary to chronic ischemia.

………………………………………………………………………………………………………
…………………………………………
108) 70 years old with progressive dementia , on brain
microscopy amyloid plaques and neurofibrillary tangles are
clearly visible also Plaques are seen : Dx
a. lewy dementia
b. Parkisonism
c. Alzehimer

The correct answer c


109) after bite, pediatric patient presented with abdominal
pain and vomiting , stool occult blood , rash over buttock
and lower limbs , edema of hands and soles , urine function
was normal but microscopic hematurea was seen:
a. Lyme
b. Henoch-Schonlein Purpura

The correct answer is b


Henoch-Schönlein purpura (HSP) is a small-vessel vasculitis
characterized by purpura, arthritis, abdominal pain, and
hematuria.
………………………………………………………………………………………………………
…………………………………………

110) for the above disorder (Q 109 ), which one is considered


pathological
a. gross hematurea
b. microscopic hematurea
c. rashes
d. ..

The correct answer is a (I'm not sure )

………………………………………………………………………………………………………
…………………………………………

111) Young adult presented with painless penile ulcer rolled


edges .. what next to do :
a. CBC
b. Darkfeild microscopy
c. Culturing

The correct answer is b

Syphilis also known as "great imitator" is a sexually


transmitted disease caused by the spirochete bacteria
Treponema pallidum. Classically presents as single painless non-
itchy skin ulceration with sharp borders. T pallidum is too small
to be seen under the light microscope. So use darkfield
microscopy when sores are present. Blood tests can confirm the
presence of antibodies. The antibodies remain in your body for
years, so the test can be used to determine a current or past
infection.
………………………………………………………………………………………………………
…………………………………………

112) Diabetic female ger 24h-urine proteinn is 150mg ,,


a. start on ACEIs
b. refer to nephrologist
c. do nothing , this is normal range

the correct answer is a

113) which prevent or decrease incidence of getting post


herpetic neuralgia
a. Amitriptylin
b. Acyclovir
c. Varicella vaccination

The correct answer is c

All are true, but if I have to choose the best it will be varicella
vaccination, because it completely prevents.

………………………………………………………………………………………………………
…………………………………………

114) Adult Polycystic kidney mode of inheritance:


a. Autosomal dominant

………………………………………………………………………………………………………
…………………………………………

115) Female came with complain of diahrrea in the last 6


months, she lost some weight, she reported that mostly
was bloody , when you preformed sigmoidoscopy you found
fragile mucosa with bleeding ,Dx
a. colon cancer
b. Chron's
c. Ulcerative colitis
d. Gastroenteritis
e. Hemrrohids

The correct answer is c


………………………………………………………………………………………………………
…………………………………………

116) Anal fissure commonest site


a. Posteriorly

………………………………………………………………………………………………………
…………………………………………

117) mother gave bitrh of baby with cleft lip and palate, she
want to get pergnant again what is the percentage of
recurrence
a. 1%
b. 4%
c. 15%

The correct answer is b

118) 4 years old girl presented with her parents to er with


sore throat and seroangious vaginal discharge with no pain
what is most propable cause,
a. Candida
b. Foreign body
c. Chlamedia
d. Gonococci
e. streptococcus
The correcr answer is e

 Pediatric vaginal discharge:


infectious vulvovaginitis: present with malodorous , yellow green, most -1
common caused by group A streptococcus. ( may be present with sexual
abus "STDs" ) 2-foreign body
3-candidal infection: may associated with diabetes … measure glucose.
4-Sarcoma botryoids (rhabdomyosarcoma ): malignant lesion
appearance of "bunches of graps" within vagina
………………………………………………………………………………………………………
………………………………………

119) patient coplaining of pain along median nerve ditribution


, And positive tinel sign treatment include casting of both
hand in what position
a. Dorsiflexion
b. plantar flexion
c. extention
d. Dduction

The correct answer is c


………………………………………………………………………………………………………
…………………………………………

120) dermatomyosistis what is true


a. distal muscle weakness
b. Underlying malignancy
c. Generalized ??? Skin rash

The correct answer is c

 Polymositis & dermatomyositis :


Polymyositis : progressive , systemic tissue disease characterized
by immune-mediate striated muscle inflammation, present with
symmetric progressive proximal muscle weaknes and pain .

DERMATOMYOSITIS : present polymyositis plus cutaneous


involvement, heiotrop rash (violaceous periorbital rash) , shawl
sign ( rash involving the shoulder, upper chest and back ) ,
Gottron"s papule ( popular rash with scale ).

-both : increase serum CK and anti-Jo antibodies . muscle biopsy uscle


fiber and inflammation.

121) pt with hypertrophic subaortic stenosis referred from


dentist before doing dental procedure what is true
a. 50 % risk of endocarditis
b. 12 % risk of endo carditis
c. No need for prophylaxis
d. post procedure antibiotic is enough

the correct answer is c


………………………………………………………………………………………………………
…………………………………………

122) pt want to quit smoking you till him that symptoms of


nicotine withdrawal appear after
a. 1-2 days
b. 2-4 days
c. 5-7 days
d. 8- 10 days

The correct answer is a


………………………………………………………………………………………………………
…………………………………………

123) pt taking bupropion to quit smoking what is SE


a. Arrythmia
b. Seizure
c. xerostomia
d. Headache

The correct answer is b

………………………………………………………………………………………………………
…………………………………………

124) 14 years old girl complaining of painless vaginal bleeding


for 2-4 days every 3Weeks to 2 months ranging from
spotting to 2 packs per day; she had 2ry sexual ccc 1 year
ago and had her menstruation since 6 months on clinical
examination she is normal sexual ccc, normal pelvic exam
appropriate action
a. OCP can be used
b. You should ask for FSH and prolactin level
??????????????

125) pt want to do dental procedure , he was dx to have


mitral valve prolapse clinically by cardiologist, he had never
done echo before what is appropriate action
a. Do echo
b. No need for prophelaxis
c. give ampicillin
d. Give amoxicillin calvulinic

I think I will do Echo, but this are some of the information I found so I
cant decide.

This is from MYO clinic

Doctors used to recommend that some people with mitral valve


prolapse take antibiotics before certain dental or medical procedures
to prevent endocarditis, but not anymore. According to the American
Heart Association, antibiotics are no longer necessary in most cases for
someone with mitral valve regurgitation or mitral valve prolapse.

Still, if you've been told to take antibiotics before any procedures in the
past, check with your doctor to see how these new recommendations
apply to you.

This is from MedicinNet

The vast majority of patients with mitral valve prolapse have an


excellent prognosis and need no treatment. For these individuals,
routine examinations including echocardiograms every few years may
suffice. Mitral regurgitation in patients with mitral valve prolapse can
lead to heart failure, heart enlargement, and abnormal rhythms.
Therefore, mitral valve prolapse patients with mitral regurgitation are
often evaluated annually. Since valve infection, endocarditis, is a rare,
but potentially serious complication of mitral valve prolapse, patients
with mitral valve prolapse are usually given antibiotics prior to any
procedure which can introduce bacteria into the bloodstream. These
procedures include routine dental work, minor surgery, and procedures
that can traumatize body tissues such as colonoscopy, gynecologic, or
urologic examinations. Examples of antibiotics used include oral
amoxicillin and erythromycin as well as intramuscular or intravenous
ampicillin, gentamycin, and vancomycin.

………………………………………………………………………………………………………
…………………………………………

126) ttt of cyclical mastalgia


a. OCP, analgisc, NSAID, Fat reduction, and magnisuem

Mastalagia : painful breast tissue that can be cyclic and usually


associated with hormonal change, often bilateral .
management : stop current hormonal therapy , reassurance,
stop smoking, fat reduction, analgesic, NSAID, OCP.

127) 4years old child what can he do


a. Copy square and triangle
b. Speak in sentences
b. ..

The correct answer is a

………………………………………………………………………………………………………
…………………………………………
128) baby can sit without support, walk by holding fourniture.
Pincer grasp, pull to stand how old is he
a. 8 months
b. 10 months
c. 12 month
d. 18 month

The correct answer is b

………………………………………………………………………………………………………
…………………………………………

129) repeated question about Alzheimer disease

 alzehimer dementia :
most common cause of dementia. age and family history are risk
factors for AD. Etiology unknown but toxic b-amyloid deposit in
brain. Present with amnesia for newly acquired information is
usually the first presentation, followed by language deficit ,
acaluia, depression, agitation and finally apraxia(inability to
perform skilled movement). Diagnosis by exclusion that can be
definitive diagnosis only on autopsy: suggested by clinical feature
and by progressive cognitive course without substantial motor
impairment. MRI & CT may show atrophy , venticule enlargement
and can rule out other causes. On brain microscopy amyloid
plaques and neurofibrially tangle. Death usually occurring
secondary to aspiration pneumonia . treatment by supportive
therapy for Pt. and family , and cholinesterase inhibitor .

………………………………………………………………………………………………………
…………………………………………

130) repeated Q also about multiinfarct dementia


 multi mico infarct dementia ( vascular demensia )
dementia associated with history of strok. Criteria for vascular
dementia include presence of dementia and 2 or more of the
following:

1- focal neurological signs


2- symptoms onset that was abrupt , stepwise, or related to strok
3- brain imaging showing evidence of fold infarction or extensive
deep white matter changes secondary to chronic ischemia.
131) 73 year patient complain of progressive loses of memory
with decrees in cognition function . C.T reveal enlarge
ventricle and cortical atrophy , diagnosis is

a- Alzheimer
b-multi infarct dementia
c- multiple sclerosis
d-????????

The correct answer is a

• Alzheimer (Dx by exclusion. Its associated with progressive memory loss,


decreased cognition function , & enlarged ventricles with cortical atrophy)
• multi infarct dementia (NOT progressive & it has focal
neurological abnormality)

• multiple sclerosis (recurrent relapsed & complete remission.


Its associated with demylenation of gray-matter)

………………………………………………………………………………………………………
…………………………………………

132) 62 female with –ve pap smear you should advice to


repeat pap smear every:
A- 6m
b- 12m
c- 18m
d- no repeat

The correct answer is d

Screening pap smear:


1- starting at age 21 years or no more than 3 years after becoming
sexually active. 2- women > 30
years who have three consecutive normal test screening ( 1 /
3yeasr). 3- screening should be discontinue for
women > 60-70 years who have had 3 or more normal pap smear.

………………………………………………………………………………………………………
…………………………………………

133) all following are criteria of chronic fatigue syndrome


except
b-???????

chronic fatigue syndrome:


characterizes by profound mental and physical exhaustion. In
associated with multiple system and neurospsychiatric symptoms
that last at least 6 mounth. Must be new ( not life long ) , must not
be relived by rest, and must result in greater than 50% reduction in
previous activity. Presentation with 4 or more of the following :
poor memory / concentration, myalgia, arthalagia, sore throat,
tender lymph node, recent onset headach, unrefreshing sleep,
excessive tiredness with exercise. Treatment by : cognitive and
excercise therapy .also, diet, physiotherapy, dietary supplements, antidepressants

134) 12 year boy with sinsoneurl hear loose , C.T scan show
mass (site in brain???) so diagnosis is
a- aqustic neroma
b-meningioma
c- barotraumas
d????????

The correct answer is a (I'm not sure )

- aqustic neroma (benign tumor of cranial nerve VIII. Mostly occur between
age of 30-60 years. , Mostly unilateral except if it's associated with
neurofibromatosis in which its bilateral)
-meningioma (mostly benign, ↑ with age, more with female. Its occur in the
cerebellopontin)
- barotraumas (mostly in the divers. The damage occur due to ↑ pressure)

..……………………………………………………………………………………………….
………………………………………………

135) 50 y with uncontrolled diabetes ,complain of black to


brown nasal discharge. So diagnoses is
a- mycomyosis
b- aspirglosis
c-foreign body
d????????

The correct answer is a

- mycomyosis (fungal infection caused by Mycorales, affect


nasal sinus & lungs, . …………………. characterized by black
nasal discharge, Dx by biopsy).
………………………………………………………….
……………………………………………………………………………………

136) 55 y complain of dyspnea, PND with past history of mitral


valve disease diagnosis is
a-LT side HF
b- RT side HF
c- pnemothrax
d-P.E
The correct answer is a
……………………………………………………………………………………………
………………………………………

137) clonidin is decrease effect of


a- benzotropin (anticholinirgic for Parkinson. Not affected
by Clonidin)
b-levo dopa (for Parkinson. Changed in the brain to
Dopamen. Clonidin ↓ the effect of Levodopa through
Unknown mechanism)
c-rubstin?????

the correct answer is b


Clonodin is α2 agonist used to TTT hypertension. α2 receptor in the
brain cause ↓ of both COP & peripheral resistant .

138) 2 y baby with gray to green patch in lower back, no


redness or hotness, diagnosis is
a- child abuse
b-no ttt need
c- bleeding tendency
d????

The correct answer is b (I'm not sure )

I think it is " Mongolian spot " , visible in 6 month and normally


disappear to 3-5 years. No need treatment.

………………………………………………………………………………………………………
…………………………………………

139) 15y boy with unilateral gyncomastia your advice is


a- my resolve spontiniously
b-there is variation from person to person
c-decrease use of soda oil or fish oil
d-???????????

the correct answer is a


- uni- or bilateral gynecomastia occur normally in newborn & at
puberty

………………………………………………………………………………………………………
…………………………………………

140) 6m baby with mild viral diarrhea , ttt by ORS as


a-100ml/kg for 4 hour then 50 ml/kg /day after
b-50>>>>>>>>>>>>>>>>.50>>>>>>>>>>
c-100>>>>>>>>>>>>>>>>100>>>>>>>>
d-50>>>>>>>>>>>>>>>>>100>>>>>>

141) 36 y female with breast mass mobile and change with


menstrual cycle , no skin dimple or fathering. Your advice is
a-repeat exam after 2 cycle
b-make biopsy
c-fine needle aspiration (there is NO singe of breast cancer.
It is Fibroadenoma. Just do FNA to exclude cancer & relive
the pt)
d-oral contraception

The correct answer is a (I'm not sure )

………………………………………………………………………………………………………
…………………………………………

142) 50y female with breast cancer and CA125 elevate. So


elevation due to
a-breast cancer
b-associate with ovarian cancer
c-due to old age
d-normal variation

The correct answer is a , beause this pt. high risk of breast cancer.

- CA125 is a tumor marker mostly used for ovarian Ca, but it's also
used with endometrial, fallopian, breast, & GIT Ca

………………………………………………………………………………………………………
…………………………………………

143) 25y female with bradicardia and palpitation. ECG normal


except HR130 and apical pulse is 210 .past history of full ttt
ovarian teratoma, so your advice is
a- struma ovari should be consider
b-vagal stimulate should be done
c- referred to cardiology

The correct answer is a

………………………………………………………………………………………………………
…………………………………………

144) pt with alcohol drinked complain of headache , dilated


pupil hyperactivity, agitation .he had history of alcohol
withdrawal last week so ttt is
a-diazepam
b-naxtrol
c-haloperidol
d?????????

the correct answer is a

-diazepam (for seizure → given 1st occur in the 1st 12h).


-haloperidol (for hallucination → given 2nd occur in the 1st 2
days).

145) 50y man with chronic psychosis and not complains for ttt
.your advice
a- depot haloperidol or floxtin
b-oral lorasepam
c-oral buspiron
d-???????

The correct answer is a

………………………………………………………………………………………………………
…………………………………………

146) 15y boy appear patch in rt lower leg these patch is clear
center , red in peripheral, no fever no other complain so
diagnosis
a-contact dermatitis
b-tinea corpora
c- lyme disease
d-???????

The correct answer is b

-tinea corpora: worm fungal infection which transmitted by


contact skin & has clinical picture as in the question .
………………………………………………………………………………………………………
…………………………………………

147) old pt with of IHD complain for 2 mon of redness in lower


leg and pulse dim- inched in dorsalis pedis these redness
increase in dependant position and limp is cold and no
swelling ,diagnosis is:
a-arterial inssuficncy
b-thrmbophibitis
c-cellulites
d??????

the correct answer is a

-thrombophilibitis: present with pain, swelling and redness .


-celluitis: present with redness, hotness and tender.

………………………………………………………………………………………………………
…………………………………………

148) pt with heart disease complain of LL ischemia your


advice
a-referred to cardiology
b-""""""""""""vascular surgery
c- start heparin
d-??????????/

?????????

149) female after sexual attack on exam hymen tear in


a-2 o'clock
b-4"""""""
c-6""""""
d-8""""""""

the correct answer is c

-tear appear between 5 and 7 o'clock

………………………………………………………………………………………………………
…………………………………………

150) 7y boy complain of limb. CT show a vascular necrosis in


epiphysis of femur your advice
a- surgical ttt
b-splint for 6m
c- physiotherapy
d????????
Q not complete: but with these MCQs , the correct answer is b

This case is "perth's diseas" : vascular necrosis of femoral head. 5-


10 years . usually self limiting with symptoms lasting< 18 months.
Present with painless limb , limited abduction and internal rotation.
Treatment: 1- observation if there is limited femoral head
involvement or full ROM . 2- if extensive or if decrease ROM,
consider bracing, hip abduction with cast.

………………………………………………………………………………………………………
…………………………………………

151) pt with trachoma in eye . for prevention you should


a- water
b- """""""""""""""""+eradication of organism
c- mass ttt
d???????????

the correct answer is a

-trachoma in the eye is a bacterial infection caused by Chlamydia


trachomatis which is transmitted by poor haygen & contaminated
H2O. TTT by antibiotic as erythromycin & Doxycycline. Surgery to
prevent scar

………………………………………………………………………………………………………
…………………………………………

152) your advice to prevent plaque disease is


a-hand washing
b-rodent eradication
c-spry insect side
d-?????????? ?????

153) pt with severe headache and decrease in visual acuity


,pupil is dilated, so ttt
a-pilocarpin drop and ophthalmology referred
b-ergotamine
c-NSID
d???????

the correct answer is a

- this is closed angle glaucoma which characterized by sudden


severe headache, red eye, ↓ visual acuity, & dilated pupil.
Pilocarpin is parasympathomimmic which help in relive the pain

………………………………………………………………………………………………………
…………………………………………
154) main ttt of non inflammatory acne is
a-ritonic acid
b-clindmycin;
c-azalic acid
d-erythromycin

the correct answer is a

-clindamycine & erythromycin are treatment of


inflammatory acne.
. –azalic acid : treatment of non-inflammatory & inflammatory
acne. .
– ritonic acid : treatment of sever acne & non-inflammatory
………………………………………………………………………………………………………
…………………………………………

155) pregnant with insulin dependant with good control, so to


decrease risk of congenital disease
a-good metabolic control before pregnancy
b-"""""""""""""""""""""""1st trimester
c-""""""""""""""""""""""""2nd """""
d-""""""""""""""""""""""""3rd """""""

the correct answer is b(I'm not sure )

………………………………………………………………………………………………………
…………………………………………

156) female not married with normal investigation except


FBS=142. RBS196. so ttt
a-give insulin subcutaneous
b-advice not become married
c-barrier contraceptive is good
d- BMI control

the correct answer is d

157) diapetic pt come to you with disturbance in conscious


RBS : 65. so main drug that cause hypoglycemia:
a-sulphnylurea
b-bugunid
c-acabos α-glucosidase inhibitor
d-pheniform

the correct answer a


-sulphnylurea (↑ insulin secretion, so, it can cause
hypoglycemia)
-bugunid (↑ insulin sensitization, so, can not cause
hypoglycemia)
-acabos α-glucosidase inhibitor ( used for postprandial
hyperglycemia, so, can . . not cause hypoglycemia)
-pheniform (↑ insulin sensitization, so, can not cause
hypoglycemia)
………………………………………………………………………………………………………
…………………………………………

158) 6m boy with fever you should give antipyretic to


decrease risk of
a-febrile convulsion
b- epilepsy
c- disseminate bacteria
d???????????????

The correct answer is a

………………………………………………………………………………………………………
…………………………………………

159) picture in computer appear vesicle , bulla and erythama


in chest skin so ttt
a- acyclovir cream
b-betamethzone cream
c-floclvir
d- erythromycin

multiple choice not clear.

This case is "herpes zoster" . treatment of herpes zoser are


antiviral, analgesic. Antiviral are ( systemic ) and include:
acyclovire, famciclovire, acyclovire. In multiple choice there is
"floclvir" and there is not drug have this name. I think the writer Q
written in wrong spelling. I think the choice c is famciclovire and it
is correct answer. Famciclovir: 500 mg T.I.D for 7 days
160) pt with scale in hair margin and nasal fold and behind
ear with papule and irregular erythema so ttt is
a-nizoral cream
b- atovit
c- acyclovir
d-antibiotic

the correct answer is a ( this is seborrheic dermatitis )

………………………………………………………………………………………………………
…………………………………………

161) 14y girl with athralgia and photosensitivity and malar


flush. And protinurea so diagnosis is
a-RA
b-lupus nephritis
c-UTI
d-??????????

The correct answer is b

………………………………………………………………………………………………………
…………………………………………

162) question about diarrhea and Yesinia bacteria

Yersiniosis : it is infectious disease caused by Yesinia bacteria.


There are 3 types of Yesinia bacteria : 1- Y.entercolitis , 2-
Y.pseudotuberculosis, 3-Y. pestis. Y.enetercolitis cause bloody
diarrhea, terminal ileitis and mesenteric adenitis. Diagnosis by
serological : rise in antibody titer . Treatment : usually self limiting
and tetracycline for sever infection.

………………………………………………………………………………………………………
…………………………………………

163) paraplegia pt with ulcer in lower back 2+2 cm and lose


of dermis and epidermis these ulcer in stage
a- I
b-II
c-III
d-IV

the correct answer is b

-stage I : non-blanchable redness that NOT subside after relive


of the pressure
-stage II : damage to epidermis & dermis but NOT deeper
-stageIII : subcutaneous tissue involvement
-stageIV : deeper than subcutaneous tissue as muscles & bones
164) PREGNANT LADY prim at labor pain , on exa cervical in
stage I of labor so pain management is
a-morphine IM
b-epidural anesthesia
c-general"""""""""
d-local"""""""

The correct answer is b

………………………………………………………………………………………………………
…………………………………………

165) Psycatric pt on antipsychotic drug most drug that lead to


impotence with antipsychotic is
a- proprnlol
b-NSAI
c-ACEI
d-?????????

the correct answer is a


………………………………………………………………………………………………………
…………………………………………

166) man present with painless ulcer in his penis with


indurate base and everted edge so diagnosis is
a- syphilis
b- gonorrhea
c- choncroid
d- HSV

The correct answer is a

- syphilis : painless ulcer in the penis


- gonorrhea: STD by bacterial infection cause pain &
discharge but NO ulcer
- choncroid: STD by bacterial infection cause pain &
discharge but NO ulcer
- HSV : STD by virus that cause painful ulcer with
discharge.
………………………………………………………………………………………………………
…………………………………………

167) man have long history of urethral stricture present with


tender right testis and WBC in urine so diagnosis is
a-epddimorchitis
b- testicular torchin
c- varicosel
d-??????
the correct answer is a
-epddimorchitis : occur with UTI & urinary retension)
- testicular torchin : sudden acute severe testicular pain
with elevated
. transversely ling testes)
- varicosel : due to ↑ intra-abdomenal pressure.

168) man use saldinafil (Viagra), to prevent hypotension you


should not use
a-nitrate
b-B blocker
c-ACIE
d-CCB

the correct answer is a

………………………………………………………………………………………………………
…………………………………………

169) female complain of painless odorless and colorless


vaginal discharge that appear after intercourse so ttt
a-give antibiotic
b-douche after intercourse
c- cervical cancer should be consider
d-may be due to chronic salpingitis

the correct answer is b

Pain OR
Color Smell Other
Itching

↑ during
Normal
Clear OR ovulation,
vaginal Odorless NO
milky sex,
discharge
breastfeeding

Bacterial White-gray
Fishy Yes ↑ during sex
vaginosis OR yellow

Yes especially
Trichomonia Watery Or
during
sis yellow
urination

White-cheesy Yes especially


Candida
thick sticky during sex
………………………………………………………………………………………………………
…………………………………………
170) UTI>14 day, most probably cause pylonphritis
a-,05%
b-,5%
c-5%
d-50%

Difficult Q , This Q referral to urologist.

………………………………………………………………………………………………………
…………………………………………

171) patient complain of right iliac fossa mass so diagnosis


a-diverticulitis
b-appendicitis
c-pancrtitis
d-chrons disease
- the correct answer is d
172) pt with long history of UC on endoscopes see polyp and
cancer lesion on left colon so ttt
a-ttt of anemia
b-left hemicolctomy
c-total colctomy
d- remove polyp

………………………………………………………………………………………………………
…………………………………………

173) female with hair on different site of body and refuse


intake of food and BMI<18 and feel as body is fat so
diagnosis
a-anorexia nervosa
b-bulimia nervosa
c-body dismorphic syndrome
d- anxiety

the correct answer is a

-anorexia nervosa: decrease body wt. amenorrhea and lanugo


(hair ).
-bulimia nervosa: normal or increase body wt. restrict eating
following by overeating then guilt.

………………………………………………………………………………………………………
…………………………………………

174) obese female, insulin resistance and hairstisim so


diagnosis
a-poly cystic ovary
b-hyperprolctinmia
c-familial
d-?"????????
the correct answer is a

………………………………………………………………………………………………………
…………………………………………

175) boy 3 day after flue symptom develop conjunctivitis with


occipital and neck L.N enlarged so diagnosis is
a-adenoviruses
b-streptococcus
c-HSV
D??????????

the correct answer is a

………………………………………………………………………………………………………
…………………………………………

176) child with asthma use betamethazone, most common


side effect is
a-increase intraocular pressure
b-epilepsy
c-growth retardation
d-?????????????
. the correct answer is c

177) curve of HBV marker

answered in Q 38

………………………………………………………………………………………………………
…………………………………………

178) sickling pt after acute attack , discharge on


a- penicillin
b-iron
c-vitamin
d???????????

the correct answer is a

prophylactic of pneumococcal infection by vaccination


and oral penicillin

………………………………………………………………………………………………………
…………………………………………

179) 6m with cough and wheezy chest .diagnosis is


a- asthma
b- broncholitis
c-pneumonia
d-F.B aspiration

The correct answer is b


- asthma : after 2 years old)
- broncholitis : before 2 years old)
-pneumonia : associated with crypitation
-F.B aspiration : sudden wheezing

………………………………………………………………………………………………………
…………………………………………

180) 15y old with pilonidal sinuse so ttt


a-incision surgery
b- local antibiotic
c-daily clean
d-?????????

The correct answer is c


. pilonidal sinus : sinus tract witch commonly contain hairs.
Treatment , firstly conservative ttt in mild case remove all hair,
washing cleaning . if not relive: surgery

181) Female pt 8 wks postpartum,not smoker diagnosed to


have asthma,her asthma was not controlled she attended
ER 3 times last month,on B2 agonist and oral steroid,she
came c/o wheezing and s.o.b mildly cyanosed using her
intercostal muscles,wheezy chest,BP:160/100 P:120 PO2:72
PEF:36,there is oedema in her foot up to the knee,the most
likely diagnosis:
a. COPD
b. pulmonary embolism
c. Acute asthma attack
d. Angioedema

The correct answer is a or c

………………………………………………………………………………………………………
……………………………………………

182) Female pt developed sudden loss of vision(both eyes)


while she was walking down the street,also c/o numbness
and tingling jn her feet ,there is discrepancy b/w the
complaint and the finding,
O/E reflexes and ankle jerks preserved,there is decrease in
the sensation and weakness in the lower muscles not going
with the anatomy,what is your action:

a. Call ophthalmologist
b. Call neurologist
c. call psychiatrist
d. reassure her and ask her about the stressors
the correct answer is : d

………………………………………………………………………………………………..........
...............................................

183) same scenario in Q.182 what is the diagnosis:

a. Conversion disorder
b. Somatoform disorder

The correct answer is a

1 – somatization disorder: female before age 30 years .


symptoms include: . . two GIT , four site of pain , one
sexual dysfunction , one pseudoneuron .
. 2- conversion disorder: symptoms include voluntary or
sensory . . 3- hypochondriasis: fear
from life threatening disease. .
4- body dysmorphic disorder: aware from his imaging.
. 5- somatoform pain disorder: intensity pain is main
symptoms

184) male pt developed corneal ulcer in his Rt eye after


trauma what is the Mx:
a. Antibiotic and cycloplagia is mydrasis and refer to
ophthalmology
b. topical steroid

the correct answer is a

Because infection is a common occurrence in corneal ulcers, your


ophthalmologist will prescribe antibiotic eyedrops. If the infection
appears very large, you may need to use these drops as often as
one drop an hour. -Oral pain medications will be prescribed to
control the pain. Pain can also be controlled with special eyedrops
that keep your pupil dilated (Anticholinergics such as atropine,
hyoscyamine, and scopolamine)
...........................................................................................................................
................................

185) female pt with Rt eye pain and redness with watery


discharge,no h.o trauma,itching,O/E there is diffuse
congestion in the conjunctiva and watery discharge what
you'll do:
a. give Ab
b. give antihistamine
c. topical steroid
d. refer her to the ophthalmologist

???????????????

………………………………………………………………………………………………………
……………………………………………..

186) Epidemic disease in poor sanitation areas affecting


children and young adults:
a. hep A
b. B
c. C
d. D

The correct answer is a

187) mths baby with crying episodes+current jelly stool,looks


slightly pale,signs of obstruction wht is your Mx:
a. barium enema
b. immediate surgery
c. I.v fluid & wait for resolution

The correct answer is a

Intussusceptions :
Condition in which one portion of bowel invaginates into an
adjacent segment "usually proximal to ileocecal valve" . most
common in first two years of life " usually between 3 months and 3
years of life" . Abrupt –onest , colicky abdominal pain , often
accompanied by flexed knee and vomiting , ( one-off pain ) child
may appear well between episodes. Classic triad : abdominal pain ,
vomiting , blood per rectum" only one third of pt." . Late signs :
bloody mucus in stool "currant jelly stool" , abdominal tenderness ,
palpable "sausage- shape " . RUQ abdominal mas.

Investigation & treatment :


-correct any volume and electrolyte abnormality and check ( cbc)
Abdominal film may be normal in early stage , and see
obstruction , perforation in late stage, US see " target sign"
In setting of high clinical suspicion >>> air-contrast barium
enema, should be performed without delay. As diagnostic in 95% of
cases and curative in 80% of cases perform surgical reduction of
gangrenous bowel.

………………………………………………………………………………………………………
……………….………………………......

188) 17 y.o adolescent, athletic ,with h/o Rt foot pain planter


surface,diagnosis is:
a. planter fasciaitis
b. valux……
c. valux…..

Q not complete.

Planter fasciitis( heel spur syndrome ) :

Repetitive strain injury causing micro tears and inflammation of


planter fascia. Common in athletes , also associated with D.M ,
obesity , seronegative and seropositive arthritis.
Clinical feature: morning pain and stiffiness , intense pain when
walking from rest and that subsides as pt. continuous to walk.
Swelling and tenderness over sole, greatest at medical calcaneal
tubercle and 1-2 cm distal to along planter fascia . pain with toe
dorsiflexion

189) pregnant lady 16 wks presented with vaginal bleeding


,enlarged abdomen,vomiting ,her uterus is smaller than
expected for the gestational age,BhCG 80,U/S snowstorm
appearance,diagnosis:
a. complete hydatiform mole
b. partial hydatiform mole

The correct answer is a

………………………………………………………………………………………………………
………………………………………….....

190) 10. 12 y.o boy c/o abdominal pain after playing football,
he denied any h/o trauma, the pain is in the Lt
paraumbilical region what inx you want to do:
a. CXR
b. ultrasound kidney

Q not complete

………………………………………………………………………………………………………
………………………………………….....

191) 5 y.o child with h.o fever and swelling of the face ant to
the both ears (parotid gland enlargement) what is the most
common complication:
a. Labrynthitis
b. meningitis
c. encephalitis
d. orchitis

The correct answer is b

- most common complication of mumps after puberty . but it is


rare occur pre-puberty.

………………………………………………………………………………………………………
…………………………………………

192) . what is the meaning of difficulty breathing:


a. dyspnia
b. tachycardia

The correct answer is a


193) A female patient on the 3rd week postpartum. She says to
the physician that the frequently visualizes snakes crawling
to her baby’s bed. She knows that it is impossible but she
cannot remove the idea from her head. She says she wakes
up around 50 times at night to check her baby. This
problem prevents her from getting good sleep and it
started to affect her marriage. What is this problem she is
experiencing?

a. An obsession

b. A hallucination

c. A postpartum psychosis

d. A Delusion

The correct answer is a ( 100 % correct  )

-obsession : persistent, unwanted, and intrusive ideas , thoughts,


impulses or images

-hallucination: perception of objective or event without existing


external stimulus -illusion: false perception of actual external
stimulus. - delusion: fixed
false idiosyncratic belief.

………………………………………………………………………………………………………
………………….………………………….

194) female pt c/o sever migraine that affecting her work,she


mentioned that she was improved in her last pregnancy,to
prevent that:
a. biofeedback
b. propranolol

The correct answer is b

- migraine prophylactic :
1- b-blocker (propranolol) : first line treatment (note: contindication
in pregnancy)
2- TCA (amitriptyline )
3- anticovulsant
4- calcium channel blocker

………………………………………………………………………………………………………
……………………………………………..
195) About DM in KSA:
a.. most of NIDDM are obese ( correct )

………………………………………………………………………………………………………
…………………………………………….

196) Flu vaccine not given to the baby who is allergic to:
a. egg

contraindication of flu vaccine are egg allergic and


hypersensitivity to thimerosal

197) 17. Pt with asymptomatic Trichomniasis:


a. treat her anyway regardless
b. treat her if she is symptomatic only

The correct answer is a

-treatment of trichomnias :

Single dose of metradinazole for symptomatic and


asymptomatic pt. and treatment partner ( because it is
sexual transmitted disease )
But not treatment of asymptomatic trichomniasis in first
trimester of pregnancy. (contraindication in pregnancy )

………………………………………………………………………………………………………
…………………..…………………………

198) 7 y.o,she missed her second dose of varecila vaccine,the


first one about 1 y ago what you'll do:
a. give her double dose vaccine
b. give her the second dose only
c. see if she has antibody and act accordingly

The correct answer is b

………………………………………………………………………………………………………
…………………………………………….

199) pt with gonorrhea infection what else you want to check


for
a. Clamydia trachomatis
………………………………………………………………………………………………………
…………………………………………….

200) female pt with Aortic stenosis,she developed syncope


while she was in the class and she recovered
immediately,what is the cause of syncope:
a. valvular rupture
b. systemic hypotension

The correct answer is b

………………………………………………………………………………………………………
……………………………………………..

201) diabetic pt well controlled,she came with h.o dizziness


and sweating after taking a medication BS:60 what drug
that cause her prob:

-Sulfanyurea can cause this side effects.

202) male pt with acute urine retention what is your action:


a. insert folly's cath and ask him to come back to the
clinic

………………………………………………………………………………………………………
…………………………………………….

203) In battered women which is true:


a. mostly they come from poor socioeconomic area
b. usually they marry a second violent man
c. mostly they come to the E/R c/o………..
d. mostly they think that the husband respond like this
because they still have . strong feeling for them

Answer ( d )

Battered women is Women who are physically and mentally


abused over an extended period, usually by a husband or other
dominant male figure. Characteristics of the battered woman
syndrome are helplessness, constant fear, and a perceived inability
to escape. So I think choice d is correct answer

………………………………………………………………………………………………………
……………………………………………..
204) smoking withdrawal symptoms peak at:
a. 1-2 days
b. 2-4 days
c. 5.7 days
d. 10-14 days

The correct answer is a

………………………………………………………………………………………………………
……………….…………………………….

205) Mother who is breast feeding and she want to take MMR
vaccine what is your advice:
a. can be given safely during lactation
b. contain live bacteria that will be transmitted to the
baby
c. stop breast feeding for 72 hrs after taking the vaccine

The correct answer is a

MMR : contraindication during pregnancy , and women should


be avoid pregnant in 4 weeks followed MMR vaccine . MMR is
safe during lactation.

206) male pt c/o pain in his Rt elbow,he said that he is using


the hummer a lot in his work diagnosis:
a. lateral epichondylitis

-Lateral epicondylitis( inflammation of common extensor tendon )


also known as (tennis elbow, shooter's elbow and archer's elbow is a
condition where the outer part of the elbow becomes sore and tender.
It is commonly associated with playing tennis and racquet sports.
-Medial epicondylitis (inflammation of common flexor elbow) also
know (golfer elbow)

………………………………………………………………………………………………………
……………………………………………

207) 50 y.o male with difficulty swallowing food with wt loss:


1. Oesophageal cancer

question not complete . but most likely is cancer.


causes of dysphagia food than liquid are :
1- carcinoma … 2- stricture … 3- plummer vision syndrome = web (
iron deficieny anemia + golssitis )
………………………………………………………………………………………………………
……………………………………………

208) . young female with pain in her elbow(lateral


epichondylitis) best treatment is :
a. ………….+NSAID
b. electric ………….

question not complete .

- treatment of lateral epichondylitis:


1- first line : NSAID +rest + ice
2- second line : corticosteroid injection
3- third line : surgery .percutaneous release of common tendon

………………………………………………………………………………………………………
………………………..…………………..

209) what drug that improve the survival in CHF


1. digoxin
2. Hydralazin

question is not complete .

Improve mortaility in CHF : B-blocker + ACE + ARB ( most


important is ACE )

210) old man with bilateral knee pain and tenderness that
increase with walking and relieved by rest;
a. RA
b. OA

The correct answer is b

OA: pain with activity and weight bearing . and improve with rest .
RA: morning stiffness > 1 hour . painful and warm swelling of
multiple symmetric joint .

………………………………………………………………………………………………………
……………………………………………

211) Regarding peritonitis:


a.Complicated appendectomy the cause is anerobe
organism
b. rigidity and the cause is paralytic ileus
c. can be caused by chemical erosions
d. …………..

The correct answer is c

………………………………………………………………………………………………………
……………………………………………..

212) regarding MI all of the following true except:


a. unstable angina,longer duration of pain and can occur
even at rest.
b. stable angina,shorter duration and occur with excertion
c. there should be q wave in MI
d. even if there is very painful unstable angina the cardiac
enzymes will be normal
e. ………………

all of the above are correct ???

………………………………………………………………………………………………………
……………………………………………

213) Pt with scoliosis, you need to refer him to the ortho when
the degree is:
a. 5
b. 10
c. 15
d. 20

The correct answer is d

214) 6 mths baby with undescending testis which is true:


a. till the mother that he need syrgery
b. in most of the cases spontaneous descent after 1 year

c. surgery indicated when he is 4 years


d. unlikely to become malignant

The correct answer is a

-after 1 year testes not desending. And surgery(orchiopexy) do


from 6 –18months
………………………………………………………………………………………………………
……………………………………………

215) 24 y.o Pt with asymptomatic congenital inguinal hernia:


a. immediate surgery
b. surgery indicated when he is >35 y
c. elective surgery if it is reducible
d. …………..

The correct answer is c


………………………………………………………………………………………………………
…………………………………………..

216) the most effective thing regarding counseling:


a. family rapport
b. well adjusted appointment before counseling
c. ……………….

?????

………………………………………………………………………………………………………
…………………………………………..
217) 5 years old C/O limping in CT there is a venous necrosis
ttt is:
a. surgery
b. splent
c. physiotherpy

Q not complete:

This case is "perth's diseas" : vascular necrosis of


femoral head. 5- 10 years . usually self limiting with
symptoms lasting< 18 months. Present with painless limb ,
limited abduction and internal rotation.
Treatment: 1- observation if there is limited femoral
head involvement or full ROM . 2- if extensive or if
decrease ROM, consider bracing, hip abduction with cast.

218) Pt. has DM and renal impairment when he had diabetic


nephropathy:there is curve for albumin
a. 5y
b. 10y
c. 20y
d. 25y
………………………………………………………………………………………………………
……………………………

219) Pt has alzahimar disease and halusination and delusion ttt:


a. Halopridole

Psychotic symptoms (e.g. hallucinations and delusions),


agitation and aggressive behavior are common in patients with
Alzheimer’s Disease. A study suggests that haloperidol at a
dose of 2-3 mgs/day is effective and well tolerated by most
patients.

220) Generalize anexity disorder best ttt:


a. SSRIs
b. tricyclic A D

The correct answer is a


……………………………………………………………………………………………
………………………………………

221) Major depression management:


a. Intial therapy even sever

 Management of major depression disorder:


1-pharmacotherapy: effective in 50 – 70% .allow for 2-6 weeks to
take effect , treat more than 6 months ( SSRI, TCAs, MAOIs).
2-psycotherapy: psychotherapy combined with antidepressant is
more effect than either treatment alone.
3- Electroconvulsion ( ECCT ).
4- phototherapy: effective for pt. who has a seasonal pattern.

……………………………………………………………………………………………
………………………………………

222) Psychiatric pt with un compliance of drugs ttt:


a. depro halopredol

……………………………………………………………………………………………
………………………………………

223) Strongest risk factor for strok:


a. Hypertension
b. Atrial fibrillation

The correct answer is a

224) Pt. with salpingitis and there is swelling in pelvis in


posterior fornex and it is fluctuant m:
a. Colpotomy
b. Laproscopic

The correct answer is a (I'm not sure )

colpotomy, also known as a vaginotomy, is a procedure by


which an incision is made in the vagina.
Purpose: A colpotomy is performed either to visualize pelvic
structures or to perform surgery on the fallopian tubes or
ovaries.

……………………………………………………………………………………………
………………………………………

225) Child swallowing battery in the oesophegus,


management?
a. broncoscoby
226) In the appendisits the histology is:
a. leukocyte in muscle
b. layer of lymphoid
c. tumor
d. plasma cell

the correct answer is a

in appendicitis : neutrophil exudation throughout mucus,


submucus, and muscularis

……………………………………………………………………………………………
………………………………………

227) water in the body:


a. 40%
b. differ depend on age and sex

the correct answer is b

Guyton's Textbook of Medical Physiology states that "the total


amount of water in a man of average weight (70 kilograms) is
approximately 40 liters, averaging 57% of his total body weight. In
a newborn infant, this may be as high as 75% of the body weight,
but it progressively decreases from birth to old age, most of the
decrease occurring during the first 10 years of life. Also, obesity
decreases the percentage of water in the body, sometimes to as
low as 45%".

……………………………………………………………………………………………
………………………………………

228) Corneal ulcer ttt:


a. Antibiotic and cycloplagia is mydrasis and refer to
ophthalmology

-Because infection is a common occurrence in corneal ulcers,


your ophthalmologist will prescribe antibiotic eyedrops. If the
infection appears very large, you may need to use these drops as
often as one drop an hour. -Oral pain medications will be
prescribed to control the pain. Pain can also be controlled with
special eyedrops that keep your pupil dilated (Anticholinergics such
as atropine, hyoscyamine, and scopolamine)
229) regarding drainage of the abscess one of the following is
true:
a. Carbuncle and frunculosis need drainage
b. Usually give ciphtriaxon and penicillin post draing

the correct answer is a

Frunculosis and carbuncles are pus-filled infected lumps on the


skin. They usually occur as a one-off in a healthy person.
Treatment commonly involves draining the pus and taking a course
of antibiotics. If you have recurring boils you may be advised to
have tests to check for an underlying cause.

…………………………………………………………………………………
…………………………………………………

230) Salpingitis and PID on penicillin but not improve the


most likely organism is :
a. Chlamydia trachomatis
b. Neisseria gonorrhoeae

The correct answer is b

Empiric antibiotic regimens should be aimed at treating likely


causative agents, that is, N. gonorrhoeae, C. trachomatis, genital
mycoplasmas, and bacterial vaginosis-associated endogenous
microflora. The latter include anaerobic (Bacteroides and Prevotella
species and anaerobic streptococci) as well as aerobic organisms
(G. vaginalis, E. coli, and facultative streptococci). Except for N.
gonorrhoeae and some anaerobes, resistance is not yet a clinical
problem.

……………………………………………………………………………………………
………………………………………

231) Wound at end inflammatory phase when:


a. Epithelial tissue formation
b. Angiogenisis
c. when the wound clean
d. Scar formation

The correct answer is b


232) Juvenile RA ttt:
a. Aspirine
b. Steroid
c. Penicillamine
d. dydrocloroquin

The correct answer is a

The treatment of JRA focuses on suppressing inflammation,


preserving and maximizing function, preventing deformity, and
preventing blindness. NSAIDs are the first choice in the treatment
of JRA.

……………………………………………………………………………………………
………………………………………

233) Pt. has hemorrhoid with 4 degree ttt:


a. Hemoridectomy

b. band ligation

c. sclerotherapy

d. fiber diet

-The correct answer is a.

- Classification of internal hemorrhoid :

First degree : hemorrhoid do not prolapsed

Second degree : hemorrhoid prolapsed upon defecation but


spontaneously reduce

Third degree : hemorrhoid prolapsed upon defecation , but must


be manually reduce
Fourth degree : hemorrhoids are prolapsed and cannot be
manually reduce

-treatment:

First & Second : life style modification ( dite)

Third : life style modification with (band ligation, sclerotherapy or


cryotherapy ) , if

Failed go to surgery.

Fourth : surgery ( hemorrhiodectomy )

234) Pt. come with history of tinia capitis ttt:


a. tar shampoo
b. Fluconazol

The correct answer is b

Newer antifungal medications, such as ketoconazole,


itraconazole, terbinafine, and fluconazole, have been reported
as effective alternative therapeutic agents for tinea capitis. Of
these agents, itraconazole and terbinafine are used most
commonly.

……………………………………………………………………………………………
………………………………………

235) Pt. with history of COPD the most action to prevent


complication is:
a. Pnemoccoccal vaccine
b. Smoking stop
c. Oral steroid
d. B2 agonist
The correct answer is b or a

……………………………………………………………………………………………
………………………………………

236) Pt. with congenital hip dislocation :


a. abducting at flexed hip can causes click or tali

Congenital hip dislocation (CHD) occurs , more commonly in girls than in


boys. The left hip is twice as often involved as the right and bilateral
dislocation occurs in more than 25 percent of affected children. The criteria
for the diagnosis of congenital dislocation of the hip include both physical and
radiographic findings. Certain clinical signs have been identified that are
helpful in the evaluation of newborns and infants for possible CHD, which
include the following:
• limited abduction of the flexed hip, due to shortening and contraction
of the hip adductors;
• increase in depth or asymmetry of the inguinal or thigh skinfolds;

• shortening of one leg;

• Allis' or Galeazzi's sign -- lower position of knee of the affected side


when knees and hips are flexed, due to location of femoral head
posterior to acetabulum in this position;
• Ortolani's "jerk" sign ("clunk of entry" or reduction sign);

• Barlow's test ("clunk of exit" or dislocation sign);

• telescoping or pistoning action of thighs, due to lack of containment of


femoral head with acetabulum;

• Trendelenburg's test -- drop of normal hip when child, standing on both


feet, elevates unaffected limb and bears weight on affected side, due
to weakness of hip abductors;

• waddling type of gait.

There are characteristic radiographic presentations which are present in each


of the stages of CHD. The term congenital hip dysplasia generally refers to
delayed or defective development of the hip joint leading to a deranged
articular relationship between an abnormal acetabulum and a deformed
proximal end of the femur. This condition is considered a precursor of the
clinical entities of subluxation and dislocation of the hip. In congenital
subluxation of the hip, there is an abnormal relationship between the femoral
head and acetabulum, but the two are in contact. Congenital dislocation of
the hip, on the other hand, is associated with a complete loss of contact of
the femoral head with the acetabular cartilage. Unfortunately, the femoral
head and acetabulum in the newborn cannot be assessed by direct
visualization, since the femoral head is not ossified and is a cartilaginous
body which is not visualized on plain films. The ossification center for the
femoral head generally appears between three and six months; a delay in its
appearance is an indication of congenital hip dysplasia. The neck of the
femur is used for ascertaining the relationship between the acetabulum and
the femoral head until the ossification center appears. The measurement
used to evaluate the relationship of the femoral head and acetabulum include
Hilgenreiner line, acetabular index, Perkin-Ombredanne line, Shenton-Menard
line, and Andren-von Rosen line.

The principal treatment for CHD is conservative, especially if diagnosed early.


The most common technique is to reduce the dislocation of the femoral head
by means of a flexion/abduction maneuver, for a sufficient period of time to
permit proper growth of the head and acetabulum, which in turn assures a
congruent and stable hip joint. This technique is usually performed on
patients in the very early stages of CHD and in infants under two years of
age; which include splinting, with a Frejka splint or Pavlik harness. Colonna or
Buck's skin traction is used in children 2 to 12 years of age, with a well-
padded spica cast applied simultaneously to the unaffected side

237) Colon cancer with stage 3 give the chemotherapy:


a. As soon as possible
b. 1 month
The correct answer is a (I'm not sure because MCQs not
complete )

Treatment for Stage 3 Colon Cancer

Treatment for stage 3 colon cancer generally consists of a surgical


resection followed by chemotherapy. In a surgical resection, a
surgeon removes the part of the colon affected by the tumor and
joins the remaining healthy sections together to form one long,
healthy piece.

The standard chemotherapy regimen used to be six months of


treatment with 5-FU and leucovorin, but that "cocktail" was
developed decades ago and is seldom used anymore. there are
many new chemotherapy regimens available for stage 3 colon
cancer.

……………………………………………………………………………………………
………………………………………

238) 56 y old present with vasomotor rhinitis


a. Local anti histamine
b. Local decongestion
c. Local steroid
d. Systemic antibiotic

The correct answer is b

Vasomotor rhinitis is a nonallergic condition that causes a


constant runny nose, sneezing, and nasal congestion.

……………………………………………………………………………………………
………………………………………

239) Sickle Cell Anemia give prophlaxis:


a. Penicillin
b. Iron
The correct answer is a

TREATMENT
■ Acute crisis: Analgesia and hydration.
■ Hydroxyurea to ↑the amount of fetal hemoglobin.
■ H. influenzae and pneumococcal vaccines; prophylactic penicillin
for
Children 5 years of age.
■ Acute chest syndrome: Respiratory support and exchange
transfusion.
240) diagnosis of thalasimia minor:
a. Hb A2 and Hb f
b. Microcytosis

the correct answer is a

……………………………………………………………………………………………
………………………………………

241) Pt. with MCV decrease and reticulocyt decrease iron


deficiency anaemia investigation:
a. Ferritin level and TIBC and serum iron

……………………………………………………………………………………………
………………………………………

242) Born BCG


a. 1month hepatitis b oral polio dpt
b. 2month s
c. 3month s
d. 9 to 12 month mmr
????? I can't understand th Q ( may the writer Q missed
some information )
243) E.histolytica cyst is destroyed by:

a. Freezing

b. Boiling

c. Iodine treatment

d. Chlorine

The correct answer is b

Amebiasis (or amoebiasis) is the name of the infection caused by E.


histolytica.
To help prevent infection:
• Avoid raw vegetables when in endemic areas as they may have been
fertilized
using human feces.
• Boil water or treat with iodine tablets
………………………………………………………………………………………………………
……………………………………………

244) Patient after accident, the left ribcage move inward


during inspiration and outward during expiration:

a. Flial chest

Flail chest is a clinical anatomic diagnosis noted in blunt trauma


patients with paradoxical or reverse motion of a chest wall
segment while spontaneously breathing. This clinical finding
disappears after intubation with positive pressure ventilation,
which occasionally results in a delayed diagnosis of the condition.
………………………………………………………………………………………………………
……………………………………………

245) Greatest risk of stroke:

a. DM

b. Elevated blood pressure

c. Family history of stroke

d. Hyperlipedemia

e. Smooking

the correct answer is b

246) Child has pallor , eats little meat , by investigation


:microcytic hypochromic anemia . what will you do:

a. Trial of iron therary

b. Multivitamin with iron daily

the correct answer is a ( I'm not sure vecause Q is not


complete )

………………………………………………………………………………………………………
……………………………………………

247) Treatment of mania that does not cause hepatotoxicity

a. Lithium
b. carbamazepine
c. valporic acid
d. lamotrigine

The correct anser is a

Treatment of mood disorder :

Lithium : hepatotoxicity
carbamezapine : agrnulocytosis
valporic asid : neural tube defect
………………………………………………………………………………………………………
……………………………………………

248) Sickle cell anemia patient , the macula is cherry red ,


and absence of afferent papillary light reflex
a. Retinal artery occlusion.
. b. veine artery occlusion

the correct answer is a , cherry red spot is sign of retinal


atery occlusion.

249) Inflammatory bowel disease is idiopathic but one of


following is possible underlying cause

a. Immunological

Inflammatory Bowel Disease Causes

Researchers do not yet know what causes inflammatory bowel


disease. Therefore, IBD is called an idiopathic disease (disease with
an unknown cause).

An unknown factor/agent (or a combination of factors) triggers


the body’s immune system to produce an inflammatory reaction in
the intestinal tract that continues without control. As a result of the
inflammatory reaction, the intestinal wall is damaged leading to
bloody diarrhea and abdominal pain.

………………………………………………………………………………………………………
……………………………………………

250) Patient present with high blood pressure (systolic 200) ,


tachycardia , mydriasis , sweating . what is the toxicity:

a. Anticholenergic
b. Sympathomimetic

c. Tricyclic antidepressant

d. Organophosphorous compounds

the correct answer b

Uses of Sympathomimetics: To treat and prevent


reversible bronchospasm associated with bronchial and
nocturnal (nighttime) asthma, chronic bronchitis,emphysema,
exercise-included bronchospasm and other obstructive airway
disease of the lungs.

To treat serious allegoric reactions (epinephrine injection only)

251) Treatment of Chlamydia with pregnancy:

a. Azithromycin

b. Erythromycin base

the correct answer b

For treatment of chlamydia during pregnancy, the CDC


recommends: -erythromycin base, 500
mg orally, four times a day for seven days; or amoxicillin (Amoxil), 500
mg, three times a day for seven days.
Alternatives include:
-Erythromycin base 250 mg, four times a day for 14 days;
-erythromycin ethylsuccinate 800 mg orally, four times a day for seven
days; or -erythromycin ethylsuccinate 400 mg orally,
four times a day for 14 days.

………………………………………………………………………………………………………
……………………………………………
252) the maximum body lenght will be reached after menarch
by

a -6 months

b. 1 year

A child will have also reached her final adult height about two
years after menarche.

………………………………………………………………………………………………………
……………………………………………

253) Patient developed sudden loss of vision bilaterally while


she was walking in the street , followed by numbness , the
subjective symptoms are different from objective , and
does not match anatomical , what is your diagnosis:

a. Conversion syndrome

Conversion disorder:
Symptoms or deficits of voluntary motor or sensory function (e.g.,
blindness, seizure) suggest a condition incompatible with medical
processes. Close temporal relationship to stress or intense
emotion. More common in young females and in lower
socioeconomic and less educated groups.
………………………………………………………………………………………………………
……………………………………………

254) After inflammatory phase of wound , there will be wound


healing by: a. If the wound is
clean
b. Angiogenesis
c. Epithelial tissue
. the correct answer is b

255) Old male with tender knee , pain , crepitus . the


diagnosis:

a. Osteorhritis
b. Ankylosin spondylitis
c. Rheumatoid

the correct answer a

Osteorhritis
OARTHRITIS (OA)
■ A chronic, noninflammatory arthritis of movable joints (e.g., DIP
joints).
Characterized by deterioration of the articular cartilage and
osteophyte
formation at joint surfaces.
■ Risk factors include a _ family history, obesity, and a history of
joint
trauma.
■ Hx/PE: Crepitus; decreaserange of motion (ROM); pain that
worsens with activity
and weight bearing but improves with rest.

………………………………………………………………………………………………………
……………………………………………

256) Mother has baby with cleft palate and asks you what is
the chance of having a second baby with cleft palate or
cleft lip ,

a. 25%
b. 50%
c. %1
d. 4%

the correct answer is d

………………………………………………………………………………………………………
……………………………………………-

257) 1 liter fluid deficit equals :


. a.1 kg
. Liter of fluid deficit equal 1 kg of fluid as hydration protocol
…………………………….
……………………………………………………………………………………………………...
.................

258) After accident patient with tachycardia, hypotension,


what will be your initial step:
a. Rapid IVF crystalloid
b. CT
Q not complete . management ABC then rapid IVF
crystalloid.

259) 6 years child was born to HBS positive mother is HBS


positive , he was only vaccinated by BCG after birth , what
you will give him now : a.
HBV + oral polio + DTP + hib
b. HBV + oral polio + dt + MMR +hib
c. HBV + oral polio + Dt + MMR

?????????? all the above are wrong

………………………………………………………………………………………………………
……………………………………………

260) Treatment of non inflammatory acne

a. Retinoic acid

Retinoids: Medicines structurally similar to vitamin A are useful


in preventing several types of acne lesions. Topical retinoids are
effective in treating the noninflammatory types of acne
(blackheads and whiteheads).

………………………………………………………………………………………………………
……………………………………………

261) Treatment of comedones:

a. Topical retinoids.

Comedones: The plural of comedo, the primary sign of acne,


consisting of a dilated (widened) hair follicle filled with keratin
squamae (skin debris), bacteria, and sebum (oil). Comedones may
be closed or open.

………………………………………………………………………………………………………
……………………………………………

262) Treatment of papules or pustules:

a.Topical benzoyl
b.Peroxide plus topical antibiotics, mainly clindamycin or
erythromycin. c. In severe cases,
intralesional steroid injection or oral antibiotics, such as
tetracycline or erythromycin may be added.

The correct answer b

TREATMENT
■ Mild acne: Topical clindamycin or erythromycin; benzoyl
peroxide; topical
retinoids.
■ Moderate acne: The above regimen plus oral antibiotics such
as tetracycline.
■ Severe nodulocystic acne: Oral isotretinoin (Accutane).

263) Which is not true In emergency management of stroke

a.Give IVF to avoid D5 50%


b.Give diazepam in convulsions
. c.Anticonvulsants not needed in if seizures
d.-Must correct electrolytes
e. Treat elevated blood pressure

the correct answer is c

………………………………………………………………………………………………………
……………………………………………

264) SCA complications in adults

a. Cerebral infarction
b. Cerebral hemorrhage

Neurological complications occur in 25% of patients, with transient


ischaemic attacks, fits, cerebral infarction, cerebral haemorrhage and coma.
Strokes occur in about
11% of patients under 20 years of age. The most common finding is
obstruction of a distal intracranial internal carotid artery or a proximal middle
cerebral artery. 10% of children
without neurological signs or symptoms have abnormal blood-flow velocity
indicative of clinically significant arterial stenosis; such patients have very
high risk of stroke. It has now been demonstrated that if children with
stenotic cranial artery lesions, as demonstrated on transcranial Doppler
ultrasonography, are maintained on a regular programme of transfusion that
is designed to suppress erythropoiesis so that no more than 30% of the
circulating red cells are their own, about 90% of strokes in such children
could be prevented.
………………………………………………………………………………………………………
……………………………………………

265) The most common risk for intracerebral stroke

a. Hypertension
266) The antidepressant used for secondary depression that
cause sexual dysfunction

a.Sertatline (SSRIs)
b. Imapiramine
c. Levofluxine

the correct answer a

………………………………………………………………………………………………………
……………………………………………

267) Previously healthy female patient presented to ER with


dysnea , anexiety , tremor , and she breath heavily , the
symptoms began 20 minutes before she came to ER , in the
hospital she developed numbness periorbital and in her
fingers , what you will do

a. Ask her to breath into a bag


. b. Take blood sample to look for alcohol toxicity

MCQs not complete


………………………………………………………………………………………………………
……………………………………………

268) What is the most important in councling

a. Exclude physical illness

b. Establishing rabbot

c. Family

d. Schedule appointement

?????

269) In breaking bad news

a. Find out how much the patient know


b. Find out how much the patient wants to know

the correct answer a

Robert Buckman's Six Step Protocol for Breaking Bad


News

1. Getting started.
2. Finding out how much the patient knows.

3. Finding out how much the patient wants to know .

4. Sharing the information.

5. Responding to the patients feelings .

6. Planning and follow-through.

………………………………………………………………………………………………………
…………………………………………

270) Patient with chest pain that aggrevated by couphing,


there is added sound on left sternal border .in ecg you will
find

a. St changes

b. Pr prolongation
c. Hypervoltage

the correct answer a . (percarditis)

………………………………………………………………………………………………………
…………………………………………

271) The most common site for visceral hemangioma is

a. Liver ( most common site for visceral hemangioma )

A hepatic hemangioma is the most common noncancerous tumor


of the liver. It is believed to be a birth defect. Hepatic
hemangiomas can occur at any time, but are most common in
people in their 30s - 50s. Women are affected more often than
men, and usually have bigger tumors than men. Babies may
develop a type of hepatic hemangioma called benign infantile
hemangioendothelioma (also called multinodular hepatic
hemangiomatosis). This rare, noncancerous tumor has been linked
to high rates of heart failure and death in infants. Infants are
usually diagnosed by the time they are 6 months old.

272) Child with large periorbital hemangioma , if this


hemangioma cause obstruction to vision , when will be
permenant decrease in visual acuity

a. After obstruction by one day

b. By 1 week

c. By 3 months

d. By 6 months

?????????

………………………………………………………………………………………………………
…………………………………………

273) The symptoms of soft tissue sarcoma

a. Progressive enlarging mass ( on growing )

In their early stages, soft tissue sarcomas usually do not


cause symptoms. Because soft tissue is relatively elastic,
tumors can grow rather large, pushing aside normal tissue,
before they are felt or cause any problems. The first
noticeable symptom is usually a painless lump or swelling.
As the tumor grows, it may cause other symptoms, such as
pain or soreness, as it presses against nearby nerves and
muscles. If in the abdomen it can cause abdominal pains
commonly mistaken for menstrual cramps, indigestion, or
cause constipation

………………………………………………………………………………………………………
…………………………………………

274) 35 year female with bilateral breast pain , that decrease


after menstruation , the breast is nodular with prominent 3
cm mass subareolar , axillary lymph nodes are not enlarged
, what you will do

a. Mammography followed by US

b. See her next cycle

c. Fine needle biopsy followed by tissue studies

the correct answer is b

275) Pregnant with bleeding for 12 hours and tissue , the


cervix is 1 cm

a. Complete abortion
. b. Incomplete abortion
. c. Molar pregnancy

the correct answer is b


276) 2 month infant with vomiting after each meal , he is in
50 centile , He passed meconium early and stool , diagnosis
is

a. Midgut volvulus
b. Meconium ileus (not passing early meconium and
stool)

c. Hischsprung disease (not passing early meconium


and stool)

I think this case is pyloric stenosis. If not mention choose a

………………………………………………………………………………………………………
…………………………………………

277) Patient with dysphagia to solid and liquid , and regurg ,


by barium there is non peristalsis dilatation of osophagus
and air-fluid level and tapering end.diagnosis is

a. Osophageal spasm

b. Achalasia

c. Osophageal cancer

the correct answer is b

………………………………………………………………………………………………………
…………………………………………

278) The most common cause of cough in adults is

a. Asthma
b. GERD
c. Postnasal drip

The correct answer c

The most common causes of acute cough is respiratory viral


infection. The most common cause of chronic cough are postnasal
drip, asthma, and acid reflux from the stomach. These three causes
are responsible for up to 90 percent of all cases of chronic cough.

279) Girl with amenorrhea for many months . BMI is 20 and is


stable over last 5 years . diagnosis
a.Eating disorder

b.Pituitary adenoma

………………………………………………………………………………………………………
…………………………………………

280) Old female with itching of vulva , by examination there


is pale and thin vagina , no discharge . what is management

a. Estrogen cream

b. Corticosteroid cream

c. Fluconazole

the correct answer is a

Some women will opt for prescription medications as they go


through menopause. The most common prescriptions include:
1 -Hormone replacement therapy or anti-depressants to minimize
hot flashes. 2 -Fosamax or Actonel (non-hormonal
medications) to reduce bone loss and reduce the risk of fractures .
3 -Selective estrogen receptor modulators (SERMs), which mimic
estrogen's beneficial effects on bone density.
4-Vaginal estrogen, administered locally, to relieve vaginal dryness
and discomfort during intercourse

………………………………………………………………………………………………………
…………………………………………

281) Patient with dysuria , frequency , urgency , but no flank


pain , what is the treatment

a. Ciprofloxacin po od for 3-5 days

b. Norfocin po od for 7 – 14 days

the correct answer is b

282) Patient with flank pain , fever , vomiting , treatment is

a. Hospitalization and intravenous antibiotics and fluid


This is most likely a case of pyelonephritis which need urgent
hospitalization

………………………………………………………………………………………………………
…………………………………………

283) Patient present with mid face pain , erethematous


lesions and vesicles on periorbital and forehead , the pain
is at nose , nose is erythematous . what is diagnosis

a. Roseola
. b. HSV
. c. Herpes zoster

the correct answer is c

………………………………………………………………………………………………………
…………………………………………

284) Male with itching in groin erythematous lesions and


some have clear centers , what is diagnosis :
a. Psoriasis
b. Tinea curis
c. Erythrasma

the correct answer is b

Patients with tinea cruris report pruritus and rash in the groin. A
history of previous episodes of a similar problem usually is elicited.
Additional historical information in patients with tinea cruris may
include recently visiting a tropical climate, wearing tight-fitting
clothes (including bathing suits) for extended periods, sharing
clothing with others, participating in sports, or coexisting diabetes
mellitus or obesity. Prison inmates, members of the armed forces,
members of athletic teams, and people who wear tight clothing
may be subject to independent or additional risk for
dermatophytosis.Large patches of erythema with central clearing
are centered on the inguinal creases and extend distally down the
medial aspects of the thighs and proximally to the lower abdomen
and pubic area.-
285) Vasoconstrictive nasal drops complication

a. Rebound phenomenon

………………………………………………………………………………………………………
…………………………………………

286) The useful excurcise for osteoarthritis in old age to


maintain muscle and bone Low resistance and high repetion
weight training:

a. Conditioning and low repetion weight training

b. Walking and weight exercise

the correct answer is b

Exercise is one of the best treatments for osteoarthritis. The


best exercises for osteoarthritis suffers depend on what joints
are affected. Swimming, walking, and cycling are often the best
exercises for people with osteoarthritis. Try to get thirty
minutes of exercise five times per week. The key is to start
slowly.

………………………………………………………………………………………………………
…………………………………………

287) Unilateral worsening headach , nausea , excacerbeted by


movement and aggrevated by light in 17 old girl.

a. Migraine (Photophobia, vomiting)


b. Cluster

the correct answer is a


288) Diet supplement for osteoarthritis

a. Ginger

A large number of dietary supplements are promoted to patients


with osteoarthritis and as many as one third of those patients have
used a supplement to treat their condition. Glucosamine-containing
supplements are among the most commonly used products for
osteoarthritis. Although the evidence is not entirely consistent,
most research suggests that glucosamine sulfate can improve
symptoms of pain related to osteoarthritis, as well as slow disease
progression in patients with osteoarthritis of the knee. Chondroitin
sulfate also appears to reduce osteoarthritis symptoms and is often
combined with glucosamine, but there is no reliable evidence that
the combination is more effective than either agent alone. S-
adenosylmethionine may reduce pain but high costs and product
quality issues limit its use. Several other supplements are
promoted for treating osteoarthritis, such as
methylsulfonylmethane, Harpagophytum procumbens (devil's
claw), Curcuma longa (turmeric), and Zingiber officinale
(ginger), but there is insufficient reliable evidence regarding long-
term safety or effectiveness.

………………………………………………………………………………………………………
………………………………………

289) Old male with abdominal pain , nausea , WBC 7. What is


true about appendicitis in elderly?

a. Ct not usefull for diagnosis.


b. WBC is often normal.
c. Rupture is common
d. If there is no fever the diagnosis of appendicitis is
unlikely e. Anemia is
common

the correct answer is c

Appendicitis in elderly
Appendicitis is a less common cause of abdominal pain in elderly patients than in younger patients, but
the incidence among elderly patients appears to be rising. Only approximately 10% of cases of acute
appendicitis occur in patients older than 60 years, whereas one half of all deaths from appendicitis
occur in this age group. The rate of perforation in elderly patients is approximately 50%, 5 times higher
than in younger adults. This is largely because 75% of elderly patients wait more than 24 hours to
seek medical attention. The diagnosis can be difficult to make, since more than one half of patients in
this age group do not present with fever or leukocytosis. Further confusing the picture, approximately
one third do not localize pain to the right lower quadrant, and one fourth do not have appreciable right
lower quadrant tenderness. Only 20% of elderly patients present with anorexia, fever, right lower
quadrant pain, and leukocytosis. The initial diagnosis is incorrect in 40-50% of patients in this age
range . ( perforation is the most common complication of appendicitis )

290) Old patient with bilateral enlarged knee , no history of


trauma , no tenderness , normal ESR and C-reactive
proteins . the diagnosis is

a. Osteoarthritis
b. Gout
c. Infectous arthritis

the correct answer is a ( I'm not sure )

………………………………………………………………………………………………………
………………………………………

291) Patient has decrease visual acuity bilateral , but more in


rt side , visual field is not affected , in fundus there is
irregular pigmentations and early cataract formation .

a. Refer to ophthalmologist for laser therapy

b. Refer to ophthalmologist for cataract surgery


??????????

the correct answer is a

………………………………………………………………………………………………………
………………………………………

292) What is the most common treatment for juvenile


rheumatoid arthritis

a. Intraarticular injection of steroid

b. Oral steroid

c. Paracetamol

d. penicillamine

e. Asprin
the correct answer is e

the most common important for JRA is ( NSAID )

………………………………………………………………………………………………………
……………………………………

293) Which of the following decrease mortality after MI

a. Metoprolol
b. Nitroglycerine
c. Thiazide
d. Morphine

the correct answer is a


only b-blocker ans ASA are mortality benefit for treatment of
angina

294) The cardiac arrest in children is uncommon but if occur it


will be due to

a. Primary

b. Respiratory arrest

c. hypovolemic shock

d. neurogenic shock

the correct answer is b

Progressive respiratory insufficiency accounts for 60% of


all paediatric arrests.

………………………………………………………………………………………………………
………………………………………

295) Old female with recurrent fracture , Vitamen D


insufeciency and smoker . which exogenous factor has the
gretest exogenous side effect on osteoporosis.

a. Old age

b. Smoking

c. . Vit D insufeciency

d. Continue smoking
e. Recurrent fracture

the correct answer is b

………………………………………………………………………………………………………
………………………………………

296) patient presented with sudden chest pain and dysnea ,


tactile vocal fremitus and chest movemebt is decreased , by
x-ray there is decreased pulmonary marking in left side ,
diagnosis

a. atelectasis of left lung

b. spontaneous pneumothorax

c. pulmonary embolism

??????????

297) boy after running for hours , has pain in knee and mass
on upper surface of tibia

a. Osgood scatter disease

b. Iliotibial band

the correct answer is a

………………………………………………………………………………………………………
………………………………………

298) pancreatitis

a. Amylase is slowly rising but remain for days

b. Amylase is more specific but less sensitive than


lipase

c. Ranson criteria has severity (predictive) in acute


pancreatitis

d. Pain is increased by sitting and relieved by lying


down
e. Contraceptive pills is associated

the correct answer is c

The Ranson and Glasgow scoring systems are based on such


parameters and have been shown to have an 80% sensitivity for
predicting a severe attack, although only after 48 hours following
presentation.

risk mortality is 20% with 3-4 signs, 40% with 5-6 signs,
100% 7 signs.
299) Patient has fever , night sweating , bloody sputum ,
weight loss , ppd test was positive . x-ray show infiltrate in
apex of lung , ppd test is now reactionary , diagnosis

a. Activation of primary TB

b. sarcoidosis

c. Case control is

d. Backward study

the correct answer is a

The tuberculosis skin test (also known as the tuberculin test or


PPD test) is a test used to determine if someone has developed
an immune response to the bacterium that causes tuberculosis
(TB).

………………………………………………………………………………………………………
……………………………………………..

300) patient with DM presented with limited or decreased


range of movement passive and active of all directions of
shoulder
a. frozen shoulder
b. impingment syndrome
c. osteoarthritis

the correct answer is a


………………………………………………………………………………………………………
……………………………………………..

301) 48 years old with irregular menses presented with


fatigue and no menstruation for 3 months with increased
pigmentation around the vaginal area with no other
symptoms. ur next step would be
a. reassure the patient
b. do a pregnancy test
c. do ultrasound

the correct answer b

302) 3 years old presented with shortness of breath and


cough at night which resolved by itself in 2 days. he has Hx
of rash on his hands and allergic rhinitis. he most likely had
a. croup
b. bronchial asthama
c. epilotitis

????????

Croup Epiglottitis

Onset Days Hours

Flu-like symptoms Yes No


Cough Sever Absent

Able to drink Yes No

Drooling saliva No Yes

Fever <38 >38

Stridor Harsh Soft

Voice Hoarse muffed

………………………………………………………………………………………………………
…………………………………………

303) a man went on vacation. he noticed a white patch in his


chest which became more clear after getting a sun tan
which was spread on his chest.ur Dx is
a . pytriasis versicolor
b. vitilligo
c. pytriasis roscea

the correct answer a


………………………………………………………………………………………………………
……………………………………………..304) a 4 years old presented
with 2 day history of shortness of breath a seal like cough with
no sputum and mild fever. on examination he did not look I'll
or in distress

a. acute epilossitis
b. croup
c. angioedema

The correct answer b

304)65 years old came with knee pain. and limited movement.
on examination had crepition on knee. dx
a. rhuematiod arthritis
b. osteoporosis
c. osteoarthritis

the correct answer c

………………………………………………………………………………………………………
……………………………………………..
305) which vitamin is given to new born to stop bleeding
a. vit. A
b. vit. D
c. vit. K
d. vit E
e. vit C

the correct answer c

………………………………………………………………………………………………………
……………………………………………..

306) a child came with congested throught and mild bulging of


tympanic membrane ur Dx was URTI. The
?????????

………………………………………………………………………………………………………
……………………………………………..

307) a child presented with erythematous pharynx, with


cervical lymph nodes and rapid strplysin test negative and
low grade fever with positive EBV. it next step
a. give antibiotics and anti pyretic
b. give anti pyretic and fluids
d. culture and sensitivity

the correct answer a


………………………………………………………………………………………………………
……………………………………………..

308) child with low grade fever and congested throat, negative
ASO and positive EBV. he has
a. infectous mononucleosis
b. URTI

the correct answer is a


………………………………………………………………………………………………………
……………………………………………..
309) the most common cause of intracerebral or
intraparynchimal bleeding is.
a. Hypertensive angiopathy
b. annurysm
c. AV malformation

the correct answer is a


310) young adult Sickle cell patients are commonly affected
with
a. dementia
b. multiple cerebral infarcts

the correct answer b

………………………………………………………………………………………………………
……………………………………………..
311) 80 years old living in nursing home for the last 3 months.
his wife died 6 months ago and he had a cornary artery
disease in the last month. he is now forgetful especially of
short term memory and decrease eye contact with and loss
of interest. dx
a. alzihiemer
b. depression
c. hypothyroidism

the correct answer a , I think Q is not complete

………………………………………………………………………………………………………
……………………………………………..

312) thyroid cancer can be from


a. hypothyroidism
b. graves disease
c. toxic nodule

?????????

………………………………………………………………………………………………………
……………………………………………..

313) a mother came with her son who is 7 years old very active
never sitting in class and with poor concentration. ur
management would be.
a. olanzipine
b. amitilyne
c. aloxane

the correct answer is b ( I'm not sure )

this case is "attention –deficit hyperactivity disorder"


treatment:
1- Methylphenidate
2- Dextroamphetamine
3- Aderall
4- Atomoxetine
5- Pemoline
6- Antidepressant ( SSRI )

314) from the graph above which of the following statement is


true
a. osteoprosis affecting majority of women above 80
b. 20% of women under 70 are affected
?????????
………………………………………………………………………………………………………
……………………………………………..

315) a patient presented with progressive weakness on


swollowibg with diplopia and fatigability. the most likely
underlying cause of her disease is.
a. antibody against acetylcholine receptors

………………………………………………………………………………………………………
……………………………………………..
316) an 69 year old non diabetic. with mild hypertension and
no hx of Coronary heart desease. the best drug in
treatment is.
a. thiazides
b. ACEI
c. ARB
d. CCB

the correct answer a


………………………………………………………………………………………………………
……………………………………………..

317) which of the following anti hypertensive is


contraindicated for an uncontrolled diabetic patient
a. hydrochlorothiazide
b. Losartan
c. hydralszine
d. spironolactone

the correct answer is a


-side effect of thiazide : hyperglycemia
………………………………………………………………………………………………………
……………………………………………..

318) a wound stays in it's primary inflammation untill


a. Escher formation
b. epitheliazation
c. after 24 hours
d. wound cleaning

the correct answer d


………………………………………………………………………………………………………
……………………………………………..
319) 70 years old male patient with mild urinary dripping and
hesitency ur Dx is mild BPH. ur next step in management is
a. transurethral retrograde prostatectomy
b. start on medication
c. open prostatectomy
the correct answer b
320) most common cause of secondary HTN is
a. Renal disease - Renal parenchymal disease

………………………………………………………………………………………………………
……………………………………………..

321) one of the following food is known to reduce cancer


a. fibers
………………………………………………………………………………………………………
……………………………………………..

322) smoking is a definitive risk of


a. sq.c.ca of bladder
b. liver
c. breast

the correct answer is a

………………………………………………………………………………………………………
……………………………………………..

323) exercise recommended for patients with CAD. is


a. isometric
b. isotonic
c. yoga

‫؟؟؟؟؟؟؟؟؟؟‬

………………………………………………………………………………………………………
……………………………………………..

324)what is the definitive treatment of frostbites


a, Rewarming

………………………………………………………………………………………………………
……………………………………………..
325) the most common cause of failure to thrive in pediatric is
a. malnutrition

………………………………………………………………………………………………………
……………………………………………..

326) which of the following causes the highest maternal


mortality in pregnancy
a. toxoplasma
b. hyperbilirubenia
c. ????
" The major causes of maternal death are bacterial infection,
variants of gestational hypertension including pre-eclampsia and
HELLP syndrome, obstetrical hemorrhage, ectopic pregnancy,
puerperal sepsis (childbed fever),

327) Ttt of frostbite

a. Immersion in water 40-45 C

b. Debridement

c. Leave it at room tepmrature

The correct answer is a

………………………………………………………………………………………………………
……………………………………………..
328) Case about a child both RBS, FBS are elevated so he has
DM1…what's the type of HLA

a.DR3

b.DR4

c.DR5

d.DR6

e.DR7

the correct answer is a

Types of HLA :

DR2 : Good's pasture syndrome & multiple myeloma

DR3 : D.M , SLE , & grave's disease

DR27 : ankylosing spondylaitis & reiter's syndrome

B51 : behceat's disease

D11 : hashimot's disease


…….
………………………………………………………………………………………………………
……………………………………..
329) Definition of PPV ?

Predictive positive value ( PPV ) : proportion of people with a


positive test who have s a disease : a/ (a+b)

………………………………………………………………………………………………………
……………………………………………..
330) What's the most common case in PHC centers

a.UTI

b.HTN

c. Coryza

the correct answer is c

331) all of the following will decrease pt compliance except:

a. involve pt in the plan

b. make simplified regimen

c. give easy written instructions

d. make appointments flexible

e. warn the patient about the danger of missing a pill

the correct answer is e

………………………………………………………………………………………………………
……………………………………………..
332) Case about old diabetic patient who still have
hyperglycemia despite increase insulin dose…the problem with
insulin in obese patients is
a. Post receptor resistance

?????? Q NOT COMPLETE

………………………………………………………………………………………………………
…………………………………………….

333) Case about a child with drooling, fever, barking cough in


sitting position, dx:

a. Croup

b. Broncholities

c. Pneumonia

The correct answer is a

………………………………………………………………………………………………………
……………………………………………..
334) Mother came to you after her son had hematoma under
the nail dt injury:

a. Send home with a pad on the head

b. Send home with acetaminophen

c. Do wedge resection

d. Evacuate the hematoma

The correct answer is d

This case is ( subungual hematoma )


335) What vaccine u'll give to a SCD ( sickle cell disease ) child

a.HBV

b. H.influenza

c. pneumococcal

d. both A and B

e. all of the above

the correct answer is c

………………………………………………………………………………………………………
……………………………………………..

336) a mother with HBsAg positive came with her child 6 yrs
old who has HBsAg +ve what will you give him:

a. oral polio, DTP, MMR

the correct answer is a

other choices all had HBV vaccine which u'll not give

………………………………………………………………………………………………………
……………………………………………..

337) Patient wil LLQ pain, vomiting, fever, high WBC (17.000),
tenderness and rebound tenderness

a. Diverticulitis
b. Sigmoid volvulus

c. Appendicitis

d. Toxic enteritis

The correct answer is a

………………………………………………………………………………………………………
……………………………………………..

338) What's the organism responsible for psuedomembranous


colitis:

a. Pseudomonas

b. Colisteridum

c. E.coli

d. Enterococcus fecalis

The correct answer is b

339) Mother came with her child who had botillism, what you
will advice her:

a. Never eat canned food again

b. Store canned food at home

c. Boil canned food for 40-50 min

d. Check expiry date of canned food

The correct answer is d


………………………………………………………………………………………………………
……………………………………………..

340) Old pt presented with abdominal pain, back pain,


pulsatile abdomen what's the step to confirm dx: this is a case
of aortic aneurysm

a. Abdominal US

b. Abdominal CT

c. Abdominal MRI

The correct answer is a b

-initial investigation US – CONFIRM by CT

………………………………………………………………………………………………………
……………………………………………..

341) 18 months old came with bite by her brother, what you
will do:

a. Give augminten

b. Give titunus toxoid

c. Suture

Q not complete

………………………………………………………………………………………………………
……………………………………………..

342) 19 yrs old after bike accident, he cant bring the spoon
infront of himself to eat, lesion is in:

a. Temporal lobe

b. Cerebellum

c. Parietal lobe
d. Occipital lobe

The correct answer is b

343) How to dx DVT:

a. Contrast venography

b. Duplex US

The correct answer is b

………………………………………………………………………………………………………
……………………………………………..

344) Pt came after RTA, GCS 14, near complete amputation of


the arm, 1st step:

a. Secure air way

b. Tourniquet on the arm

The correct answer is a

………………………………………………………………………………………………………
……………………………………………..

345) Pt with hx of prolonged heavey bleeding 2 hrs post


partum, you will give:

a. Ringers lactate

b. NS
c. NS+ packed erythrocytes

The correct answer is c ( Q NOT COMPLETE )

………………………………………………………………………………………………………
……………………………………………..
346) What is special about placenta abruption:

a. Abnormal uterine contractions

b. PV bleeding

c. Fetal distress

The correct answer is b , PA : dark , painful vaginal bleeding and


complication is fetal hypoxia

………………………………………………………………………………………………………
……………………………………………..

347) Pt 34 wks, hx of PV bleeding for many hrs, dark blood,


abdominal tenderness, FHR 120, uterine contractions every 3
min, Dx:

a. Abruption placenta

348) One of the steps in managing epistaxis:

a. Packing the nose


b. Press the fleshy parts of nostrils

c. Put patient of lateral lying position

The correct answer is b

………………………………………………………………………………………………………
……………………………………………..

349) Pt with hx of diarrhea, abdominal pain, agitation,


headache, dizziness, weakness, pulstile thyroid, unsteady gate.
Examination was normal. Dx:

a. Hypochondriasis

b. Somatization disorder

c. Thyroid Ca

d. Anxiety

The correct answer is b

………………………………………………………………………………………………………
……………………………………………..

350) Child with headache that increase by changing head


postion, unilateral, photophobia

a. Sinus headache

b. Migraine

c. Cluster headache

d. Tension headache

The correct answer is b

………………………………………………………………………………………………………
……………………………………………..
351) Ttt of isolated fracture of femur (repeated) ???

………………………………………………………………………………………………………
………………………………………………

352) Younge pt with pain in LL after running 3 kgs, more at


night, swelling, XR was NL

a. Stress fracture

353) HTN pt, with decrease vision, fundal exam showed


increase cupping of optic disc dx:

a. Open angle glaucoma

b. Closed angle glaucoma

c. Cataract

d. HTN changes

The correct answer is a ( I'm not sure )

………………………………………………………………………………………………………
……………………………………………..

354) Pt with unilateral red eye, tearing, clear tears no


swelling or discharge:

a. Give topical antihistamin

b. Give topical AB

c. Give systemic AB

d. Reassurance

The correct answer is d


………………………………………………………………………………………………………
……………………………………………..

355) Business man went to Pakistan, came with bloody


diarrhea, stool examination showed trophozoite with RBC
inclusion, Dx:

a. Amebic desyntry (entamoeba histlolytica )

………………………………………………………………………………………………………
……………………………………………..

356) 5 yrs old child with abdominal pain after 2 wks of URTI,
HB 8, retics 12% WBC NL peripheral blood smear showed target
cells, RBC inclusions dx:

a. SCA (the only hemolytic anemia in the answers)

?????

………………………………………………………………………………………………………
……………………………………………..

357) Child had hb electrophoresis showed hb 2%

a. HBA 40 HBS 35 HBF 5 HBA2

b. Sickle cell trait

c. Thalacemia major

d. Minor

?????

358) Younge pt with hx of cough, chest pain, fever CXR


showed RT lower lobe infiltrate:

a. Amoxicillin
b. Ceferuxim

c. Emipenim

d. Ciprofloxacin

??????

The correct answer is d (I'm not sure )

………………………………………………………………………………………………………
……………………………………………..

359) Best thing to reduce mortality rate in COPD:

a. Home O2 therapy

b. Enalipril

c. Stop smoking

The correct answer is c

………………………………………………………………………………………………………
……………………………………………..

360) Drug that will delay need of surgery in AR:

a. digoxin

b. verapamil

c. nefidipin

d. enalpril

the correct answer is c

………………………………………………………………………………………………………
……………………………………………..
361) Child with skin rash, pericarditis, arthritis dx:

Kawasaki

………………………………………………………………………………………………………
……………………………………………..
362) Pic of skin with purple flat topped polygonal papules, dx:

a. Lichen plannus

………………………………………………………………………………………………………
……………………………………………..

363) Child with atopic dermatitis, what you will give other
than cortisone

a. There were many drugs, steroids and AB and only one


strange name maybe moisturizer. That's the answer

364) A man had increase shoe size and jaw, the responsible is:

a. ACTH

b. Somato??

c. TSH

d. Cortisone

The correct answer is b

………………………………………………………………………………………………………
……………………………………………..

365) The best advice to patient travelling is:

a. Boiled water

b. Ice

c. Water

d. Salad and under cooked sea shells


The correct answer is a

………………………………………………………………………………………………………
……………………………………………..

366) Seldinfil is contraindicated with:

a. Nitrate

b. Methyldopa

c. Gabapentine

The correct answer is a

………………………………………………………………………………………………………
……………………………………………..

367) Pt with TB, had ocular toxicity symptoms, the drug


responsible is:

a. INH

b. Ethambutol

c. Rifampicin

d. Streptomycin

The correct answer is b

INH : peripheral neuritis and hepatitis . so add ( B6 )

Ethambutol : optic neuritis

Rifampicine : body fluid organe


368) Another pt treated for TB started to develop numbness,
the vit deficient is:

a. Thiamin

b. Niacin

c. Pyridoxine

d. Vit C

THE correct answer is c

………………………………………………………………………………………………………
……………………………………………..

369) Pt with bilateral breast disease, dx:

a. Paget

b. Papilloma

c. Meduallary

d. Lobular

………………………………………………………………………………………………………
……………………………………………..

370) Child fell on her elbow and had abrasion, now swelling is
more, tenderness, redness, swelling is demarcated (they gave
dimensions) child has fever. Dx:

a. Gonoccal arthritis

b. Synovitis

c. Cellulitis of elbow

The correct answer is c


………………………………………………………………………………………………………
……………………………………………..

371) About burn

a. You will give 1\2 fluid in the 1st 8 hrs

b. 1\4 in the 1st 8 hrs

The correct answer is a

Parkland formula : fluid in first 24 hours = 4 * wt. in kg * % BSA

50% in first 8 hours and another in next 16 hours

372) You r supposed to keep a child NPO he's 25 kgs, how


much you will give:

a. 1300

b. 1400

c. 1500

d. 1600

The correct answer is d

For the first 10 kg of weight, a child needs 100 mL per kg of


weight.
For the next 10 kg of weight (11-20 kg), a child only needs 50 mL
per kg of weight.
for anything over 20 kg (21 kg of weight and higher), the child
only needs 20 mL per kg of weight.
Take a 35 kg child, for example. He needs 1000 mL for his first 10 kg of
weight (10 kg x 100 mL), 500 mL for his second 10 kg of weight (10 kg
x 50 mL), and 20 mL/kg for any weight above 20 kg (15 kg x 20 mL). A
35 kg child, therefore, needs approximately 1800mL of water or free
liquids.
………………………………………………………………………………………………………
……………………………………………..

373) Young patient with pharyngitis, inflammation of oral


mucosa and lips that has whitish cover and erythmatous base,
febrile, splenomegaly. Dx: (this is infectious mono)

a. Scarlet fever

b. EBV

c. HZV

The correct answer is b

………………………………………………………………………………………………………
……………………………………………..

374) Female with greenish vaginal discharge, red cervix. Dx:

Trchimoniosis

………………………………………………………………………………………………………
……………………………………………..

375) Another female with malodorous discharge and pain


maybe. Dx:

Bacterial vaginosis

………………………………………………………………………………………………………
……………………………………………..
376) The best way to reduce the weight in children is:

a. stop fat intake

b. Decrease calories intake

c. Drink a lot of water


The correct answer is b

377) You have to advice a teenager that mainly eats fast food
to take:

a. Folic acid and Ca

b. Vit C and Ca

c. Ca alone

The correct answer is c ( I'm not sure )

………………………………………………………………………………………………………
……………………………………………..

378) Patient has symptoms of infection, desquamation of


hands and feet, BP 170\110 dx:

a. Syphilis

b. Toxic shock syndrome

c. Scarlet fever

The correct answer is b

Toxic shock syndrome :

Caused by S.aureus , often with 5 days of onset of menestral


period in women who have used tampons. Feature: abrupt fever
( 39 c or more ), vomiting, diffuse macular erythematous rash,
desquamation especially in palms and soles, nonpurulent
conjunctivitis. Diagnosis : blood culture are –ve .so, diagnosis by
clinical. Treatment : 1st step rehydration and antibiotic

………………………………………………………………………………………………………
……………………………………………..
379) Snellin chart, if patient can read up to line 3, how much
is his vision loss :
Q not complete:

but this pt. see letters at 20 feet , where normal person see it at 70
feet.

380) I cant remember the case but gram +ve cocci were
isolated, dx is sterpt bcz it was the only gm +ve cocci.

………………………………………………………………………………………………………
……………………………………………..

381) Diabetic mother asking about risk of diabetes related


congenital defect. It will be in:

a. 1st trimester

b. 2nd trimester

c. 3rd

The correct answer is a (I'm not sure )

………………………………………………………………………………………………………
……………………………………………..

382) If diabetic mother blood sugar is always high despite of


insulin, neonate complication will mostly be:
a. Maternal hyperglycemia

b. Maternal hypoglycemia

c. Neonatal hypoglycemia

d. Neonatal hyperglycemia

The correct answer is c

………………………………………………………………………………………………………
……………………………………………..

383) Obssive neurosis patients will have:

a. Major depression

b. Lake of insight

c. Schizophrenia

The correct answer is a

………………………………………………………………………………………………………
……………………………………………..

384) Before giving bipolar patient lithium you will do all of the
following except:

a. TFT

b. LFT

c. RFT

d. Pregnancy test

The correct answer is b

385) Patient came with PV small bleeding (she fell down and
felt decrease in fetal movement but on examination baby was
ok) all her 3 previous pregnancies were normal, you will do:
a. Immediate CS

b. Careful observation of the bleeding

c. Medication (strange name)

d. Mg sulphate

Q not complete , depend on date of gestation.

………………………………………………………………………………………………………
……………………………………………..
386) Pt came to you missing her period for 7 wks, she had
minimal bleeding and abdominal pain, +ve home pregnancy
test, 1st thing to order is:

a. BHCG

b. US

c. Drugs maybe

The correct answer is a

………………………………………………………………………………………………………
……………………………………………..

387) Patient came to you and you suspect pre eclampisa,


which of the following will make it most likely:

a. Elevated blood pressure

b. Decrease fetal movement

c. ??

The correct answer is a

………………………………………………………………………………………………………
……………………………………………..

388) Old patient male, hematuria, passing red clots and RT


testicular pain:
a. Testicular Ca

b. RCC ( renal cell carcinoma )

c. Cystitis

The correct answer is b

389) to get more information from the pt


a. open end question

( avoid leading question to get more information )

………………………………………………………………………………………………………
……………………………………………..
390) gingivits most likely cause
a.HSV

( q not complete but : The most common cause of gingivitis is poor


oral hygiene that encourages plaque to form.)

………………………………………………………………………………………………………
……………………………………………..
391) watery discharge from eye , conjunctivitis treated by
a. topical corticosteroid

(Its symptoms include watery discharge and variable itch. The


infection usually begins with one eye, but may spread easily to the
other

Ttt: cold compresses] and artificial tears)

………………………………………………………………………………………………………
……………………………………………..
392) blow out fracture eyelid swelling , redness other syptoms
a. present air fluid level
b. enopthalmos

The correct answer is b


its enophaloms , other name of blow out orbital floor fracture

………………………………………………………………………………………………………
……………………………………………..
393) drinking of dirty water causes
a. heptitis A
b. B
c. C
d. D

The correct answer is a

………………………………………………………………………………………………………
……………………………………………..

394) giemsa stained blood film


a. malaria

395) regarding peritonitis

a.Complicated appendectomy the cause is anerobe


organism
b. rigidity and the cause is paralytic ileus
c. can be caused by chemical erosions.

The correct answer is c

………………………………………………………………………………………………………
……………………………………………..
396) fraucture of humerus associated with
a. radial N injury

b. median nerve injury


the correct answer is a ( radial groove )

………………………………………………………………………………………………………
……………………………………………..

397) there is case i cant remember they asked about diagnosis


of acute lymphocytic leukemia ALL

The total number of white blood cells may be decreased, normal, or


increased, but the number of red blood cells and platelets is almost
always decreased. In addition, very immature white blood cells
(blasts) are present in blood samples examined under a
microscope.

A bone marrow biopsyis almost always done to confirm the


diagnosis and to distinguish ALL from other types of leukemia

………………………………………………………………………………………………………
……………………………………………..
398) diagnosis of hemochromatosis
a. serum ferritin?!

Hemochromatosis is suggested by a persistently elevated


transferrin saturation in the absence of other causes of iron
overload. It is the initial test of choice .

Ferritin concentration can be high in other conditions, such as


infections, inflammations, and liver disease .

Ferritin levels are less sensitive than transferrin saturation in


screening tests for hemochromatosis .

399) kawasaki disease associated with


a.strawberry tongue
 Kawasaki disease

Multisystem acute vasculitis that primary affected young


children. Fever plus four or more of the following criteria for
diagnosis:

1-fever > 40 C for at least five days

2-bilateral, nonexudative, painless conjunctivitis

3-polymorphous rash ( primarily truncal )

4-cervical lymphadenopathy ( often painful and unilateral )

5- diffuse mucous membrane erythema ( strawberry


tongue ) , dry red

6- erythema of palm and sole

7- other maindfestation : gallbladder hydrops, hepatitis, arthritis

Untreated Kawasaki diease can lead to coronary aneurysms and


even MI

Treatment :

1-high dos ASA ( for fever and inflammation) & IVIG ( to prevent
aneurrysmal )

2-referral to pediatric cardiologist

………………………………………………………………………………………………………
……………………………………………..
400) most common tumor in children
a.ALL

b. rabdomyosarcoma

c. wilm's tumor

the correct answer is a

-ALL : most common childhood tumor


-rabdomyosarcoma : most common soft tissue tumor

-wilm's tumor: most common intra- abdominal childhood tuomr

………………………………………………………………………………………………………
……………………………………………..
401) Most common intra- abdominal tumor in children:

a.wilm's tumor

b. lymphoma

the correct answer is a

the most common intra-abdominal tumor in children are


neruoblastoma and wilm's tumor.

402) pt c/o of hypopigmentful skin , nerve thicking diagnosis


a.leprosy

Leprosy : chronic granulomatous infection caused by


Mycobacterium leprae, acid-fast bacillus preferentially affecting cooler
regions of body ( e.g skin , mucousmembrane, peripheral nerve) .
diagnosis by skin lesion ( hypopigmentation or hyperpigmentation
macule ) and enlarge nerves eith sensory loss.
………………………………………………………………………………………………………
……………………………………………..
403) regrading COPD to reduce complication we should give
a. theophyline
b. pneumococcal vaccine

c. smoking cessation.

The correcr answer is c

………………………………………………………………………………………………………
……………………………………………..

404) pseud-gout its


a. CACO3
b. CACL3

The correct answer is a

Gout : deposition of ( MSUM ) monosodium urate monohydrate –ve


of birefringent , needle shape,

Psudogout : deposition of ( cppd ) calcium pyrophosphates


dehydrate crystal , +ve birefringent , rhomboid shape, ( CACO3)

.
405) man with history of alcohol assocation with
a. high MCV
b. folic acid deficiency
c. B12 deficieny
e. hepatitis

The correct answer is b

………………………………………………………………………………………………………
……………………………………………..
406) neanate 9 days on breast feed develop jandice
a. breast feed jaundice
b. pathological jaundice

c. physiological jaundice

the correct answer is a

1-pathological jaundice : conjugated ( direct ) bilirubin, start in 1 st


24 hours, bilirubine > 15 mg / dl , persistent 1 week in term infant and
2 weeks in preterm infant

2-physilogical jaundice: unconjugated ( indirect ) bilirubine , start in


2- 3 days, bilirubine < 15 mg/dl , persistent 1 week in term infant and
2 weeks in preterm infant.

3-breast feeding jaundice : unconjucated ( indirect ) bilirubine ,


start in 7 days of life , persistent to one or more months.

………………………………………………………………………………………………………
……………………………………………..
407) lady c/o headche bandlike pain
a. tension headache

………………………………………………………………………………………………………
……………………………………………..
408) regarding breast screening
a. self breast examination early detection of tumor
b. mammogram not advise before 35 y

??????

………………………………………………………………………………………………………
……………………………………………..
409) 19 yrs old c/o abdo pain within menestration for last 6
years diagnosis
a. primary dysmenorrhea
b. secondary dysmenorrheal

The correct answer is a


Primary dysmenorrhea :
Onset within 6 months after menarche . Lower abdominal/pelvic
pain begins with onset of menses and lasts 8-72 hours

410) bilateral breast mass diagnosis


a. ductal carcinoma
b. pagets disease

it could be Intraductal papilloma ??????

………………………………………………………………………………………………………
……………………………………………..
411) beriberi cause of deficiency
a. VIT B1
b. VIT B2
c. VIT B3

The correct answer is a


………………………………………………………………………………………………………
……………………………………………..
412) chronic uses of estrogen association ?????

………………………………………………………………………………………………………
……………………………………………..
413) Asystole
a. adrenalin
b. atropine

The correct answer is


asystol has only 3 durgs epinephrine-vasopressine-atropin

………………………………………………………………………………………………………
……………………………………………..
414) pt has diarrhea , dermatitis and dementia diagnosis
a. pellagra

………………………………………………………………………………………………………
……………………………………………..
415) regarding injectable progestron
a . an cause skin prob ?!!

Injected progesterone ( medroxy-progestrone ) : advantages : light


or no periods, safe with breastfeeding, IM injection every 3 months.
Disadvantages: irregular bleeding, wt. gain, decrease bone mineral
density ( reversible ) , delay fertility discontinue .

………………………………………………………………………………………………………
……………………………………………..
416) sencodary prevention is
a. coronary bypass gaft

Secondary prevention generally consists of the identification and


interdiction of diseases that are present in the body, but that have
not progressed to the point of causing signs, symptoms, and dys-
function

417) 22 yrs old c/o insomina/ sleep disturb treatment


a. SSRI

Short history ,

But the initial management of insomnia : good sleep hygiene .

………………………………………………………………………………………………………
……………………………………………..
418) celluitis in children most commin causes
a. group A steptoccus

In children, the most common cause of cellulitis is S aureus or


group A streptococcus.

………………………………………………………………………………………………………
……………………………………………..
419) antideprssants associated with hypertensive crisis
treatment
a. SSRI

b. MOAIs

c. TCAs

the correct answer is b

esp hydrazines ( phenelzine )


………………………………………………………………………………………………………
……………………………………………..

420) pt his MBI = 24 kg he is


a. noramal weight
b. over weight
c. morbid weight
d. mild weight

The correct answer is a

………………………………………………………………………………………………………
……………………………………………..
421) baby c/o fever , chills , rigors and head rigidity +ve
kurnings sign rash on his lower limb diagnosis :
- meningoccal meninigits

???????

………………………………………………………………………………………………………
……………………………………………..
422) 48 yrs pt with abdo pain , neusea, vominting tenderness in
right hypochondrial
- acute cholecysitis

Fever ?!

423) 29 pt c/o dysurea his microscopic showed G -ve organism is


a. legonealla

b. E. coli

the correct answer is b

………………………………………………………………………………………………………
……………………………………………..
424) 30 yrs pt c/o feeling heaviness in the lower abdomen
having pulge papable at the top scrotum that was
reducible and icreasing in valsalva maneuver diagnosis
a. hydrocele
b. variocele
c. indirect inguinal hernia
d. direct inginal hernia

The correct answer is c

………………………………………………………………………………………………………
……………………………………………..
425) anticoagulation prescrib for
a. one month
b. 6 months
c. 6 weeks
d. one year

The correct amswer is b

………………………………………………………………………………………………………
……………………………………………..
426) cushing syndrome best single test to confirm
a. palsma cortison
b. ATCH
c. Dexamethasone Suppression Test

The correct answer is c

………………………………………………………………………………………………………
……………………………………………..

427) 23 yrs old history of URTI then he developed ecchomysis


best treated
a. local AB
b. local antiviral
c. steroid

The correct answer is c

428) chronic psycotic disorder manged by


a. haloperidol
………………………………………………………………………………………………………
……………………………………………..
429) 29 yrs old lady B-HSG 160 c/o vomiting , abdomenal pain
which is more accurate to diagnosis
a. BHCG serial
b. pelvic US
c. laprascopy

Q not complete, but may be this case is molar and chooce is a .

………………………………………………………………………………………………………
……………………………………………..
430) 70 yrs old man c/o fever , vesicular rash over forehead
mangement
a. IV AB
b. IV antiviral

c. Acyclovir

the correct answer is c (i"m npt sure )

………………………………………………………………………………………………………
……………………………………………..
431) celiac disease involves :
a. proximal part of small intestin
b. distal part of small intestin
c. proximal part of large intesin
d. distal part of large intensin

The correct answer is a

………………………………………………………………………………………………………
……………………………………………..
432) 6 yes old pt cyanosis past history of similar attack 6 month
ago u will do for him
a. CxR

b. PFT
c. secure airway
d. CBC
Q is not clear or not complete , if pt. came with life threatening
cyanosis don't waste time with investigation and start with ABC
( scure airway ) , but if pt. came with past history cyanosis choose
b.

433) side affect of diazepam

Sedation , depence, respiratory supression , anterograde amnesia ,


confusion (especially pronounced in higher doses) and sedation

………………………………………………………………………………………………………
……………………………………………..

434) endemic means:

Endemic :is the constant presence of a disease or infectious agent in a certain


geographic area or population group. ( usually rate of disease )

Epidemic : is the rapid spread of a disease in a specific area or among a


certain population group. ( excessive rate of disease )

Pandemic : is a worldwide epidemic; an epidemic occurring over a wide


geographic area and affecting a large number of people.

………………………………………………………………………………………………………
……………………………………………..

435) Epidemic curve :

a graph in which the number of new cases of a disease is plotted


against an interval of time to describe a specific epidemic or
outbreak .
436) Pt with hodgkin's lymphoma , and red strunberg cell in
pathology and there is esinophil lymphocyte in blood so
pathological classification is:

a. Mixed-cellularity subtype

b. nodular sclerosis subtype of Hodgkin's lymphoma

the correct answer is b

Classical Hodgkin's lymphoma can be subclassified into 4


pathologic subtypes based upon Reed-Sternberg cell morphology
and the composition of the reactive cell infiltrate seen in the
lymph node biopsy specimen (the cell composition around the
Reed-Sternberg cell(s))

Name descreption
Nodular sclerosing Is the most common subtype and is composed of large tumor nodules
CHL showing scattered lacunar classical RS cells set in a background of
reactive lymphocytes, eosinophils and plasma cells with varying
degrees of collagen fibrosis/sclerosis.

Mixed-cellularity Is a common subtype and is composed of numerous classic RS cells


subtype admixed with numerous inflammatory cells including lymphocytes,
histiocytes, eosinophils, and plasma cells. without sclerosis. This type is
most often associated with EBV infection and may be confused with the
early, so-called 'cellular' phase of nodular sclerosing CHL

Lymphocyte-rich or Is a rare subtype, show many features which may cause diagnostic
Lymphocytic confusion with nodular lymphocyte predominant B-cell Non-Hodgkin's
predominance Lymphoma (B-NHL). This form also has the most favorable prognosis

Lymphocyte depleted Is a rare subtype, composed of large numbers of often pleomorphic RS


cells with only few reactive lymphocytes which may easily be confused
with diffuse large cell lymphoma. Many cases previously classified
within this category would now be reclassified under anaplastic large
cell lymphoma

437) 62 y male with DVT and IVC obstruction due to thrombosis


so most like dd is
a. neohrotic syndrome
b. SLE
C. Chirstm disease
D. ?!

………………………………………………………………………………………………………
……………………………………………..

438) Pt with bdominal pain heamatutea , HTN, and have


abnormalty in chromosm 16 , diagnosis is
a. POLY CYCTIC KIDNEY

………………………………………………………………………………………………………
……………………………………………..

439) 17 year pt with dyspnea Po2 , PCO2 ,Xray normal PH


increase so dd is:
a. acute attack of asthma
b. P E
c. pneumonia
d. pnemothrax

the correct answer is a

………………………………………………………………………………………………………
……………………………………………..

440) A long scenario about patient with polydipsia ad polyuria.


Serum osmolrity high . desmoprsin inductin no change
urine omolarity and plasma osmolrity so dd is
a. nphrognic type
b. central tupe

The correct answer is a

………………………………………………………………………………………………………
……………………………………………..

441) 50 year old Man presented to ER with sudden headach,


blurred of vision and eye pain. The diagnosis is:
a. Acute glaucoma
b. Acute conjunctivitis
c. Corneal ulcer
d. -----

The correct answer is a

442) RTA with hip dislocation and shock so causes of shock is


a. blood lose
b. urtheral injery
c. nurogenic

The correct answer is a

………………………………………………………………………………………………………
……………………………………………..

443) most common causes of hand infection


a. truma
b. imunocrombromise
c. ………

the correct answer is a


………………………………………………………………………………………………………
……………………………………………..

444) ttt of cholestatoma is


a. antibiotic
b. steroid
c. surgery
d. Grommet tube

the correct answer is c

-Cholesteatoma is a destructive and expanding growth consisting


of keratinizing squamous epithelium in the middle ear and/or
mastoid . treatment : Surgery is performed to remove the sac of
squamous debris and a mastoidectomy is performed.

445) twins one male and other female . his father notice that
femle become puberty before male so what you say to
father

a. female enter puberty 1-2 year before male


b. female enter puberty 2-3 year before male
c. female enter puberty at the same age male

the correct answer is b

………………………………………………………………………………………………………
……………………………………………..

446) in devolping countery to prevent dental carise , it add to


water
a. florid
b. zink
c. copper
d. iodide

the correct answer is a

………………………………………………………………………………………………………
……………………………………………..

447) 12 y.o boy c/o abdominal pain after playing football,he


denied any h/o trauma ,the pain is in the Lt paraumbilical
region what inx you want to do:
a. CXR
b. ultrasound kidney

The correct answer is b , I'm not sure because MCQs are not
compete.

………………………………………………………………………………………………………
……………………………………………..

448) pt child with back pain that wake pt from sleep


So diagnosis
a. lumber kyphosis
b. osteoarthritis
c. RA
d. Scoliosis

the correct answer is d

449) child with papule vesical on oropharnx and rash in palm


and hand so dd:

a. CMV
b. EBV
c. MEASLS
d. ROBELLA

???? Coxsackie A virus. Vesical in buccal mucusa is


measls.
………………………………………………………………………………………………………
……………………………………………..

450) chid with dental caries and history of bottle feading


So dd
a. nurse milk caris

………………………………………………………………………………………………………
……………………………………………..

451) Generalize antxity disorder best ttt: -19


a. Ssri
b. tricyclic a d
d. MAOI

The correct answer is a

………………………………………………………………………………………………………
……………………………………………..

452) Major depression management:


a. Intial MONOTHERABY even sever sever deprisson
b. Ttt should be change if no response during 2wk (AT
LEAST 6 . WEEKS )

????

………………………………………………………………………………………………………
……………………………………………..

453) psychotherapy, medication, and electroconvulsive therap


Psychiatric pt with un compliance of drugs ttt:

a. depro halopredol injection


b. oral colonazepam

the correct answer is a

454) mild diarrhea mangement :


a. oral antibiotis
b. IVF
c. ORS

the correct answer is c


………………………………………………………………………………………………………
……………………………………………..

455) salpingitis and pid on penicillin but not improve the most
likely organism is :
a. chlamydia
b. nessiria
c. SYPHLIS
d. HSV

the correct answer b

………………………………………………………………………………………………………
……………………………………………..
456) Used for treatment of pseudomembranous colitis:

PO metronidazole is the empiric treatment of choice (500 mg TID


for 10-14 days). For severe infections, oral Vancomycin is the
treatment of choice (125 mg QID 10-14 days).

………………………………………………………………………………………………………
……………………………………………..
457) Lady with of right hypochondrial pain, fever and slight
jaundice. What is your diagnosis?

a. Acute cholecystitis.

Q not complete.

Most common presenting symptom of acute cholecystitis is


upper abdominal pain often radiating to right scapula.
Nausea and vomiting generally present. Patients may report
fever.

458) Deep jaundice wit palpable gallbladder ?

.a. Cancer head of pancreas .i


Q not complete

Pancreatic cancer presents with abdominal pain radiating toward


the back, as well as with
jaundice, loss of appetite, nausea, vomiting, weight loss,
weakness, fatigue, and indigestion.
Examination: may reveal a palpable, nontender gallbladder
(Courvoisier’s sign) or migratory thrombophlebitis
(Trousseau’s sign). Diagnose by CT. treatment usually
palliative (due to metastasis at Dx).
………………………………………………………………………………………………………
……………………………………………..

459) Patient with perforated gallbladder underwent


cholecystectomy. Return to you with fever. On abdominal
X-ray, there is elevation of right hemidaiaphragm. What is
the possible diagnosis?

a. Subphrenic abcess

……………………………………………………………………………………………………
………………………………………………..

Patient with acute perianal pain since 2 days (460 (460


with black mass 2*3 pain 4increase with defecation
:Rx
a. Evacuation under local anesthesia

Q not complete

………………………………………………………………………………………………………
……………………………………………..

461) Rhumatoid arthritis distal interphalengial nodules


a. Hebrerden's nodes

Heberden's nodes are at the DIP while Bouchard's


nodes are at PIP.
462) Elderly patient with RLQ fullness, weight loss, changed
bowel habit, anemic and pale. What is the investigation of
choice?

a. Colonoscopy.

This is now the investigation of choice if colorectal cancer is


suspected provided the patient is fit enough to undergo the bowel
preparation. It has the advantage of not only picking up a primary
cancer but also having the ability to detect synchronous polyps or
even multiple carcinomas, which occur in 5% of cases.
……………………………………………………………………………………………………
………………………………………………..
463) Investigation of choice in Iron deficiency anemia?
a. serum iron
b. ferritin
c. TIBC

the correct answer is b

Serum ferritin is the most sensitive lab test for iron deficiency
anemia.
……………………………………………………………………………………………………
………………………………………………..

464) In patients with hypertension and diabetes, which


antihypertensive agent you want to add first?
a. β-blockers
b. ACE inhibitor
c. α-blocker
d. Calcium channel blocker

the correct answer is b

Diuretics (inexpensive and particularly effective in African-


Americans) and β-blockers (beneficial for patients with CAD) have
been shown to reduce mortality in uncomplicated hypertension.
They are first-line agents unless a comorbid condition requires
another medication. (see table)

Population Treatment
Diabetes with ACEIs.
proteinuria
CHF β-blockers, ACEIs, diuretics (including
spironolactone).
Isolated systolic Diuretics preferred; long-acting
hypertension dihydropyridine calcium channel
blockers.
MI β-blockers without intrinsic
sympathomimetic activity, ACEIs.
Osteoporosis Thiazide diuretics.
BPH α-antagonists.

465) Patient presented with retinal artery occlusion, which of


the following is wrong:
a. painful loss of vision
b. painless loss of vision

the correct answer is a

The most common presenting complaint of retinal artery


occlusion is an acute persistent painless loss of vision.

……………………………………………………………………………………………………
………………………………………………..
466) ECG finding of acute pericarditis?
a. ST segment elevation in all leads

Classic ECG findings in pericarditis: Low-voltage, diffuse ST-


segment elevation.
……………………………………………………………………………………………………
………………………………………………..
467) Epidemiological study want to see the effect of smoking in
the bronchogenic carcinoma, they saw that is 90% of
smokers has bronchogenic carcinoma.
30% of non–smokers has the disease, the specificity of the
disease as a risk factor is:
a. 70 %
b. 30 %
c. 90 %

MCQs not complete

Sensitivity: The probability that a diseased patient will have a


positive test result.
Specificity: The probability that a nondiseased person will have a
negative test result.

Disease No
Presen Disea
t se
Positive a b
test
Negative c d
test

Sensitivity = a / (a + c) Specificity = d / (b + d)

Note:
- A sensitive test is good for ruling out a disease.
- High sensitivity = good screening test (↓ false negatives).
- High specificity = good for ruling in a disease (good confirmatory
test).

468) Baby present with pain in the ear, by examination there is


piece of a glass deep in his ear canal, the mother mention
a history of a broken glass in the kitchen but she clean
that completely. We treat that by:
a. By applying a stream of solution to syringing the ear.
b. Remove it by forceps.
c. Refer her to otolaryngology

* Foreign bodies in the ear canal:


Techniques appropriate for the removal of ear foreign bodies include
mechanical extraction, irrigation, and suction. Irrigation is
contraindicated for organic matter that may swell and enlarge
within the auditory canal (e.g seeds). Insects, organic matter, and
objects with the potential to become friable and break into smaller
evasive pieces are often better extracted with suction than with
forceps. Live insects in the ear canal should be immobilized before
removal is attempted.

Consultations: Consult an ENT specialist if the object cannot be


removed or if tympanic membrane perforation is suspected.

……………………………………………………………………………………………………
………………………………………………..
469) Treatment of open tibial fracture:
a. cephazolin
b. cephazolin+gentamycin
c. gentamicin
d. cephazolin+gentamicin+metronidazole

treatment of open fracture

1- First-generation cephalosporins (Gram-positive coverage) such


as cephalexin (1-2 g q6-8h) suffice for Gustilo type I open fractures.
2- An aminoglycoside (Gram-negative coverage) such as
gentamycin (120 mg q12h; 240 mg/d) is added for types II and III
injuries.
3- Additionally, metronidazole (500 mg q12h) or penicillin (1.2 g
q6h) can be added for coverage against anaerobes.
4- Tetanus prophylaxis should be instituted.
5- Antibiotics generally are continued for 72 hours following wound
closure.
Open fracture classification
Gustilo classification
I Low energy, wound less than 1 cm
II Wound greater than 1 cm with moderate soft tissue damage
High energy wound greater than 1 cm with extensive soft
tissue damage
III IIIa Adequate soft tissue cover
IIIb Inadequate soft tissue cover
IIIc Associated with arterial injury

470) Which of the following found to reduce the risk of


postherpetic neuralgia:
a) corticosteroids only
b) corticosteroids + valacyclovir
c) valacyclovir only

The correct answer is c

Patients with herpes zoster should receive treatment to control


acute symptoms and prevent complications. Patients over the age
of 50, irrespective of other risk factors, are at much greater risk of
developing postherpetic neuralgia and should be offered
treatment. By inhibiting replication of varicella zoster virus, the
antiviral agents acyclovir, famciclovir, and valaciclovir attenuate
the severity of zoster—specifically, the duration of viral shedding is
decreased, rash healing is hastened, and the severity and duration
of acute pain are reduced. Attenuation of the severity of the acute
infection and the neural damage it causes should reduce the
likelihood of postherpetic neuralgia.

……………………………………………………………………………………………………
………………………………………………..

471) 8 months old infant with on & off recurrent crying


episodes & hx of current jelly stools:
a) intussception
b) intestinal obstruction
c) mickel's diverticulitis
d) strangulated hernia

The correct answer is a


472) a man with oblong swelling on top of scrotum increase in
size with valsalva maneuver most likely Dx:
a) direct inguinal hernia
b) indirect inguinal hernia
c) varicocele
d) femoral hernia

the correct answer is b

* Indirect hernia: Herniation of abdominal contents through the


internal and then external inguinal rings and eventually into the
scrotum (in males).
- The most common hernia in both genders.
- Due to a congenital patent processus vaginalis.
- Indirect inguinal hernia increases in size it becomes apparent
when the patient coughs, and persists until reduced.

* Direct hernia: Herniation of abdominal contents through the


floor of Hesselbach’s
Triangle. (Hesselbach’s triangle is an area bounded by the inguinal
ligament, inferior epigastricartery, and rectus abdominis)
- Hernial sac contents do not traverse the internal inguinal ring;
they herniate directly through the abdominal wall and are
contained within the aponeurosis of the external oblique muscle.
- Most often due to an acquired defect in the transversalis fascia
from mechanical breakdown that ↑ with age.
……………………………………………………………………………………………………
………………………………………………..
473) 3 weeks old male newborn with swelling of scrotum
transparent to light & irreducible:
a) epidydemitis
b) hydrocele

The correct answer is b

………………………………………………………………………………………………………
……………………………………………..
474) a young girl experienced crampy abdominal pain &
proximal muscular weakness but normal reflexes after
receiving septra (trimethoprim sulfamethoxazole) :
a) functional myositis
b) polymyositis
c) guillian barre syndrome
d) neuritis

the correct answer is d

475) Which drug causes SLE like syndrome:


a) hydralazine
b) propranolol
c) amoxicillin

The correct answer is a

- Drug-induced SLE causes: Chlorpromazine, Hydralazine,


Isoniazid, Methyldopa, Penicillamine, Procainamide, Quinidine,
Sulfasalazine.

……………………………………………………………………………………………………
………………………………………………..
476) When to give aspirin and clopidogrel?
a) pt with a hx of previous MI
b) Acute MI
c) hx of previous ischemic stroke
d) hx of peripheral artery disease
e) after cardiac capt

The correct answer is b

……………………………………………………………………………………………………
………………………………………………..
477) 17 years with history of right iliac fossa pain rebound
tenderness +ve guarding what is the investigation that
you will do?

a. Laparoscopy

b. US
c. CT scan

the correct answer is c

In appendicitis, CT scan with contrast has 95–98% sensitivity which


shows periappendiceal streaking.

478) Child with history of URTI before 3 weeks now has both
knees tender, red and inflamed. What is your diagnosis?

a. Rheumatoid arthritis

Q is not complete

The diagnosis of JRA is established by the presence of arthritis,


the duration of the disease for at least 6 weeks, and the exclusion
of other possible diagnoses. Although a presumptive diagnosis of
systemic-onset JRA can be established for a child during the
systemic phase, a definitive diagnosis is not possible until arthritis
develops. Children must be younger than 16 years old at time of
onset of disease; the diagnosis of JRA does not change when the
child becomes an adult. Because there are so many other causes of
arthritis, these disorders need to be excluded before providing a
definitive diagnosis of JRA (Table 89-2). The acute arthritides can
affect the same joints as JRA, but have a shorter time course. In
particular, JRA can be confused with the spondyloarthropathies,
which are associated with spinal involvement, and enthesitis, which
is inflammation of tendinous insertions. All of the pediatric
spondyloarthropathies can present with peripheral arthritis before
other manifestations and initially may be diagnosed as JRA (Table
89-3).

* Juvenile rheumatoid arthritis: mono- and polyarthropathy with


bony destruction that occurs in patients ≤ 16 years of age and
lasts > 6 weeks. Approximately 95% of cases resolve by puberty.
Subtypes: Pauciarticular: ≤ 5 joints involved, associated with
an ↑risk of iridocyclitis that lead to blindness if left untreated.
Polyarticular: ≥5 small joints. Systemic features are less
prominent; carries a ↓risk
of iridocyclitis. Acute febrile: The least common subtype;
manifests as arthritis withdaily high, spiking fevers and an
evanescent, salmon-colored rash. Hepatosplenomegaly and
serositis may also be seen. No iridocyclitis is present; remission
occurs within one year.

……………………………………………………………………………………………………
………………………………………………..
479) Old female came complain from pan in her joint increase
with walking, what is your diagnosis?
a. Osteoarthritis

Physical signs of OA: Crepitus, ↓ range of motion, pain that


worsens with activity
and weight bearing but improves with rest.

480) Patient came with history of neck discomfort, palpitation,


cold sweat, TSH low, T4 high, tender neck. What is your
diagnosis?

a. Subacute thyroiditis.

- Thyroiditis: Inflammation of the thyroid gland. Common types


are subacute granulomatous, radiation, lymphocytic, postpartum,
and drug-induced (e.g., amiodarone) thyroiditis.
- Hx/PE: In subacute and radiation, presents with tender thyroid,
malaise, and URI symptoms.
- Dx: Thyroid dysfunction (typically hyperthyroidism followed by
hypothyroidism),
all with ↓ uptake on RAIU.
- Rx:
- β-blockers for hyperthyroidism; levothyroxine for hypothyroidism.
- Subacute thyroiditis: Anti-inflammatory medication.
……………………………………………………………………………………………………
………………………………………………..
481) In ulcerative colitis, what you will start treatment with?
a. Corticosteroids ??

- Sulfasalazine or 5-ASA (mesalamine).


- Crticosteroids and immunosuppressants indicated if no refractory
disease.
……………………………………………………………………………………………………
………………………………………………..
482) Treatment of prostatitis?
a. Ciprofloxacin
treatment is for 4–6 weeks with drugs that penetrate into the prostate,
such as trimethoprim or ciprofloxacin.

………………………………………………………………………………………………………
……………………………………………..
483) In IV canula and fluid:
a.Site of entry of cannula is a common site of infection.
……………………………………………………………………………………………………
………………………………………………..
484) Patient with neck rigidity, rigor, fever, petechial rash over
extremities. The causative m\o is:
a. Meningococcal meningitis

Discuss meningitis !
……………………………………………………………………………………………………
………………………………………………..
485) Female patient works in office came with vulva itching,
yellow vaginal discharge:
a. Trichomoniasis

486) another one old c/o bilateral knee pain with mild joint
enlargement

ESR and CRP normal dx:

a. Osteoarthritis

b. Rheumatoid arthritis

c. Gout?

The correct answer is a

………………………………………………………………………………………………………
……………………………………………..
487) .Scitica increased incidence of :

a. Lumbar lordosis

b. Parasthesis

Q not complete but with these MCQs , correct answer is b


………………………………………………………………………………………………………
……………………………………………..
488) old male c/o knee pain on walking with crepitus xray show
narrow joint space and subchondoral sclerosis:

a. Rheumatoid arthritis

b. Osteoarthritis

c. Gout

The correct answer is b

………………………………………………………………………………………………………
……………………………………………..

489) 14y f with BMI 32.6 (associated big chart):

a. Overweight

b. Obese

c. Normal weight

The correct answer is b

BMI < 16 : severe under WT.

BMI 16 – 20 : under wt.

BMI 20 – 25 : normal

BMI 25 – 30 : over wt.

BMI 30 – 35 : obese classic 1

BMI 35 – 40 : obese classic 2

BMI > 40 : obese classic 3


490) vaginal discharge odorless watry microscopy show clue
cells :

a. Bacterial vaginosis

b. Candidiasis

The correct answer is a

………………………………………………………………………………………………………
……………………………………………..

491) Pregnant women on 3rd trimester infected with measels can I give
her MMR?????

………………………………………………………………………………………………………
……………………………………………..

492) lactating women infected with rupella ……..management is


:

a. MMR

b. Stop lactation

The correct answer is a

MMR is safe during lactating

………………………………………………………………………………………………………
……………………………………………..

493 ) which of following not live vaccine:

a. BCG

b. Hepatitis B

c. Oral polio
d. MMR

The correct answer is b

………………………………………………………………………………………………………
……………………………………………..

493) the most commone cause of community acquired


pneumonia:

a. Hemophilus influenza

b. Strept.pneumonia

c. Mycoplasma

d. Kliebsella

The correct answer is b

494) female after vaginal hesterectomy she complain of urin


come from vagina………dx:

a. Vesicovaginal fistula

b. Urethrovaginal fistula

c. Ureterovaginal fistula

The correct answer is a

………………………………………………………………………………………………………
……………………………………………..
495) child rt ear pain and tenderness on pulling ear , no fever ,
O/E inflamed odemateous rt ear canal with yellow
discharge >>>>>>>>>>dx:

a. Otitis media

b. Otitis externa

c. Cholesteatoma

The correct answer is b

……………………………………………………………………………………………………
………………………………………………..
496) male with perianal pain ,tenderness ,fluctuant (perianal
abscess)

a. Incision and drainage

b. Warm bath

The correct answer is a

……………………………………………………………………………………………………
………………………………………………..
497) . pregnant 34w with vaginal bleeding………. what would
you ask about :

a. Cigarette smoking

b. Recent sexual intercourse

Q not complete , but with theses MCQs may be correct


answer is a

……………………………………………………………………………………………………
………………………………………………..
498) In duodenal obstruction of neoborn what is the sign that
apper in xray:

a. Double bubble
499) the following more commone with type2 DM than type1
DM:

a. Weight loss

b. Gradual onset

c. Hereditary factors

d. HLA DR3+-DR4

The correct answer is c

………………………………………………………………………………………………………
……………………………………………..
500) pt with open angle glaucoma and k/c of COPD and DM ttt:

a. Timelol

b. betaxolol

c. Acetazolamide

The correct answer is c

………………………………………………………………………………………………………
……………………………………………..

501) pt child with back pain that wake pt from sleep So


diagnosis
a. lumber kyphosis
b. osteoarthritis
c. RA
d. Scoliosis

The correct answer is a or d

………………………………………………………………………………………………………
…………………………………

502) 59 y/o presented with new onset supraventicular


tachycardia with palpitation,no Hx of SO Or chest pain
,chest examination normal , oxygen sat in room air = 98%
no peripheral edemaOthers normal, the best initial
investigation:

a. ECG stress test


b. Pulmonary arteriography
c. CT scan
d. Thyroid stimulating hormone

the correct answer is d

503) Lactating women presented with breast engorgment and


tendress Your managements:

a. Warm compressor and continoue breast feeding

b. Dicloxacillin and continoue breast feeding

c. Dicloxacillin and milk expression

d. Discontinoue breast feeding and cold compressor

the correct answer is a . tis case is ( engorgement of breast ) . but


if there are redness , swelling , fever this case is ( mastitis ) and
choose b

504) The Fastest route of antipsychotic is:

a. IM

b. IV

c. Oral

d. Sublingual

the correct answer is b

505) While you are in the clinic you find that many patients
presents with red follicular conjactivitis (Chlamydia ) your
management is:

a. Improve water supply and sanitation


b. Improve sanitation and destroying of the vector

c. Eradication of the reservoir and destroying the vector

d. Destroy the vector and improve the sanitation

the correct answer is a

506) The most important exogenous risk factor for


osteoporosis is:

a. Alcohol intake

b. Age

c. Smoking

d. Lack of exercise

the correct answer is c

507) Patient with family history of coronary artery disease his


BMI= 28 came to you asking for the advice:

a. Start 800 calorie intake daily

b. Decrease carbohydrate daytime

c. Increase fat and decrease protein

d. Start with decrease ……. K calorie per kg per week

the correct answer is d

508) Lactating mother newly diagnosed with epilepsy , taking


for it phenobarbital you advice is:

a. Discontinue breastfeeding immediately

b. Breastfeed baby after 8 hours of the medication


c . Continoue breastfeeding as tolerated

the correct answer is c

-very vague question , some books avoid Phenobarbital during


breast feeding if possible. And in American academy of pediatric
classified Phenobarbital as adrug that cause major advers effect
in some nursing infant, and should be given to nursing women
with cation .

509) Pregnant women has fibroid with of the following is True:

a. Presented with severe anemia


b. Likely to regress after Pregnancy
c. Surgery immediately
d. Presented with Antepartum He

the correct answer is b

Fibroids may also be the result of hormones. Reproductive


hormones like estrogen and progesterone can stimulate cell
growth, causing fibroids to form. During pregnancy, your influx
of hormones may cause your fibroids to grow in size. After
pregnancy and during menopause most fibroids begin to shrink,
due to a lack of hormones.

510) A known case of chronic atrial fibrillation on the warfarin


5 mg came for follow up you find INR 7 but no signs of
bleeding you advice is:

a. Decrease dose to 2.5 mg


b. Stop the dose and repeat INR next day
c. Stop warfarin
d. Continoue same and repeat INR

the correct answer is b

INR ACTION

>10 Stop warfarin. Contact patient for examination. MONITOR INR


7-10 Stop warfarin for 2 days; decrease weekly dosage by 25% or by 1 mg/d
for next week (7 mg total); monitor INR

4.5- Decrease weekly dosage by 15% or by 1 mg/d for 5 days of next week (5
7 mg total); repeat monitor INR

3- Decrease weekly dosage by 10% or by 1 mg/d for 3 days of next week (3


4.5 mg total); repeat monitor INR.

2-3 No change.

1.5- Increase weekly dosage by 10% or by 1 mg/d for 3 days of next week (3
2 mg total);

<1.5 Increase weekly dose by 15% or by 1 mg/d for 5 days of next week (5 mg
total);

511) Patient is a known case of CAD the best exercise:

a. Isotonic exercise

b. Isometric exercise

c. Anerobic exe

d. Yogha

the correct answer is c

1-anerobic exercise ( endurance ) : for improve cardiac function


2- weight bearing excercise ( isometric ) : for build muscle strength
, bone density
3- stretching excercise : for prevent cramp , stiffness and back pain

512) The mechanism of action of Aspirin:

a. Inhibit cycloxgenase

b. Inhibit phospholipase A2

c. Inhibit phospholipid D

the correct answer is a

513) The absolute contraindication of breastfeeding is :


a. Asymptomatic HIV patient

b. Active hepatitis C

c. Pulmonary TB on treatment 3 months

The correct answer is a

Absolute contraindication of breastfeeding :

1-Infants with galactosemia.

2-Mothers who use illegal drugs.

3-Mothers infected with HIV, human T-cell lymphotropic virus type I


or type II, or who have an active herpes lesion on the breast.

4-Mothers taking any of the following medications: radioactive


isotopes, cancer chemotherapy agents, such as antimetabolites
.

514) A boy felt down on his elbow , the lateral x-ray shows:

a. Anterior Pad sign

b. Posterior pad sign

c. Anterior line of humerous intersecting the cubilium

d. Radial line forming 90 degree with cubilium

the correct answer is b

515) A known case of treated hodgkin lymphoma(mediastinal


mass) with radiotherapy Not on regular follow up presented
with gradual painless difficulty in swallowing and SOB ,
There is facial swelling and redness : DX

a. SVC obstruction

b. IVC obstruction

c. Thoracic aortic aneurysm

d. Abdominal aortic aneursm

The correct answer is a


________________________________________________________________________

516) Patient is presented with hand cellulitis and red streaks


in the hand and tender axillary lymphadenopathy. This
condition is more likely to be associated with:

a. Malignancy

b. Pyoderma

c. Neuropathy

d. Lymphangitis
the correct answer is b (I'm not sure )

516) Young aged male presented to ER after blunt trauma to


Abdomen, CT scan shows intramural hematoma: your
management is

a. Lapratomy with evacuation of the hematoma

b. Dissection of duodenum

c. Observation

the correct answer is c

517) Patient presented with sore throat, anorexia, loss of


appetite , on throat exam showed enlarged tonsils with
petechi on palate and uvula , mild tenderness of spleen and
liver :DX

a. Group A strep
. b. EBV

The correct answer is b

518) Patient with GERD has barret esophagus , this metaplasia


increase risk of :
a. Adenocarcinoma
b. Squmaou cell carcinoma

the correct answer is a

519) Complication of Sleep apnea is :

a.CHF
b. ……

the correct answer is a

sleep apnea : Hypoxic pulmonary vasoconstriction  PAH 


Cor Pulmonale  CHF

complication of sleep apnea : sleep apnea increases health risks such as


cardiovascular disease, high blood pressure, stroke, diabetes, clinical depression, weight
gain and obesity. The most serious consequence of untreated obstructive sleep apnea is to
the heart. In severe and prolonged cases, there are increases in pulmonary pressures that
are transmitted to the right side of the heart. This can result in a severe form of congestive
heart failure (cor pulmonale).

520) Which of the following medication can be used as


prophylaxis in appendectomy:

a. Cephalexin

b. Ceftriaxone

c. Metronidazole

d. Vancomycin

e. Ampicillin

The correct answer is b

1st line of antibiotic : 1- cefoxitin 2- cefotetan

2nd line of treatment : 1- metradinazole 2- ampicikkin-sulbactam

521) Which of the following prognostic factor for SLE:

a. ANA levels
b. Sex

c. Age

d. Renal involvement

The correct answer is d

522) The most common site for osteomyelitis is:

a. Epiphysis

b. Diaphysis

c. Metaphysis

d. Blood flow

The correct answer is c

523) In “holding breath holding” which of the following True:

a. Mostly occurs between age of 5 and 10


b. Increase Risk of epilepsy
c. A known precipitant cause of generalized convulsion
d. Diazepam may decrease the attack

Breath holding spells are the occurrence of episodic apnea in children,


possibly associated with loss of consciousness, and changes in postural tone.
They are most common in children between 6 and 18 months and usually not
present after 5 years of age. They are unusual before 6 months of age. A
positive family history can be elicited in 25% of cases. It may be confused
with a seizure disorder.

There are four types of breath holding spells.

1-The most common is termed simple breath holding spell, in which the
manifestation is the holding of breath in end expiration. There is no major
alteration of circulation or oxygenation and the recovery is spontaneous.
2-The second type are the Cyanotic breath-holding spells. They are
usually precipitated by anger or frustration although they may occur after a
painful experience. The child cries and has forced expiration sometimes
leading to cyanosis (blue in color), loss of muscle tone, and loss of
consciousness. The majority of children will regain consciousness. The child
usually recovers within a minute or two, but some fall asleep for an hour or
so. Physiologically, there is often hypocapnea (low levels of carbon dioxide)
and usually hypoxia (low levels of oxygen. There is no "post ictal" phase (as is
seen with seizures), no incontinence, and the child is fine in between spells.
EEGs are normal in these children. There is no relationship to the subsequent
development of seizures or cerebral injury as a consequence of breath
holding spells.

3-In the third type, known as Pallid breath-holding spells, the most
common stimulus is a painful event. The child turns pale (as opposed to blue)
and loses consciousness with little if any crying. The EEG is also normal, and
again there is no post ictal phase, nor incontinence. The child is usually alert
within a minute or so. There may be some relationship with adulthood
syncope in children with this type of spell.

4- A fourth type, known as Complicated breath-holding spells, may


simply be a more severe form of the two most common types. This type
generally begins as either a cyanotic or pallid spell that then is associated
with seizure like activity. An EEG taken while the child is not having a spell is
still generally normal.

DX: clinical , good history include sequence of event , lack of incontinence


and no post ictal phase. Treatment : reassurance and iron.

524) Infant brought by the mother that noticed that the baby
has decreasing feeding , activity and lethargic On
examination febrile(39), tachycardic ,his bp 75/30, with
skin rash . DX:

a.Septic shock

Q not complete

525) Old patient newly diagnosed with hyperthyroidism


presented with (hyperthrodism symptoms) The best initial
symptomatic treatment is:

a. BB ( beta-blocker )

b. PTU
The correct answer is a

_______________________________________________________________________

526) Infant presented with hemangioma on the back . your


management is:

a. Intralesional injection of corticosteroids

b. Topical corticosteroids

c. Excise of the lesion

527) Pregnant lady , 34 wk GA , presented with vaginal


bleeding more than her menstruation. On examination ,
cervix is dilated 3 cm with bulging of the membrane, fetal
heart rate = 170 bpm . The fetus lies transverse with back
facing down . us done and shows that placenta is attached
to posterior fundus and sonotranulence behind placenta.
Your management is :

a. C/S
b. Oxytocin
c.Tocolytics
d.Amniotomy
the correct answer is a

528) Infant with congenital hip dislocation:

a. +ve click in flexion ,abduction

b. The only treatment is surgery

c. Not reduced with flexion and abduction of the hip

the correct answer is a . this is ortolani test

529) In irritable bowel S. the following mechanism ?


contraction and slow wave myoelectricity seen in:
a.Constipation

b. Diarrhea

the correct answer is a

529) Which is not found in coarctation of the aorta:

a. Upper limb hypertension

b. Diastolyic murmur heard all over precordium

c. Skeletal deformity on chest x-ray

the correct answer is a (I'm not sure )

530) Female patient presented with tender red swelling in the


axilla with history of repeated black head and large pore
skin in same area: ttt is

a. Immidate surgery

b. Topical antibiotic

c. Cold compressor

d. Oral antibiotic

531) In indirect hernia the relation of the sac to the cord


structure is:

a. Anteromedial

b. Anterolateral

c. Posteromedial

d.Postrolateral

the correct answer is b

532) The most common cause of croup is:


a. Parainfluenza

b. Influenza

the correct answer is a

533) kwashikor disease usually associated with :

a. decrease protein intake, decrease carbohydrate

b. increase protein , increase carbo

c. decrease protein , increase carbo

the correct answer is c. in the book, written there is


decrease protein and adequate amount of carbohydrate,
but with this only MCQs the answer is c

534) in cachectic patient, the body utilize the proteins of the


muscles :

a. to provide Amino acid and protein synthesis

b. to maintain bloodflow to vital organ

c. to increase body fat

the correct anwser is a ( I'm not sure )

535) patient is complaining of memory loss. Alzehimer disease


is diagnosed what is the cause of this:

a. brain death cell

Q not complete

536) Parents brought their baby to you who is on bottle


feeding. On exam whitish lesion on either side of teeth
seen with blackish lesion on maxillary incisors and second
molar teeth. There is history of leaving the baby with bottle
in his mouth during sleeping. The Dx:

a. Nursery dental caries

b.Gingvostomatis

the correct answer is a


537) Which of the following medication if taken need to take
the patient immidiatly to the hospital:

a. Penicillin

b. diphenhydramine

c. OCPs

d. Quinine or Quinidine

the correct answer is d (I'm not sure ) , quinidine is


antiarrhythmia .

538) 43 y/o female presented with severe DUB other


examination normal . your management is

a. D &C

b. Ocps

c. Hysterectomy

d. Blood transfusion

the correct answer is a

539) Baby with vesicles on the face and honey comb crust
which of the following organism cause it: Staph aureus

540) Female patient presented with migraine headache which


is pulsatile, unilateral , increase with activity . Dosn't want
to take medication. Which of the following is appropriate:

a. Bio feedback

b. TCA

c. BB

the correct answer is a


Biofeedback has been shown to help some people with
migraines. Biofeedback is a technique that can give people better
control over body function indicators such as blood pressure, heart
rate, temperature, muscle tension, and brain waves. The two most
common types of biofeedback for migraines are thermal
biofeedback and electromyographic biofeedback.

541) Infant born with hemangioma on the rt eyelid what is


appropriate time to operate to prevent amylopia:

a. 1 day

b. 1 week

c. 3 months

d. 9 months

difficult Q. but may be the correct answer is b or c

542 ) Young patient on anti TB medication presented with


vertigo which of the following drug cause this:

a. Streptomycin

b. Ethambutol

c. Rifampcin
the correct answer is a , streptomycin cause 8th nerve damage.

543) The CPR for child is

a. 30 chest compression 2 ventilation … (Lone rescuer)

b. 15 chest c 2 ventilation … (2 or more rescures)

c. m15 chest compression 1 ventilation

the correct answer is a


544) Picture show large ulcer over medial side of the leg . what
is your management

a. Shave biobsy

b. Elevate the legs and stocking

c. Topical steroids

the correct answer is b

………………………………………………………………………………………………………
……………………………………………..
545) 2months infant with white plaque on tongue and
greasy ,past h/o clamydia conjunctivitis after birth treated
by clinamycin what is ttt:

a. Oral nystatin

b. Topical steroids

c. Topical acyclovair

d. Oral tetracycline

the correct answer is a. oral nystatin : antifungal

………………………………………………………………………………………………………
……………………………………………..
546) child rt ear pain and tenderness on pulling ear , no
fever , O/E inflamed odemateous rt ear canal with yellow
discharge >>>>>>>>>>dx:

a. Otitis media

b. Otitis externa

c. Cholesteatoma

the correct answer is a

………………………………………………………………………………………………………
……………………………………………..
547) 34y female with HIV pap smear negative, about cervical
cancer screening :

a. After 3m if negative repeat after 6m


b. After 6m …………………………… annually

c. After 1y………………………………….annually

The correct answer is b ( I'm not sure , confuse with choice c )

548) female about 30y with breast cancer (given cbc –chem.
And reavel low hb and hematocrite….) what is the next step
in mangment:

a. Staging

b. Lumpectomy

c. Mastectomy

d. chemotherapy

the correct answer is a

………………………………………………………………………………………………………
……………………………………………..
549) child with moderate persistant BA On bronch.dilat
inhaler. Presented with acute exacerbation what will you
add in ttt:

a. Corticosteroid inhaler

b. Ipratropum bromide inhaler

the correct answer is b

………………………………………………………………………………………………………
……………………………………………..

550) about head and neck injury :

a. Hoarsness of voice and stridor can occure with mid


facial injury

b. Tracheostomies contraindicated

c. Facial injury may cause upper air way injures


………………………………………………………………………………………………………
……………………………………………..
551) lactating women 10 days after delivary developed
fever ,malaise, chills tender Lt breast with hotness and
small nodule in upper outer quadrant with axillary LN
.Leucocytic count was 14 *10/L dx:

a. Inflammatory breast cancer

b. Breast abscess

c. Fibrocystic disease

the correct answer is b

552) 70y male with osteoporosis the T score of bone


densometry would be :

a. -3.5

b. -2.5

c. 1

d. 2

e. 3.5
the correct answer is b

………………………………………………………………………………………………………
……………………………………………..

553) child with epistaxis…….. management:

a. Compression on nose and leaning forward

b. ………………………………………………….backward

the correct answer is a

………………………………………………………………………………………………………
……………………………………………..
554) scenario about female underwent abdominal operation
she went to physician

For check……….. U/S reveal metal thing inside abdomen (a.e


missed during operation)

What will you do :

a. Call the surgeon and ask him what to do

b. Call attorney and ask about legal action

c. Tell her what you found

d. Tell her that is one of possible complications of


operation

e. Don't tell her what you found

The correct answer is c

………………………………………………………………………………………………………
……………………………………………..
555) male ptn with scaly fine papular rash on fornt of
scalp,nose and retroauricular……..(i think tinea capitis) ttt
is:

a. Ketoconazole cream…

b. Oral augmentin
c. ……… cream

the correct answer is a

556) All can cause gastric ulcer except:


a- Tricyclic antidepressant.
b- Delay gastric emptying.
c- Sepsis.
d- Salicylates.
e- Gastric outlet incompetent.

The correct answer is a


Tricyclic antidepressant use in treatment of peptic ulcer.
………………………………………………………………………………………………………
……………………………………………..

557) 48year old female lost her menstruation for 2 cycles, the
method of contraception is condom, examination was
normal except for dusky discoloration of the cervix. What u
will do next:
a.Progesterone challenge.
b.Beta HCG.
c.Pelvic u/s

The correct answer is b

………………………………………………………………………………………………………
……………………………………………..

558) ttt of yeast vaginitis: Clotrimazole , Fluconazole ,


Ketokonazole

Treatment of yeast vaginitis ( candida albicans ) : topical


azole or PO fluconazole , oral azole should be avoided in
pregnant.

………………………………………………………………………………………………………
……………………………………………..

559) 15 y/o female complaining of pain during menstruation,


not sexually active, medical hx unremarkable, physical
examination normal, how to treat:
a- NSAID.
b- Danazole.
c-

The correct answer is a

………………………………………………………………………………………………………
……………………………………………..

560) What is true about alpha blocker:


a. Causes hypertension.
b. Worsen benign prostatic hyperplasia.
c. Cause tachycardia.
d-

the correct answer is c


alpha blocker :cause orthostatic hypotension and tachycardia.

561) cyclic menstruation that increase in frequency:


a. Polymenorrhea.
b. Hypermenorrhea.
c. Menorrhagia.
d. Dysmenorrhea.
e.

The correct answer is a


Polymenorrhea : frequent menestration ( < 21 day cycle )
Hypermenorrhea or menorrhagia : abnormally heavy and
prolonged menstrual period at regular intervals . more than 80 ml
of blood loss per cycle or prolonged bleeding , more than 8 days.
Oligomenorrhea : increase length of time between menses ( 35-
90 days b\w cycle)
Metrorrhagia : bleeding bwteen period.
Menometrorrhagia : exessive and irreglar bleeding.

………………………………………………………………………………………………………
……………………………………………..

562) A pregnant lady came to you to screen her fetus for down
syndrome, what is the best method:
a. Amniocentesis. + Karyotyping
b. Choriocentesis.
c.

the correct answer is a

………………………………………………………………………………………………………
……………………………………………..

563) What is the most common chromosomal abnormality?


a. Trisomy 13
b. Trisomy 21

the correct answer is b


-down syndrome ( trisomy 21 ) is the most common
chromosomal abnormality.

………………………………………………………………………………………………………
……………………………………………..

564) Best 2 test to screen for hepatocellular carcinoma:


a. Liver biopsy and alpha-fetoprotein.
b. Liver ultrasound and alpha-fetoprotein.
c. Abdomen CT and
d.

The correct answer is b

565) A child came to ER with fever, stridor, … , x-ray showed


swollen epiglottis, in addition to oxygen, what u will do?
a. Throat examination.
b. An emergency tracheostomy.
c. Endotracheal intubation.
d. Nasopharyngeal intubation.

The correct answer is c


………………………………………………………………………………………………………
……………………………………………..

566) 24 y/o female newly diagnosed type 2 DM, she is wearing


glasses for 10 years, how frequent she should follow with
ophthalmologist:
a. Every 5 years.
b. Annually

The correct answer is b


-for type 1 diabetic : retina screening annually beginning 5
years after onset of diabetes, general not before onset of puberty.
- for type 2 diabetic : screening at the time of diagnosis then
annual
………………………………………………………………………………………………………
……………………………………………..

567) What is the initial management for a patient newly


diagnosed knee osteoarthritis.
a. Intra-articular corticosteroid.
b. Reduce weight.
c. Exercise.
d. Strengthening of quadriceps muscle.
The correct answer is b
………………………………………………………………………………………………………
……………………………………………..

568) A lady came to your clinic said that she doesn’t want to
do mammogram and preferred to do breast self-
examination, what is your response?
a- Mammogram will detect deep tumor.
b- Self-examination and mammogram are complementary.
c- Self-examination is best to detect early tumor

answer is b ( I'm not sure because MCQs not complete and depend
on age of pt.

………………………………………………………………………………………………………
……………………………………………..

569) What is the best frequency for breast self-examination?


a. Daily.
b. Weakly.
c. Monthly.
d. Annually.

The correct answer is c


570) Patient with left bundle branch block will go for dental
procedure , regarding endocarditis prophylaxis:
a. No need
b. Before procedure.
c. After the procedure.
d.

The correct answer is a


………………………………………………………………………………………………………
……………………………………………..

571) classical characteristic for genital herpes.


Painful ulcers & vesicles

………………………………………………………………………………………………………
……………………………………………..

572) most common vaginal bleeding :


a. cervical polyps
b. menstruation
c. ????

only with these MCQs , the correct answer is b


………………………………………………………………………………………………………
……………………………………………..

573) best stimulant for breast milk secretion:

a. breast feeding
b. oxytocin
c. ???

the correct answer is a


………………………………………………………………………………………………………
……………………………………………..

574) child with aspirin intake overdose ...what kind of acid


base balance:
a. metabolic alkalosis wt respiratory
b. metabolic acidosis wt respiratory alkalosis
c. respiratory alkalosis with metabolic acidosid
d. respiratory acidosis with metabolic alkalosis

The correct answer is c


Aspirin toxicity : in early stages, salicylate will stimulate respiratory center →
↑RR → respiratory alkalosis that will be compensated by metabolic acidosis. In
late stage, it will interfere with COH, fat, & protein metabolism as well as
Oxidation phosphorylation lead to ↑ lactate, pyrovate, & keton bodies. All will lead
to ↓pH. Signe & symptoms includes: nausea, vomiting, ↑RR, ↑temp, ↑HR,
sweating, cerebral or pulmonary edema, & coma. +ve anion gap. TTT: hydration,
correct K+, gastric lavage or activated charcoal, urine alkalization, hemodialysis)

575) therapeutic range of INR :


a. 2.5-3.5
b. 2.0-3.0
c. ???

The correct answer is b

………………………………………………………………………………………………………
……………………………………………..

576) q about antidepressant:


a. start single type even patient have sever depression
b. start any one of them they all have the same efficacy
c. stop the medication after 2 weeks if no improvement

The correct answer is a

………………………………………………………………………………………………………
……………………………………………..
577) patient with depression started on amitryptaline , he had
headache or dizzness , vomitting?? im not sure what
exactly was the symptoms
a. change to SSRI
b. ??

Q not complete , but amitriptyline is TCA .

………………………………………………………………………………………………………
……………………………………………..

578) patient had horsenss of voice for 3 weeks... next to do:


a. throat swab
b. laryngoscopy
c. ???

The correct answer is b , I'm not sure because MCQs


are not complete.

………………………………………………………………………………………………………
……………………………………………..

579) single diagnostic for stroke:


a. high cholesterol
b. high systolic blood pressur

the correct answer is b , I'm not sure , but with these mcqs , b is
correct.

580) unfaivrable prognosis for schezophrenia:

a. family Hx

b. failed marrige

c. adolscen age?? not sure

d. presence of psychosis

the correct answer is a


………………………………………………………………………………………………………
……………………………………………..

581) Rt lung :
a. have 1 fissure
b. contain 7 segment
c. ??? read about lung anatomy segment names!!

Rt lung has 3 lobes & 10 segments (medial basal segment only in inf.
Lobe of Rt lung)
Lft lung has 2 lobes & 9 segments (Lingular segments in lft lung)

………………………………………………………………………………………………………
……………………………………………..

582) q about specifity:


a. test is +ve in disese population
b. test is +ve in healthy poplation
c. test is -ve in diseased population
d. test is -ve in healthy population

The correct answer is d


………………………………………………………………………………………………………
……………………………………………..

583) stage 3 colon ca  Give Chemo ASAP

………………………………………………………………………………………………………
……………………………………………..

584) patient with upper abdominal pain, nausea vomitting,with


back pain, he is smoker for long time daily, fecal fat was
+ve
a. acute pancreatitis
b. chronic pancreatitis
c. pancreatic CA

The correct answer is b

585) in school, 10 of patient had the dis of the first week ,


then 30 pt. had the dieses the next week , the percentage
infection of this school is:
a. 10%
b. 20%
c. 30%
d. 40%

Q is not clear or not complete, but I think the correct


answer is b.
………………………………………………………………………………………………………
……………………………………………..
586) patient had headche describe it as aband around his
head, increase with stress and , i dont remeber the ques
DX is:
a. tension headache
b. migraine
c. cluster headache

The correct answer is b

………………………………………………………………………………………………………
……………………………………………..
587) Patient was presented by bollus in his foot , biopsy
showed sub dermal lysis , fluorescent stain showed IgG ,
what is the most likely diagnosis :

A. Bolus epidermolysis .

B. Pemphigoid vulgaris .

C. Herpetic multiform .

D. Bullous pemphigoid .

The correct answer is d

………………………………………………………………………………………………………
……………………………………………..

588) Patient was presented by difficulties of breathing from


one side of his nose , on examination there was
erythramatus swelling , what is the best initial treatment :

A. Decongestant .

B. Steroid .

C. Sympathomimetics . ??

The correct answer is c ( I'm not sure )

Explain: this swelling might be nasopharyngeal angiofibroma which is


highly vascular swelling & prone to epistaxis, so to decrease the
size we might use local or systemic decongestants which are
sympathomimetics, so answer is not clear.

589) Patient with cystic nodule (acne) and scars , what is the
best treatment :

A. Retinoin .

B. Erythromycin .

C. Doxycyclin .

The correct answer is a ( I'm not sure )

this seems to be an advanced inflammatory stage of acne since


there r scars so if the retinoiD meant here is systemic (isotretinin)
so it will be the choice. If topical it will not be effective as it is
effective only in the non-inflammatory stage (comedones). Both
erythromycin (local) & doxycyclin (systemic) can be used but in
advanced stage systemic is better.

………………………………………………………………………………………………………
……………………………………………..

590) Patient was presented by constipation, vomiting ,


abdominal distension , with old scar in the lower abdomen ,
x ray showed dilated loops with air in the rectum , what is
the best initial management :

A. NGT decompression , and IV line .

B. Rectal decompression and antibiotics .

C. Suppositories .

The correct answer is a

this is a case of intestinal obstruction due to adhesions from


previous abdominal surgery (commonest cause) so the initial
treatment would be decompression of the intestine & replace lost
fluids

………………………………………………………………………………………………………
……………………………………………..

591) Which of the following is not living vaccine :

A. Hepatitis B .

B. MMR .

C. Oral polio

D. BCG

The correct answer is a

Hepatitis B is the only vaccine prepared by genetic engineering so its


not living vaccine.

592) Six years old child was born to a mother with hepatitis
B , he does not received any vaccines before , what you will
give now :

A. DTP , MMR , Hib

B. DTP , MMR .

C. DT , MMR , Hib

D. DT, MMR .

The correct answer is d

both Pertussis & H.influenza vaccine are not needed after 1 year
of age.

………………………………………………………………………………………………………
……………………………………………..

593) Which of the following describes the end of the early


inflammatory phase .
A. Formation of eschar .

B. Formation of ground base of collagen .

C. The end of angiogenesis .

The correct answer is c

………………………………………………………………………………………………………
……………………………………………..

594) Female patient , known case of VitD deficiency ,


smoking , and recurrent fall, which of the following is the
greatest exogenous risk for osteoporosis :

A. Advanced age .

B. Recurrent fall .

C. Vit D .

D. Smoking .

The correct answer is d

Advanced age & female gender are the most important factors
for osteoporosis but they are endogenous. Smoking is an
independent exogenous risk factor for osteoporosis

595) Blood sugar in DM type 1 is best controlled by :

A. Short acting insulin .

B. Long acting .

C. Intermediate .
D. Hypoglycemic agents .

E. Basal and bolus insulin .

The correct answer is e

Very vague question. We can exclude hypoglycemic agents.


Short acting insulin is best in emergencies like DKA as it can be
given IV. We can use either long acting alone daily or a mixture of
short & intermediate acting insulin daily. Basal & bolus , ( short
acting + intermediate or long ), bolus’ of short-acting or very-
short-acting insulin before meals to deal with the associated rise in
blood-sugar levels at these times. In addition, they take an evening
injection of long- or intermediate-acting insulin that helps
normalise their basal (fasting) glucose levels. This offers greater
flexibility and is the most commonly adopted method when
intensified insulin therapy is used to provide optimal glycaemic
control.

………………………………………………………………………………………………………
……………………………………………..

596) Which of the following is true regarding perths disease :

A. Commonly seen between 11-16 years of age .

B. Always unilateral .

C. May present by painless limp .

D. Characteristically affect the external rotation of hip .

E. More in female .

The correct answer is c

Perthes disease (Leg-Calve perthes disease) is a condition


affecting the hip joint where there is degenerative avascular
necrosis of the femoral head. It affects children aged 3-12 years &
is more common in males. It is most commonly but not always
unilateral ( 85% is unilateral ). It presents mainly by severe hip
pain & limping that increases by movement but it can present by
painless limp. It characteristically affects the internal rotation &
abduction of the hip & limits these movements.

597) Which of the following is true regarding gastric lavage :

A. Patient should be in the right lateral position .

B. It is not effective after 8 hours of aspirin ingestion .

The correct answer is b

lavage is effective only 1 hour after ingestion of any poison. After


that its ineffective

………………………………………………………………………………………………………
……………………………………………..

598) SSRI was prescribed to a patient with depression , the


effect is suspected to be within :

A. One day .

B. Two weeks .

C. Three to four weeks .

The correct answer is c

Allow 2 – 6 weeks to take effect , and treat for > 6 months

………………………………………………………………………………………………………
……………………………………………..

599) Randomized controlled study will be stronger by :

A. Systemic >>>>

B. Following at least 50 % of the participant .


???????

………………………………………………………………………………………………………
……………………………………………..

600) Partner lost his wife by AMI 6 months ago , presented by


loss of appetite , low mood , sense of guilt , what is the
diagnosis :

A. Beverament .

B. Major depression episode .

The correct answer is a

- Major depression is a psychiatric condition that occurs regardless


of events that happen in life, while normally most people would
have beverament after death of a close person.

601) An adult was presented by sore throat , congestion ,


fatigue , petechia in soft palate , tender spleen , and liver ,
what is the most likely diagnosis :

A. EBV .

Only 1 choice?

This could be Rubella infection as petechia in soft palate


(Forschheimer spots) occur in rubella also. But liver &
spleen affection are more common in EBV. Also since it is
an adult then we might suspect leukemia as the cause.

………………………………………………………………………………………………………
……………………………………………..

602) Patient was presented by blepharitis , acne roseca , but


no keratitis , what is the best treatment :

A. Topical chlorophenicol .

B. Topical gentamicin .
C. Oral doxycyclin .

The correct answer is c

………………………………………………………………………………………………………
……………………………………………..

603) Diabetic patient was presented by spastic tongue ,


dysarthria , spontenous crying what is the most likely
diagnosis :

A. Parkinson .

B. Bulbar palsy .

C. Pseudobulbar .

D. Myasthenia gravis .

The correct answer is c

This is a bit tricky. Bulbar palsy is the LMNL of the last 4 CN, while
pseudobulbar palsy is the UMNL of the last 4 CN. So spasticity of
tongue is UMNL. But Diabetes causes LMNL of cranial nerves due to
peripheral neuropathy. So maybe the cuase here is CNS affection
due to atherosclerosis from macroangiopathy of diabetes.

604) Which of the following is describe the normal


developmental stage for 6 months old child :

A. Sits without support .

B. Rolls front to back . (7-8 months)

C. No head lag . (3 months)

D. Stand alone . (1 year)


The correct answer is a

………………………………………………………………………………………………………
……………………………………………..

605) Which of the following personality is characterized by


inflexibility , perfectionism ?

A. OCD .

B. Not otherwise specified .

C. Narcissistic .

The correct answer is b

Obsessive compulsive personality disorder is different from


obsessive compulsive disorder and is characterized mainly by
perfectionism while OCD is characterized by repetitive actions due
to compulsion. if OCPD is present in the choices it would be
the answer.

………………………………………………………………………………………………………
……………………………………………..

606) What is the commonest cause of otorrhea :

A. Acute otitis media .

B. Basal skull fracture .

C. External otitis .

The correct answer is a

………………………………………………………………………………………………………
……………………………………………..

607) Patient was presented by ear pain , red tympanic


membrane , apparent vessels , with limited mobility of the
tympanic membrane , what the most likely diagnosis :
A. Acute otitis media .

B. Tympanic cellulitis .

C. Mastoditis .

The correct answer is a

608) What is the management of acute congestive glaucoma :

A. IV acetazolamide and topical pilocarpine . (true)

………………………………………………………………………………………………………
……………………………………………..

609) Eight years old child with late systolic murmur best
heard over the sterna border , high pitch , crescendo ,
decrescendo ,>> diagnosis is :

A. Physiological murmur .

B. Innocent murmur .

C. Ejection systolic murmur .

D. Systolic regurgitation murmur .

The correct answer is c ( I'm not sure )

………………………………………………………………………………………………………
……………………………………………..

610) Child was presented by coryza , skin rash ,


conjunctivitis , and multiple spots in the mouth :

A. Measles .

B. Rubella .
The correct answer is a.

Measles : due to conjunctivitis which is not present in rubella

………………………………………………………………………………………………………
……………………………………………..

611) Which of the following could be seen in patient with


bulimia :

A. Hypokalmeia .

B. Metabolic acidosis .

The correct answer is a

bulimia is aka bing eating which means the patient eats a lot
then does forced vomiting so there is loss of acids & electrolytes
which leads to hypokalemia & metabolic alkalosis.

612) Maximum spinal height is reached after menarche by


how many years ?

A. Months .

B. Two years .

C. Three years .

The correct answer is c

menarche is at 12-14 years and maximum height is about 17 years


………………………………………………………………………………………………………
……………………………………………..

613) Pregnant lady 34 weeks of gestation presented by


vaginal bleeding , which of the following is relevant to ask
about :

A. Smoking .

B. Desire of future pregnancy .

C. The result of last pap smear .

D. Hx of vaginal irritation .

The correct answer is a , because smoking is an important risk


factor for antepartum he

………………………………………………………………………………………………………
……………………………………………..

614) Female patient with marginal placenta previa , with 6 cm


dilated cervix , what is your management :

A. Fetal monitoring .

B. Delivery if fully dilated .

C. Tocolytics >

The correct answer is a ( I'm not sure )

I couldn’t find the answer correctly so based upon the given


data, fetal monitoring is a must in any case of high risk pregnancy.
Also as the PP is marginal, there is possibility of delivery without
severe PPH and the cervix is already dilated.
615) Which of the following drugs increase the survival in a
patient with heart failure :

A. Beta blocker .

B. ACE inhibitors .

C. Digoxin .

D. Nitrites .

The correct answer is b

New updated information. As ACE inhibitors inhibit aldosterone


which if present in high concentrations causes modification of the
cardiac myocytes in the long term.

………………………………………………………………………………………………………
……………………………………………..

616) Elderly patient presented by SOB , rales in auscultation ,


high JVP , +2 lower limb edema , what is the main
pathophysiology :

A. Left ventricular dilatation .

B. Right ventricular dilatation .

C. Aortic regurgitation.

D. Tricuscpid regurgitation .

The correct answer is a ( cofuuuuuuuse )

Difficult question. Here we have both symptoms of Lft


ventricular failure (SOB, Rales) & Rt ventricular failure (High JVP &
LL edema). So, more commonly lft ventricular failure leads to rt
ventricular failure due to overload and not vice versa . so the most
correct is Lft ventricular dilatation.
………………………………………………………………………………………………………
……………………………………………..

617) Patient with high output fistula , for which TPN was
ordered , after 2 hours of the central venous
catheterization , the patient become comatose and
unresponsive , what is the most likely cause :

A. Septic shock .

B. Electrolytes imbalance .

C. Delayed response of blood mismatch .

D. Hypoglycemia .

E. Hypernatremia .

The correct answer is a

618) Female patient presented by tender mass below the


axilla , with previous history of pores and black heads in
that’s area <<< what is your management :

A. Go to surgery immediately .

B. Oral antibiotics .

C. Topical antibiotics .

D. Steroid .

The correct answer is a , This is mostly an infected abscess.

………………………………………………………………………………………………………
……………………………………………..

619) Child was presented by erythema , and swelling in his


hand after 18 hours of bee sting , what is your management
in this case :
A. Epinephrine .

B. Antihistaminic drugs .

C. Hospitalization .

The correct answer is b

18 hours is a long duration so its unlikely to be anaphylactic shock


& more likely a regular reaction to bee stings so antihistaminics is
the best solution

………………………………………………………………………………………………………
……………………………………………..

620) Which of the following is true regarding metformin :

A. Main complication is hypoglycemia .

B. Can lead to weight gain .

C. It suppress the hepatic glauconeogenesis .

The correct answer is c

………………………………………………………………………………………………………
……………………………………………..

621) Child was presented by congested throat , coryza , high


grade fever , which of the following is true regarding this
condition :

A. Viral > bacterial .

B. Bacterial > viral .

C. Antibiotics should be given any way .

D. It is most likely due to EBV .

The correct answer is b ??????


622) Benign tumors of stomach represent almost :

A. 7 %

B. 21 %

C. 50 %

D. 90 %

The correct answer is a

………………………………………………………………………………………………………
……………………………………………..

623) 60 years old patient presented by recurrent venous


thrombosis including superior venous thrombosis , this
patient most likely has :

A. SLE .

B. Nephrotic syndrome .

C. Blood group O .

D. Antiphospholipid syndrome .

The correct answer is d

………………………………………………………………………………………………………
……………………………………………..

624) Elderly male patient was presented by left lower


abdomen tenderness , fever , elevated WBC count , what is
the most likely diagnosis :

A. Inflammatory bowel .

B. Diverticulitis .

The correct answer is b


………………………………………………………………………………………………………
……………………………………………..

625) 22 years old male patient was presented by recurrent


attacks of diarrhea , constipation , and abdominal pain
relieved after defecation , but no blood in the stool , no
weight loss : what is the diagnosis :

A. Irritable bowel Syndrome

626) Patient was presented by back pain relieved by


ambulation , what is the best initial treatment :

A. Steroid injection in the back .

B. Back bracing .

C. Physical therapy .

The correct answer is c

………………………………………………………………………………………………………
……………………………………………..

627) Patient with long history of constipation presented by


painful defecation , followed by bleeding , pain persist for
hours after defecation what is the diagnosis :

A. Anal fissure .

B. Thrombosed external piles .


C. Internal hemorrhoid .

The correct answer is a

………………………………………………………………………………………………………
……………………………………………..

628) Elderly male patient underwent colectomy for colon


cancer in which micrometastais was detected in the lymph
nodes , what is the best explaniation :

A. Good prognosis .

B. Liver metastasis .

C. It is sensitive to chemotherapy . (Dukes class C cancer best


for chemotherapy)

D. It is locally advanced .

The correct answer is c

………………………………………………………………………………………………………
……………………………………………..

629) IV drug abuser was presented by fever , arthralgia ,


conjunctival hemorrhage , what is the diagnosis :

A. Bacterial endocarditis .

………………………………………………………………………………………………………
……………………………………………..
630) Female patient was presented by dysurea , epithelial
cells were seen urine analysis , what is the explanation in
this case :

A. Contamination .

B. Infection .

The correct answer is b


631) Patient was presented by tremor , fever , palpitation ,
diagnosed as case of hyperthyroidism , what is your initial
treatment :

A. Surgery .

B. Radio iodine .

C. Beta blockers . Propylthioracil .

The correct answer is c , firstly B-blocker then Prophylthiouracil


because we are afraid of arrhythmias

………………………………………………………………………………………………………
……………………………………………..

632) Young female patient presented by pain during the first


2 days of menses , menarche was 2 years back , what is
your initial treatment :

A. OCP .

B. NSAID .

The correct answer is b , NSAID it is the best initial treatment for


dysmenorrheal

………………………………………………………………………………………………………
……………………………………………..

633) Female is planning to get pregnant , she want to get


MMR vaccine also , what is your action :

A. Delay the pregnancy 2-3 months after vaccination .

B. It is safe during pregnancy .

The correct answer is a


………………………………………………………………………………………………………
……………………………………………..
634) Which of the following is true regarding varicella vaccine
during breast feeding :

A. It is safe .

B. No breast feeding except after 3 days of the immunization .

Couldn’t find the answer

………………………………………………………………………………………………………
……………………………………………..
635) Patient underwent abdominal surgery due to intestinal
perforation many years back , presented by abdominal pain
, distension , constipation , what is the best investigation in
this case :

A. Barium enema .

B. Ultrasound .

C. Small bowel barium study .

The correct answer is c

636) True negative test is best described as following :

A. Not suspected to have the disease who actually do not have


.

………………………………………………………………………………………………………
……………………………………………..

637) Patient with AMI and multiple PVC , is your treatment for
this arrhythmia :

A. Amiadrone .

B. No treatment .
………………………………………………………………………………………………………
……………………………………………..

638) Adenosine dose should be reduced in which of the


following cases :

A. Chronic renal failure .

B. Patients on thiophyline .

The correct answer is a

………………………………………………………………………………………………………
……………………………………………..

639) Which the following is the commonest complication of


patient with chronic atrial fibrillation :

A. Sudden death .

B. Cerebra vascular accidents .

The correct answer is b , due to multiple atrial thrombi

………………………………………………………………………………………………………
……………………………………………..

640) Which of the following is most likely seen in case of


active glomerlonephritis :

A. RBC casts .

………………………………………………………………………………………………………
……………………………………………..

641) Target HbA1c is less than :

A. 6.5

B. 8
C. 9

The correct answer is a

642) Female patient with candida most likely has :

A. DM .

B. SLE .

The correct answer is a

………………………………………………………………………………………………………
……………………………………………..

643) Which of the following indicates good prognosis in


schizophrenia :

A. Family history of schizophrenia .

B. Gradual onset .

C. Flat mood .

D. Prominent affective symptoms .

E. No precipitating factors .

The correct answer is d

………………………………………………………………………………………………………
……………………………………………..

644) Pregnant woman with suspected DVT , what is the best


initial investigations :

A. Duplex US .

B. D dimer .

C. Platysomgraphy .

D. Venogram .
The correct answer is a

………………………………………………………………………………………………………
……………………………………………..

645) Which of the following is true regarding crohns disease :

A. Partial thickness involvement .

B. Fistula formation .

C. Continuous area of inflammation .

D. Mainly involve the recto sigmoid area .

The correct answer is b

646) Patient serology showed antibodies to the surface


antigen for hepatitis B , what is your diagnosis :

A. Previous infection or immunization .

B. Chronic carrier .

C. Highly infective .

The correcr answer is a

………………………………………………………………………………………………………
……………………………………………..
647) Young male patient presented by acute scrotal pain , US
showed reduced blood flow , what is the diagnosis :

A. Testicular torsion .

B. Trauma .

C. Infection .

D. Hernia .

The correct answer is a

………………………………………………………………………………………………………
……………………………………………..

648) Female patient is sure that she is pregnant for 2


months , on examination , the uterus is larger than
suspected , B-hcg is very high , the doctor diagnosed her as
having tumor which is chemo sensitive , what is the
diagnosis :

A. Ovarian cancer

B. Endometrial cancer .

C. Gestational trophoblastic .

The correct answer is c

………………………………………………………………………………………………………
……………………………………………..

649) Which of the following is true regarding infertility :

A. It is Failure to conceive withing 6 months . (1 year)

B. Male factor > female factors . (the reverse)

C. It could be due to high prolactin levels .

D. Rare to be due anovulotion . (common)

E. Only diagnosed by HSG . (need full lab & imaging


investigations)
The correct answer is c

650) 32 years old female patient presented by irregular


menses , menses occurs every two months , on examination
every thing is normal , which of the following is the LEAST
important test to ask about first :

A. CBC .

B. Pelvic US .

C. Coagulation profile .

D. DHES .

The correct answer is c

………………………………………………………………………………………………………
……………………………………………..

651) Patient with early rheumatoid arthritis , what is your


management to decrease the limitation of movement :

- Do not use analgesics or steroids, use DMARDs like


methotrexate or antiTNF, hydroxychloroquine

………………………………………………………………………………………………………
……………………………………………..

652) Patient with truncal obesity , easy bruising ,


hypertension , buffalo hump , what is the diagnosis :

A. Cushing .
………………………………………………………………………………………………………
……………………………………………..

653) Patient is known case of cervical spondylolysis ,


presented by parasthesis of the little finger , with atrophy
of the hypothenar muscles , EMG showed cubital tunnel
compression of the ulnar nerve , what is your action now :

A. Ulnar nerve decompression .

B. Steroid injection .

C. CT scan of the spine .

The correct answer is a

654) Well known case of SCD presented by plueritic chest pain


, fever , tachypnea , respiratory rate was 30 , oxygen
saturation is 90 % what is the diagnosis :

A. Acute chest syndrome .

B. Pericarditis :

C. VOC .

The correct answer is a , or pneumonia would be more correct if it


was the answer

………………………………………………………………………………………………………
……………………………………………..

655) Child with hemangioma around the eye , operation


should be done within ……… :
A. 1 Week

B. 3 months .

C. 6 months .

The correct answer is a ( I'm not sure )

We only fear of reduction in the function of the eye due to


hemangioma so it should be removed.

………………………………………………………………………………………………………
……………………………………………..

656) Which of the following is the recommended diet to


prevent IHD :

A. Decrease the intake of meat and dairy .

B. Decrease the meat and bread .

C. Increase the intake of fruit and vegetables .

????

The correct answer is c

………………………………………………………………………………………………………
……………………………………………..

657) Which of the following can lead to polyhydrmnios :

A. Duodenal atresia .

B. Renal agenesis .  Oligohydramnios

C. Post term pregnancy . Oligohydramnios

D. Diabetes inspidious .

The correct answer is a


658) Adult polycystic kidney disease is inherited as:

A. Autosomal dominant .

B. Autosomal recessive .

C. X linked .

The correct answer is a

………………………………………………………………………………………………………
……………………………………………..

659) The best treatment for bacteroid :

A. Clindamycin .

………………………………………………………………………………………………………
……………………………………………..

660) You have an appointment with your patient at 10 am who


is newly diagnosed DM , you came late at 11 am because
you have another complicated patient , what are you going
to say to control his anger :

A. Do not say any thing .

B. Told him that there is another patient who really need your
help .

Q not complete, but may be The correct answer is b

………………………………………………………………………………………………………
……………………………………………..

661) Well known case of DM was presented to the ER with


drowsiness , in the investigations : Blood sugar = 400
mg/dl , pH = 7.05 , what is your management ?

A. 10 units insulin + 400 cc of dextrose .


B. 0.1 unit/kg of insulin , subcutaneous .

C. NaHCO .

D. One liter of normal saline .

The correct answer is d

………………………………………………………………………………………………………
……………………………………………..

662) What is the best method for history taking :

A. Yes or no questions .

B. Open ended .

C. Silent listening .

The correct answer is b

663) What you will give to prevent hemorrhagic disease of


newborns :

A. Vitamin K .

………………………………………………………………………………………………………
……………………………………………..
664) Arterial injury is characterized by :

A. Dark in color and steady .

B. Dark in color and spurting .

C. Bright red and steady .

D. Bright red and spurting .

The correct answer is d

………………………………………………………………………………………………………
……………………………………………..
665) 14 years old girl complaining of painless vaginal bleeding
for 2-4 days every 3Weeks to 2 months ranging from
spotting to 2 packs per day; she had 2ry sexual ccc 1 year
ago and had her menstruation since 6 months on clinical
examination she is normal sexual ccc, normal pelvic exam
appropriate action

a. OCP can be used

b. You should ask for FSH and prolactin level

c. Don’t do anything & explain this is normal

the correct answer is c

………………………………………………………………………………………………………
……………………………………………..
666) 4 years old child what can he do

a. Copy square and triangle

b. Speak in sentences

the correct answer is a , if copy square 4 years , but if copu circle 3


years

………………………………………………………………………………………………………
……………………………………………..
667) baby can sit without support, walk by holding fourniture,
Pincer grasp, pull to stand how old is he

a. 8 months

b. 10 months

c. . 12 month

d. 18 month

The correct answer is b

668) Patient came after deep laceration at the anterior part


of the wrist:

a. Wrist drop

b. Sensory loss only

c. Claw hand

d. Unable to do thumb opposition


The correct answer is d

2- Radial nerve injury : wrist drop , common with humers injury


(humers groove)

3- Unlar nerve injury : claw hand , common with elbow injury

4- Median nerve inury :unable to do thumb opposition , common


with wrist injury

………………………………………………………………………………………………………
……………………………………………..

669) Best way to diagnose post streptococcus


glomerulonephritis (spot diagnosis):

a) Low C3

b) ?or RBC casts

c) ?

The correct answer is a ( I'm not sure )

With these MCQs , I think the correct answer is a , but most


sensitive in diagnosis of post streptococcus glomerulonephritis is
increase ASO titer : lumpy-bumby immunofluorescence. or
(streptozyme test, which tests antibodies to ASO) ( if ASO
titer written with MCQs , it is correct answer .

……………………………………………………………………………………………………
………………………………………………..

670) COPD patient with emphysema has low oxygen prolonged


chronic high CO2, the respiratory drive is maintained in this
patient by:

a. Hypoxemia

b. Hypercapnemia

c. Patient effort voluntary

d. ?
The correct answer is a ( I'm not sure )

………………………………………………………………………………………………………
……………………………………………..

671) Child with ear pain with positive pump test for tympanic
membrane, treatment is:

a. Maryngiotomy

b. Amoxicillin/Potassium …….

The correct answer is a

672) Patient has fatigue while walking last night. He is on


atrovastatin for 8 months, Ciprofluxacin, Dialtizem and
alphaco….. the cause of this fatigue is:

a. Dialtizem and Atrovastatin

b. Atrovastatin and Ciprovluxacin

c. Atrovastatin and Alphaco

d. ?

the correct answer is b ( I'm not sure )

………………………………………………………………………………………………………
……………………………………………..

673) Patient with ischemic stroke present after 6 hours, the


best treatment is:

a) ASA

b) TPA

c) Clopidogril

d) IV heparin

e) Other anticoagulant
The correct answer is a

1- TPA : administered within 3h of symptoms onset ( if no


contraindication)

2- ASA: use with 48h of ischemic stroke to reduce risk of


death.

3- Calopidogril : can be use in acute ischemic

4- Heparin & other anticoagulant : in patient has high risk of


DVT or AF

………………………………………………………………………………………………………
……………………………………………..

674) Patient with lateral and vertical diplobia, he can’t abduct


both eyes, the affected nerve is:

a) II

b) III

c) VI

d) V

I think this is diplopa because nerve 4 , but with this Q I choose


c

675) Infant with heamangioma on forehead obscuring the


vision, best thing to do is to remove the lesion to avoid
amblyobia, after:

a) One day
b) One week

c) One month

d) 6 months

e) One year

The correct answer is b ( I'm not sure )

………………………………………………………………………………………………………
……………………………………………..

676) Photophobia, blurred vision, keratic ?? behind cornea


and cells in anterior chamber, the best treatment is :

a) Topical antifungal

b) Topical Acyclovir

c) Antibiotic

d) ?

The correct answer is b

………………………………………………………………………………………………………
……………………………………………..

677) Most common medical problems faced in primary health


care is:

a) Coryza

b) UTI

c) Hypertension

d) Diabetes

The correct answer is a


………………………………………………………………………………………………………
……………………………………………..

678) Old lady afraid of Osteoprosis, to avoid the risk, you


should advise her to do:

a) Weight bearing exercise

b) ?

c) ?

d) ?

Q is not complete.

679) Obese lady with essential hypertension, lab work


showed: high Na, High K, The reason for Hypertension is:

a) Obesity

b) High Na intake

c) High K intake

d) ?

The correct answer is a , more than 85% of essential


hypertension with BMI > 25 .

……………………………………………………………………………………………………
………………………………………………..

680) Patient with eruptive purpuric rash,


hepatosplenomegaly, …………

a) ?

b) ?

c) ?
d) ?

………………………………………………………………………………………………………
……………………………………………..

681) Single thyroid nodule showed high iodine uptake, best


treatment is:

a) Radio Iodine 131

b) Send home

c) Antithyriod medication

d) Excision if present

The correct answer is c

………………………………………………………………………………………………………
……………………………………………..

682) Athletic with tina pedis best treatment is:

a) Topical antifungal

b) Systemic antifungal

c) Drug starts with trebenafine

d) ?

The correct answer is a

683) Young patient with congested nose, sinus pressure,


tenderness and green nasal discharge, has been treated
three times with broad spectrum antibiotics previously,
what is your action?

a) Give antibiotic
b) Nasal corticosteroid

c) Give anti histamine

d) Decongestant

The correct answer is b

………………………………………………………………………………………………………
……………………………………………..

684) Thyrotoxicosis include all of the following, Except:

a) Neuropathy

b) Hyperglycemia

c) Peripheral Proximal myopathy

d)

Q is not complete , all MCQs are wrong , may be choice d is


correct if mention .

………………………………………………………………………………………………………
……………………………………………..

685) Male patient has hair loss started as fronto-temporal and


moving toward the vertex (top of the head) the diagnosis
is:

a) Androgenic alopecia

b) Tinea Captus

c) ?

d) ?

The correct answer is a

………………………………………………………………………………………………………
……………………………………………..
686) Young healthy male has abdominal pain after basketball.
Examination fine except for Left paraumbilical tenderness,
what to do:

a) Abdominal US

b) Flat plate graph

c) Send home & reassess within 48 hours

d) ?

The correct answer is c ( I'm not sure ) , confuse with choice a

687) 10 year-old boy with ….to tell that spinal cord length will
stop after:

a) ?

b) ?

Length stop at L1 \ L2

………………………………………………………………………………………………………
……………………………………………..

688) Child with atopic dermatitis at night has stridor plus


barking cough on and off from time to time, diagnosis is:

a) BA

b) Croup

c) Spasmadic Croup

d) ?

The correct answer is c


Spasmadic croup : recurrent sudden upper airway
obstruction which present as sridor and cough . approximately
50% of children have atopic disease.

………………………………………………………………………………………………………
……………………………………………..

689) Sickle cell anemia patient presented with asymptomatic


unilateral hip pain, most likely diagnosis is:

a) Septic arthritis

b) Avascular Necrosis

c) ?

The correct answer is b

………………………………………………………………………………………………………
……………………………………………..

690) Best drug to treat depression in children and adolescent


is:

a) Fluxetine (Prozac)

b) ?

………………………………………………………………………………………………………
……………………………………………..

691) Old male with neck stiffness, numbness and parasthesia


in the little finger and ring finger and positive raised hand
test, diagnosis is:

a) Thoracic outlet syndrome

b) Impingement syndrome

c) Ulnar artery thrombosis

d) Do CT scan for Cervical spine

The correct answer is a


692) Patient diagnosed with obstructive jaundice best to
diagnose common bile duct obstruction:

a) ERCP

b) US

c) ?

The correct answer is a

………………………………………………………………………………………………………
……………………………………………..

693) Endometriosis best diagnosed by

a) US

b) Laproscopy

c) Lapratomy

d) ?

The correct answer is b

………………………………………………………………………………………………………
……………………………………………..

694) Child was sick 5 days ago culture taken showed positive
for meningococcal. Patient now at home and asymptomatic
your action will be:

a) Rifampicin

b) IM Ceftrixone

c) ?

d) ?

Q not complete ( i.m ceftriaxon single dose or oral rifampicin


2-5 days )
………………………………………………………………………………………………………
……………………………………………..

695) Infant with bright blood, black stool and foul smelling
stool. Best way to know the diagnosis:

a) US

b) Radio Isotop scan

c) Angiogram

d) ?

All MCQs are wrong , if stool analysis is present , it is correct.

696) Primary hyperaldosteronism associated with:

a) Hypernatremia

b) Hypomagnesemia

c) Hypokalemia

d) Hyperkalemia

The correcr answer is c

………………………………………………………………………………………………………
……………………………………………..
697) A vaccination for pregnant lad with DT

a) Give vaccine and delivery within 24 hrs

b) Contraindicated in pregnancy

c) Not contraindicated in pregnancy


d) ?

The correct answer is c

………………………………………………………………………………………………………
……………………………………………..
698) A study done to assess the risk of long taking Ca in two
groups the diseased group with long Ca plus control
according to geographical location, site, and population. It
adds (??) this type of study:

a) Cohort

b) Case Control (retrospective)

c) Correlation study

d) ?

The correct answer is a

Cross-Sectional Study:
status of individual with respect to presence and absence of
both exposure and disease assessed at one point in time

case control study ( retrospective ) :


samples a group of people who already have a particular
outcome (cases) and
compares them to a similar sample group without that outcome
(controls)

cohort study : ( prospective, incidence , longitudinal )


subjects are sampled and as a group are classified on the basis
of presence or
absence of exposure to a particular risk factor
………………………………………………………………………………………………………
……………………………………………..

699) Define Epidemiology

a) ? “The study of the distribution and determinants of


disease prevalence in man”

700) Case Control description


a) Start with the outcome the FU risk factors

b) ?

……………………………………………………………………………………………………
………………………………………………..
701) Using gastric lavage :

a) Useless after 8 hours of ASA ingestion

b) No benefit after 6 hours of TCA ingestion

c) ?

The correct answer is a

………………………………………………………………………………………………………
……………………………………………..
702) 10 years old child with rheumatic fever treated early, no
cardiac complication. Best to advice the family to continue
prophylaxis for:

a) 1 month

b) 3 ys

c) 4 ys

d) 15 ys

The correct answer is d ( I'm not sure , if there is 11 years


choose it )

American heart associated has recommended all patient with


history rheumatic fever be placed on longe term penicillin
prophylaxic. Duration of prophylaxis depends on presence or
absent of carditis, but for children without carditis , duration
minimum 5 years or at age 21, whichever is longer.
703) Picture of large neck mass only no other manifestations
or organomegaly or lymphadenopathy, diagnosis is:

a) Mononucleosis

b) I would say Goiter

c) ?

d) Lymphoma

The correct answer is b

………………………………………………………………………………………………………
……………………………………………..

704) Patient with nausea, vomiting, and diarrhea developed


postural hypotension. Fluid deficit is:

a) Intracellular

b) Extracellular

c) Interstitial
The correct answer is b
………………………………………………………………………………………………………
……………………………………………..

705) Cardiac syncope:

a) Gradual onset

b) Fast recovery

c) Neurological sequence after

d) ?

The correct answer is b

………………………………………………………………………………………………………
……………………………………………..

706) Best way to decrease pain in elderly with bilateral knee


pain and crepitation is:

a) NSAID

b) Decrease weight

c) Exercise

d) ?

The correct answer is b , (I'm not sure )

707) Young female with whitish grey vaginal discharge KOH


test ?? smell fish like diagnosis is:

a) Gonorrhea

b) Bacterial Vaginosis

c) Trachomanous Vaginalis
d) ?

The correct answer is b

………………………………………………………………………………………………………
……………………………………………..

708) Old lady with osteoporosis asked for treatment for


prevention:

a) VIT. D

b) VIT. E

c) Retonic Acid

d) ?

The correct answer is a

………………………………………………………………………………………………………
……………………………………………..

709) Young male with morning stiffness at back relieved with


activity and uveitis:

a) Ankylosing Spondylitis

b) ?

………………………………………………………………………………………………………
……………………………………………..

710) Best way to prevent infection in medical practice in


pediatric

a) Wear gloves

b) Wash hand

c) Wear mask

d) Wear gown
The correct answer is b

………………………………………………………………………………………………………
……………………………………………..

711) High risk for developing colon cancer in young male is:

a) Smoking, high alcohol intake, low fat diet

b) Smoking, low alcohol intake, high fat diet

c) Red meat diet, garden’s disease (Gardner syndrome)

d) Inactivity, smoking

The correct answer is c

712) Alternative therapy for severe depression and resistance


to anti-depressant medications are:

a) SSRI

b) TCA

c) ECT

The correct answer is c

………………………………………………………………………………………………………
……………………………………………..

713) Patient had history of pancreatic cancer on


chemotherapy then improved completely, came to doctor
concerning about recurrence of cancer and a history of
many hospital visits. This patient has:

a) Malingering

b) Hypochondriasis

c) Factitious

d) Conversion
The correct answer is b

………………………………………………………………………………………………………
……………………………………………..

714) Patient came with neck swelling, ,moves when patient


protrude his tongue. Diagnosis is:

a) Goiter

b) Tyroglossus Cyst

c) Cystic Hygroma

d) ?

The correct answer is b

………………………………………………………………………………………………………
……………………………………………..

715) Pregnant patient came with neck swelling and multiple


nodular non-tender goiter the next evaluation is:

a) Thyroid biopsy

b) Give anti-thyroid medication

c) Radiation Iodine

d) TSH & Free T4, or just follow up

The correct answer is d

716) Young patient with HTN came complaining of high blood


pressure and red, tender, swollen big left toe, tender
swollen foot and tender whole left leg. Diagnosis is:

a) Cellulitis

b) Vasculitis
c) Gout Arthritis

d) ??

The correct answer is a , because tender and swollen whole left


leg.

………………………………………………………………………………………………………
……………………………………………..

717) What is the injection that is routinely given to newborn


to inhibit hemorrhage:

a) Vit. K

b) Vit. C

c) Vit. D

d) Vit. E

The correct answer is a

………………………………………………………………………………………………………
……………………………………………..

718) Patient with strong genetic factor for colon cancer, what
is the medication that could decrease the risk of colon
cancer:

a) Folic Acid.

b) Vit. C

c) Vit. K or A

d) Vit. E

The correct answer is a ( folaic acid and vit. C both are prevent
colon cancer , but folat reduce risk in people who genic
predisposing )
………………………………………………………………………………………………………
……………………………………………..

719) Patient with asthma, well controlled by albutarol, came


complaining of asthma symptoms not respond to albutarol,
what medication could be added:

a) Corticosteroid inhaler

b) Long acting B-agonist

c) Oral corticosteroid

d) Theophyline

The correct answer is a

720) Henosch-Scholen purpura affect:

a) Capillary

b) Capillary and venule

c) Arteriole, capillary and venule

d) Artery to vein

The correct answer is c

………………………………………………………………………………………………………
……………………………………………..

721) Contraceptive pill that contain estrogen increase risk of:

a) Breast Ca

b) Ovary Ca

c) Cervical Ca

d) ?
The correct answer is a

………………………………………………………………………………………………………
……………………………………………..

722) Patient came with upper respiratory tract infection with


red conjunctiva, the cause is:

a) Viral infection

b) Bacterial infection

c) Fungal infection

d) ?

The correct answer is a

………………………………………………………………………………………………………
……………………………………………..

723) Healthy patient with family history of DM type 2, the


most factor that increase chance of DM are:

a) HTN and Obesity

b) Smoking and Obesity

c) Pregnancy and HTN

d) Pregnancy and Smoking

The correct answer is a

724) Patient complaining of back pain and hypersensitive skin


of the back, on examination, patient had rashes in the
back, tender, red base distributed as blunt shape on the
back, diagnosis is:

a) Herpes Zoster
b) CMV

c) ?

d) ?

The correct answer is a ( q not complete )

………………………………………………………………………………………………………
……………………………………………..

725) Old patient complaining of hematuria, on investigation,


patient has bladder calculi, most common causative
organism is:

a) Schistosoma

b) CMV

c) ? virus

d) ? virus

The correct answer is a

………………………………………………………………………………………………………
……………………………………………..

726) Blood culture show gram negative rod shape that grow
only on charcoal free fungal organism is:

a) Staph. Aureus

b) Chlamydia

c) Klebsiella

d) Mycoplasma

All above are wrong , legionella : grame negative rod growth on


charcoal agar
………………………………………………………………………………………………………
……………………………………………..

727) Klebesilla feacalis cause the following disease:

a) Pneumonia

b) ?

c) ?

d) ?

Enterococcus faecalis  Cause Gastroenteritis

………………………………………………………………………………………………………
……………………………………………..

728) Dermatomyositis came with the following symptoms:

a) Proximal muscle weakness

b) Proximal muscle tenderness

c) ?

d) ?

The correct answer is a

………………………………………………………………………………………………………
……………………………………………..

729) Bursitis of the elbow joint caused by:

a) Elbow trauma

b) Autoimmune disease

c) Staph. Aureus

d) ? rupture of bursa
The correct answer is a

………………………………………………………………………………………………………
……………………………………………..

730) Patient came with symptoms of anxiety including


palpitation, agitation, and worry. The first best line for
treatment is:

a) SSRI

b) TCA

c) B-blocker

d) MAOI

The correct answer is a , I'm not sure

May be the correct answer is b because there is agitation ( also,


side effect of SSRI)

………………………………………………………………………………………………………
……………………………………………..

731) Pregnant diagnosed with UTI. The safest antibiotic is:

a) Ciprofloxacin

b) Ampiciln

c) Tetracycline

d) ?

With these MCQs , the correct answer is b , but if present


nitrofurantion is more accurate answer . UTI in pregnancy
treated by : nitrofurantion or cephalosporine ( 3– 7 days ) in
symptomatic or asymptomatic UTI . avoid fluroroquinolone
( which include : ciprofloxacin, gatifloxacine, levofloxacin,
norfloxacin ).

732) Patient is complaining of right side pharynx tenderness


on examination patient had inflamed right tonsil and
redness around tonsil with normal left tonsil. The diagnosis
is:
a) Parenchymal tonsillitis

b) Quinse parapharyngeal abscess

c) ?

d) ?

The correct answer is b

………………………………………………………………………………………………………
……………………………………………..

733) Patient came complaining of fever, night sweating, and


hemoptysis with positive PPD test. Examination was
normal, CXR shows infiltrate of left apical lung but in lateral
X-ray showed nothing the repeated PPD test showed
normal result diagnosis is:

a) Sarcoidosis

b) Reactivated TB

c) Mycoplasma infection

d) Viral infection

The correct answer is b

……………………………………………………………………………………………………
………………………………………………..

734) Femal patient came with lower abdominal pain, fever on


exam patient has lower abdominal tenderness and tender
cervical fornix, the most appropriate way to diagnose the
problem is:

a) Laproscopy

b) Heterosalpingography

c) Abdominal CT

d) Radionuclar Study
The correct answer is a

………………………………………………………………………………………………………
……………………………………………..
735) The best non-medical therapy is proven to be of benefit
for osteoarthritis is:

a) Muscle strength exercise

b) Give NSAID

c) Back slap

d) ?

The correct answer is a

736) Hematological disease occurs in children, treated with


heparin and fresh frozen plasma what is the disease:

a) Hemophilia A

b) Hemophilia B

c) Von-wille brand disease

d) DIC thrombosis

The correct answer is d

………………………………………………………………………………………………………
……………………………………………..

737) Child with URTI is complaining of bleeding from nose,


gum and bruising the diagnosis is:

a) Hemophilia A

b) ITT

c) ?

d) ?
The correct answer is b

………………………………………………………………………………………………………
……………………………………………..

738) Patient came complaining of vague abdominal pain for 6


hours then shifted to right lower quadrant diagnosis is :

a) Acute appendicitis

b) Diverticulitis

c) ?

d) ?

The correct answer is a

………………………………………………………………………………………………………
……………………………………………..

739) Female patient is complaining of abdominal distension,


fever and nausea abdominal x-ray showed (Ladder sign)
management is:

a) Colostomy

b) Ileus treatment

c) Rectal de-obstruction

d) ?

The correct answer is b ( I'm not sure because Q IS NOT


COMPLETE )

740) The best stimulus for breast milk secretion is :

a) Estrogen

b) Breast feeding
c) ?

The correct answer is b

………………………………………………………………………………………………………
……………………………………………..

741) Female patient did urine analysis shows epithelial cells in


urine, it comes from:

a) Vulva

b) Cervix

c) Urethra

d) Ureter

The correct answer is c

………………………………………………………………………………………………………
……………………………………………..

742) All of the following are risk factors for heart disease
except:

a) High HDL

b) Male

c) Obesity

The correct answer is a

………………………………………………………………………………………………………
……………………………………………..

743) The most signs and symptoms of abruption of placenta


is:

a) Vaginal bleed
b) Fetal distress

c) Uterus pain and back pain

d) Abnormal uterine contraction

The correct answer is a

744) Sign of severe hypokalemia is:

a) P-wave absence

b) Peak T-wave

c) Wide QRS complex

d) Seizure

The correct answer is d , ( I'm not sure , may be choice e if


mention , is corect )

severe hypokalemia is defined as a level less than 2.5


mEq/L.

Severe hypokalemia is not linked with any symptoms,


but may cause:
1- muscle
2- myalgia or muscle pain
3- disturbed heart rhythm including ectopy (disturbance of the
electrical conduction system of the heart where beats arise from
the wrong part of the heart muscle)
4- serious arrhythmias (electrical faster or slower than normal)
5- greater risk of hyponatremia (an electrolyte disturbance in
humans when the sodium concentration in the plasma decreases
below 135 mmol/L) with confusion and seizures
ECG changes in hypokalemia :
1-T-wave flattening
2-U-wave : ( additional wave after the T wave )
3-ST – segment depression

ECG changes in hyperkalemia :


1- peak T wave
2- wide QRS ( in severe case )
3- PR prolong ( in severe case )
4- loss of P wave

………………………………………………………………………………………………………
……………………………………………..

745) Child came with his father and high BMI and look older
than other children with same age, on exam child has >95 th
percentile of weight and tall, management is:

a) Observe and appoint

b) Life style change

c) Give program to decrease the weight

d) ?

The correct answer is a

746) Pregnant on 36th week came with 7 cm cervical width at 0


station. During birth, CTG shows late deceleration,
management is:

a) Give Oxytocin

b) O2 and change mother position

c) Give Mg sulfate

d) ?

The correct answer is b


Type of etiology management
deceleration

early Head compression No treatment


from uterine
contraction (normal )

late Uteroplacenta • Place patient on side


insufficiency and fetal • Discontinue oxytocin.
hypoxima • Correct any hypotension
• IV hydration.
• If decelerations are associated with
tachysystole consider terbutaline 0.25
mg SC
• Administer O2
• If late decelerations persist for more
than 30 minutes despite the above
maneuvers, fetal scalp pH is indicated.
• Scalp pH > 7.25 is reassuring, pH 7.2-
7.25 may be repeated in 30 minutes.
• Deliver for pH < 7.2 or minimal
baseline variability with late or
prolonged decelerations and inability to
obtain fetal scalp pH

variable Umbilical cord • Change position to where FHR


compression pattern is most improved.
Trendelenburg may be helpful.
• Discontinue oxytocin.
• Check for cord prolapse or imminent
delivery by vaginal exam.
• Consider amnioinfusion
•  Administer 100% O2

747) The way to determine the accuracy of occult blood test


for 11,000 old patients is by measuring:

a) Sensitivity

b) Specificity
c) Positive predictive value

d) Negative predictive value

The correct answer is a

………………………………………………………………………………………………………
……………………………………………..

748) Sickle cell patient, asymptomatic with history of


recurrent gall-stones and recurrent crisis the management
is:

a) Cholecystectomy

b) Hydroxyurea

The correct answer is a

………………………………………………………………………………………………………
……………………………………………..

749) Patient came with HTN, KUB shows small left kidney,
arteriography shows renal artery stenosis, what is the next
investigation:

a) Renal biopsy

b) Renal CT scan

c) Renal barium

d) Retrograde pyelography

The correct answer is

………………………………………………………………………………………………………
……………………………………………..

750) The way to differentiate between low iron level from iron
deficiency anemia and anemia of chronic disease is:
a) Ferritin

b) TIBC

c) Serum Iron

d) Serum Transferrin

The correct answer is a

751) Patient came with hallucination and illusion the


medication that should be given is:

a) Carbamezapin

b) ? Haloperidol

c) ?

Q not complete, but with this scenario and MCQs , the correct
answer is b

………………………………………………………………………………………………………
……………………………………………..

752) As doctor if you see patient and you face difficulty to get
accurate information from him the best tactic to do it is:

a) Ask direct question

b) Ask open question

c) Control way of discussion

d) ?

The correct answer is a

………………………………………………………………………………………………………
……………………………………………..
753) (long question) patient came with MDD so during
communication with patient you will find :

a) Hypomania

b) Late morning awake

c) Loss of eye contact

d) ?

The correct answer is b

………………………………………………………………………………………………………
……………………………………………..

754) Child patient after swimming in pool came complaining of


right ear tenderness on examination patient has external
auditory canal redness, tender, and discharge the
management is:

a) Antibiotics drops

b) Systemic antibiotics

c) Steroid drops

d) ?

The correct answer is a

755) Child came with inflammation and infection of the ear the
most complication is:

a) Labrynthitis

b) Meningitis
c) Encephalitis

d) Mastoiditis

The correct answer is d

………………………………………………………………………………………………………
……………………………………………..

756) Elderly patient complaining of urination during night and


describe when he feel the bladder is full and need to wake
up to urinate, he suddenly urinate on the bed this is:

a) Urgency incontinence

b) Urge incontinence

c) Stress incontinence

d) Flow incontinence

The correct answer is d

types Hx of urine loss mechanism treatment

total Uncontrolled loss at Loss of sphincter surgery


all times and in all efficiency or abnormal
position connection b\w urinary
tract and skin( fistula)

stress After increase Urethral sphincter Kegal


abdominal pressure insufficiency due to exerciseand
(coughing, sneezing, laxity of pelvic floor pessary
lifting ) musculature , common
in multiparous women surgery

urge Strong, unexpected Detrusor hyperreflexia Anticholinergic –


urge to void that is or sphincter medication or
unrelated to position dysfunction or TCA : behavior
or activity neurologic disorder training

overflo Chronic urinary Increase intravesical Placement of


w retention pressure that just urethra
exceed the outlet catheter , treat
resistance , allow small underlying
amount of dribble out causes

757) Newborn came with red-lump on left shoulder, it is:

a) Hemangioma

b) ?

c) ?

Q is not complete

………………………………………………………………………………………………………
……………………………………………..

758) Child came to ophthalmology clinic did cover test, during


eye cover , his left eye move spontaneously to left, the
most complication is:

a) Strabismus

b) Glaucoma

c) Myobloma

d) ?

The correct answer is a

………………………………………………………………………………………………………
……………………………………………..

759) Newborn came with congenital hepatomegaly, high LFT,


jaundice the most organism cause this symptoms is:

a) Congenital TB

b) Rubella
c) HIV

d) CMV

The correct answer is d

………………………………………………………………………………………………………
……………………………………………..

760) Gross motor assessment at age of 6 months to be asked


is:

a) Sitting without support

b) Standing

c) Role from prone to supine position

d) Role from supine to prone position

The correct answer is a

761) Female child came with precocious puberty the most


cause is:

a) Idiopathic

b) Adrenal tumor

c) Brain tumor

d) ? ovarian tumor

The correct answer is a


………………………………………………………………………………………………………
……………………………………………..

762) Hemorrhoid usually occurs in:

a) Pregnancy and portal HTN

b) ?

c) ?

d) ?

Q is not complete

Most common causes of hemorrhoid : increase straining


( constipation ) , portal HTN , increase abdominal pressure
( chroming cough , pelvic tumor ) , obesity , pregnancy ,
smoking

………………………………………………………………………………………………………
……………………………………………..

763) The immediate urgent referral of child that take

a) 10 pills contraceptive

b) 10 pills antibiotics?

c) 75 mg ?

?????

………………………………………………………………………………………………………
……………………………………………..

764) Most unwanted side effect of anti-cholinergic drugs is :

a) Constipation

b) ?

c) ?
more than 50% of patients taking anticholinergic have side
effects : dry mouth, blurry vision, constipation and urinary
retention.

765) Patient with DM II with good vision, to prevent eye


disease (Retinal back ground) to develop is to avoid:

a) HTN, Smoking

b) Obesity, Smoking

c) HTN, Obesity

d) ?

The correct answer is c

The risk factors that increase diabetic retinopathy


background are:

1- HTN

2- Poor glucose control or long case D.M

3- Raised level of fat ( cholesterol)

4- Renal disease

5- Pregnancy ( but not in diabetes caused by pregnancy )


………………………………………………………………………………………………………
……………………………………………..

766) Best medication to be given for GDM (gestational) is:

a) Insulin

b) Metformin

c) ?

The correct answer is a

767) A child is complaining of severe headache which is


unilateral, throbbing and aggravated by light, diagnosis is:

a) Migraine

b) Cluster Headache

c) Stress Headache

d) ?
The correct answer is a

………………………………………………………………………………………………………
……………………………………………..

768) The most important factor for smoker to quit is :

a) Patient desire

b) Give nicotine pills

c) Give programmed plan

d) Change life style

The correct answer is a

………………………………………………………………………………………………………
……………………………………………..

769) Patient is complaining of irritation, tachycardia, night


sweating, labs done showed TSH: Normal, T4: High,
diagnosis is:

a) Grave’s disease

b) Secondary Hypothyriodism

c) Hashimoto’s thyroiditis

d) ?

All MCQs are wrong , choice d may be correct if mention

Normal TSH and increase T4 : in thyroid tropin


secretary pitutay adenoma = thyroid resistance .

………………………………………………………………………………………………………
……………………………………………..

770) The most active form is:

a) T4
b) T3

c) TSH

d) TRH

The correct answer is b

771) Middle age man found to have heaviness in his groin. On


physical examination there was swelling just above his
testis which apparent with valsalva maneuver. What is the
diagnosis:

a) Direct inguinal hernia

b) Indirect inguinal hernia

c) Femoral Hernia

d) Testicular mass

e) Hydrocele

f) Varicocele

The correct answer is b

………………………………………………………………………………………………………
……………………………………………..

772) Gardener has recurrent conjunctivitis. He can’t avoid


exposure to environment. In order to decrease the
symptoms in the evening, GP should advise him to:

a) Cold compression

b) Eye irrigation with Vinegar Solution

c) Contact lenses

d) Antihistamines
The correct answer is d

………………………………………………………………………………………………………
……………………………………………..
773) 48 year-old male complaining of lower back pain with
morning stiffness for 30 minutes only. On exam he was
having spasm centrally on the lower back. What is the
appropriate management :

a) Epidural steroids injection

b) Back brace

c) Facet lysis

d) Physiotherapy

The correct answer is d

………………………………………………………………………………………………………
……………………………………………..
774) 41 weeks pregnant lady last biophysical profile showed
oligohydroamnios. She has no complaints except mild HTN.
What is the appropriate management :

a) Wait

b) Induce labor post 42 wks

c) Induce labor

d) Do biophysical profile twice weekly

The correct answer is c

775) Full term wide pelvis lady, on delivery station +2, vertex,
CTG showed late deceleration, the most appropriate
management:

a) C/S

b) Suction

c) Forceps Delivery

d) Spontaneous Delivery
The correct answer is c (I'm not sure )

………………………………………………………………………………………………………
……………………………………………..

776) Recent study revealed that anti psychotic meds cause the
following complication:

a. wt gain
b. alopecia
c. cirrhosis

the correct answer is a

………………………………………………………………………………………………………
……………………………………………..

777) compilication of rapid correction of hypernatremia :


a.brain edema

in hypernatremia : gradually correction to prevent


cerebral edema

in hyponatremia : gradually correction to prevent


,myelinolysis ( include paraparesis \ quadriparesis , dysarthria
and coma

………………………………………………………………………………………………………
……………………………………………..
778) prophylaxis of cholera :

Cholera is an infection of the small intestine that is caused by the


bacterium Vibrio cholerae. The main symptoms are profuse
watery diarrhea ( rice-watery diarrhea ) vomiting and
dehydration . infection by fecal-oral rout .

treatment : 1- rehydration , 2-antibiotic : young & adult :


doxcycline or tetracycline , for children : SMX-TMP , for pregnant :
furazolidone .

prphylaxis : good hygiene and sanitation and oral vaccine , in


epidemic public : mass single dose of vaccine and tetracycline.
778)which one of the anti TB medications cause tinnitus,
imbalance..

a. streptomycin

b. isoniazide

c. pyrizinamide

the correct answer is a , sterptomycine : cause 8th nerve


damage

……………………………………………………………………………………………………
………………………………………………..
779) the following combination drug should be avoided :

a. levodopa & digoxin

Q is not complete

………………………………………………………………………………………………………
……………………………………………..
780)young male had pharyngitis,then cough ,fever,most likely
org

a. staph aureus

b. strept pneumonia

the correct answer is b

………………………………………………………………………………………………………
……………………………………………..
781)most effective measure to prevent spread of infection
among health care workers & pts in a nursery:

a. wash hand befor and after examining each pt

b. wear gown and gloves before entering the nursery

c. wear shoe cover

the correct answer is a


782) 27 yrs old female with perianal pain for 4 days tender
erythematous fluctuating

a. Abx

b. local CS

c . Sitz bath

e. evac & drain

the correct answer is e

………………………………………………………………………………………………………
……………………………………………..
783)female pt developed extreme from zoo,park, sporting
events, the fear prevented her from going out:

a. agoraphobia

b. social phobia

c. schizophrenia

the correct answer is a , ( agoraphopia : fear going out from


the home )

………………………………………………………………………………………………………
……………………………………………..
784)child starts to smile:

a. at birth

b. 2month

c. 1month

The correct answer is b

……………………………………………………………………………………………………
………………………………………………..
785)child recognize 4 colors, 5 words, hops on one foot,
consistent with which age:

a. 12 mons

b. 24 mons
c. 36 mons

d. 18 mons

the correct answer is c

786)pt with hx of 5 yrs HTN on thiazide, came to ER midnight


screaming holding his Lt foot, o/e pt afebrile,Lt foot tender
erythema, swollen big toe most tender and painful, no
other joint involvement

a. cellulitis

b. gouty arthritis

c. septic arthritis

the correct answer is b

………………………………………………………………………………………………………
……………………………………………..
787)23 yrs old married for 3 mons, c/o not getting pregnant,
they have intercourse 3-4 times/week, normal gynecologic
hx, husband 25 yrs old healthy wt would you advice:

a. cont. trying
b. obtain sperm analysis
c. study of tubes patency

the correct answer is a

………………………………………………………………………………………………………
……………………………………………..
788)pt with AF what is the most common complication:

a. cerebrovascular events

b. v.tach

c. AMI

d. v.fib

the correct answer is a

………………………………………………………………………………………………………
……………………………………………..
789)case scenario : oral+genital ulcer +arthritis

a. behcet disease

b. syphilis

c.herpes simplex

the correct answer is a

790) 20 yrs old lady, pregnant, exposed to rubella virus since 3


days , never was vaccinated against rubella mumps or
measles , what's the best thing to do:

a. give IG

b. vaccine

c. do nothing

d. terminate the pregnancy

the correct answer is c , only supportive . ( 100 % correct  )

………………………………………………………………………………………………………
……………………………………………..

790)which of the following not compatible with head


engagement:

a. vertex at zero station

b. crowning of the head

c. 3/5 head felt in the abdomen

d. BPD at ischial spines

the correct answer is c , When the fetal head is engaged,


2/5 or less of the head is palpable above the pelvic
791)58 yrs old female, known case of osteopenia, she's asking
you abt the best way to prevent compression vertebral
fracture, what would you advice her:

a. avoid obesity

b. vit.D daily

c. wt bearing exercise

the correct answer is c , cnfuse with choice b

mild osteopenia ( T score -1 to -2 ): life style modification


( stop smoking ) , daily calcium intak ( 1500 mg daily ) ,
exercise , and reassessed after 3 – 5 years .

severe osteopenia ( T score -2 to – 2.5 ) and osteoporosis


( T score > 2.5 ) : same mild pus pharmacological .

………………………………………………………………………………………………………
……………………………………………..
792) Mother complains of sharp pain on radial styloid when
carrying her baby. The pain increase with extension of the
thumb against resistance, Finkelstein test was positive,
Dx :

a. Osteoarthritis of radial styloid

b. De Quervain Tenosynovitis

the correct answer is b

……………………………………………………………………………………………………
………………………………………………..
793) 1st line in Trigeminal Neuralgia management:

a. Carbamazepine

……………………………………………………………………………………………………
………………………………………………..
794) Contraindicated in acute glaucoma management:

a. Pilocarpine

b. Timolol

c. B-blockers, CA inhibitors, NSAID, Mannitol

d. ?

All of MCQs can be use in acute glaucoma . may be choice d is


correct if mention.

795) True about systolic hypertension:

a. could be caused by mitral Regurg

b. More serious than diastolic hypertension

c. systolic>140 and diastolic <90

d. ?

the correct answer is c

……………………………………………………………………………………………………
………………………………………………..
796) Pregnant 38GA, presented in labor, dilated cervix,
station 42, late deceleration on CTG, management:

a. continuo spontaneous labor

b. Forcep delevary

c. Vacuum Delevary

d. CS

The correct answer is a ( I'm not sure )


………………………………………………………………………………………………………
……………………………………………..
797) Pt, febrile, tender prostate on PR:

a-Acute Prostitis

………………………………………………………………………………………………………
……………………………………………..

798) Proven to prevent some cancers:

a. Ca

b. Folic Acid

c. Vit.D

the correct answer is c

………………………………………………………………………………………………………
……………………………………………..

799) Patient with cont. Murmur:

a. PDA

b. Coarctation of Aorta

the correct answer is a

800) Old man, post OR, Complains of unilateral face swelling:

a. Sjorjan syndrome

b. Sarcoidosis

c. Salivary gland tumor

d. Salivary gland stone

the correct answer is d

………………………………………………………………………………………………………
……………………………………………..
801) At what level LP done:
a. L2-L3

b. L3-L4

c. L5-S1

The correct answer is b

………………………………………………………………………………………………………
……………………………………………..

802) 2months old with scaling lesion on scalp and forehead,


Dx:

a. Seberrhoic Dermatitis

b. Erythema multiforme

the correct answer is a

………………………………………………………………………………………………………
……………………………………………..
803) Pt have high Blood Pressure on multible visits, so he was
diagnosed with hypertension, what is the Pathophysiology:

a. increased peripheral resistance

b. increased salt and water retention

the correct answer is a

………………………………………………………………………………………………………
……………………………………………..
804) 13years old with hx of pneumonia and managed with abx
2 weeks back, now he came with diarrhea, abdominal pain,
and +ve WBC in stool, the causative organism is:

a. Clostridium dificile

………………………………………………………………………………………………………
……………………………………………..

805) Best Manag. Of Clost.Deficile is:

a. Metronidazole
b. Doxacycline

the correct answer is a

806) Prophylaxis of arrhythmia post MI :

a. Quinidine

b. Quinine

c. Lidocaine

d. Procainamide

the correct answer is c , if b-blocker is present choose it

………………………………………………………………………………………………………
……………………………………………..
807) Chronic Diarrhea is a feature of:

a. HyperNatremia

b. HyperCalcemia

c. HypoMagnesemia

d. Metabolic Alkalosis

The correct answer is d

………………………………………………………………………………………………………
……………………………………………..
808) Pt with have Polyuria and thirst, he had Hx of bipolar on
lithium, Dx:

a. Nephrogenic Diabetes Insipidus

b. central Diabetes Insipidus

the correct answer is a

………………………………………………………………………………………………………
……………………………………………..
809) In city with population of 15000 people & 105 birth /yr ,
4 are still birth ,3 died within months ,2 died before their 1 st
birthday , with 750 p come out & 250 come in what is the
birth mortality rate in this city :

a. 4
b. 6

c. 8

d. 9

Q not complete ( I did't finde any thing about bierth


mortality rate )

810) Female with recently inserted IUCD coming with watery


brownish vaginal discharge & abdominal pain what is the
most likely Dx :

a. Uterine rupture

b. Ovarian torsion

c. Bacterial vaginosis

d. Ectopic pregnancy

The correct answer is d

……………………………………………………………………………………………………………
………………………………………..
811) 38 y/o male calling you complaining of fear from going
outside & fear from seeing the front door & don’t sit in the
room containing the front door what is most likely Dx:
a. Depression

b. Agoraphobia

c. Malingering

d. Anti-social personality

The correct answer is b

…………………………………………………………………………………………………………
…………………………………………..
812) 60 y/o male known to have ( BPH) digital rectal
examination shows soft prostate with multiple nodularity &
no hard masses , the pt request for ( PSA) for screening for
prostatic ca what will you do :

a. Sit with the pt to discuss the cons & rods in PSA test

b. Do trans-rectal US because it is better than PSA in


detection

c. Do multiple biopsies for different sites to detect


prostatic ca

d. …………………….

The correct answer is a

……………………………………………………………………………………………………………
………………………………………..
813) What is the most effective ttt for rocasea :

a. Clindamycine

b. Erythromycin

c. Topical steroids

d. ……..

Q is not clear or not complete. ( if mention tetracycline choose it )

Treatment of Rosacea : Oral tetracyclines & topical


metronidazole or topical erthyromycin or topical clinamycine . for
severe case : isotretinoin . surgical treatment for
rhinophyma .

814) 7 y/o child coming with SOB and wheezing he was sitting
in bed , leaning forward , with drooling & strider what is
Dx :

a- Epiglottits

b- Bronchial asthma

c- ………

d- ………

The correct answer is a

……………………………………………………………………………………………………………
………………………………………..

815) 10 y/o child brought by his parents because they where


concern about his weight , he eats a lot of fast food and
French fries , your main concern to manage this pt is :

a- His parents concerning about his weight

b- His BMI > 33

c- Family Hx of heart disease

d- Eating habit ( fast food , French fries )

……………………………………………………………………………………………………………
………………………………………..
816) What is an absolute contraindication of OCP :

a. Hx of previous DVT

b. Ovarian ca

c. Breast ca

d. ……….
The correct answer is a , both hx of DVT and breast cancer are
absolute contraindication , but in hx DVT is more accurate

……………………………………………………………………………………………………………
………………………………………..
817) OCP is proven to :

a. Decrease ovarian ca

b. Decrease endometrial ca

c. Increase breast ca

d. Increase risk of ectopic pregnancy

The correct answer is c

818) Baby born & discharge with his mother , 3weeks later he
started to develop difficulty in breathing & become
cyanotic what is most likely DX :

a- VCD

b- Hypoplastic lt ventricle

c- Coarctaion of aorta

d- Subaortic hypertrophy

The correct answer is b

……………………………………………………………………………………………………
………………………………………………..
819) 60 y/o male Dx to have acute pancreatitis ( there was
values for ranson criteria that I couldn’t memorize ) what is
the appropriate nutrition :

a- TPN

b- Regular diet with low sugar

c- High protein ,high ca , low sugar

d- Naso-jujenal tube
The correct answer is d

………………………………………………………………………………………………………
……………………………………………..

820) 15 y/o boy brought by his mother with lab values shows
microcytic hypochromic anemia what is most likely Dx :

a- Thalassemia

b- IDA ( iron deficiency anemia )

c- SCD

d- …….

The correct answer is b

………………………………………………………………………………………………………
……………………………………………..
821) Female with Hx of lt flank pain radiating to groin ,
symptoms of UTI & N+V what is DX:

a- Appendicitis

b- Diverteculitis

c- Renal colic

d- ……..

The correct answer is c

822) Old pt with neck swelling , nodular ,disfiguring , with Hx


of muscle weakness , cold intolerance , hoarseness , what is
your management :

a- Levothyroxine

b- Carbamazole

c- Thyroid lobectomy

d- Radio-active iodine
The correct answer is a

………………………………………………………………………………………………………
……………………………………………..
823) Pregnant woman with symptoms of hyperthyroidism ,
TSH low :

a- Propylthyiouracil

b- Radio-active iodine

c- Partial thyroidectomy

d- ………….

The correct answer is a

………………………………………………………………………………………………………
……………………………………………..
824) Pt with thyroid mass , firm ,2x2 cm what is most
appropriate for Dx :

a- Neck US

b- FNA

c- Neck CT

d- Surgery

The correct answer is b

………………………………………………………………………………………………………
……………………………………………..
825) 56 y/o female complaining of back pain , her mother
suffer from vertebral compression Fx , investigation shows
early osteoporosis what is the best ttt :

a- Advice with exercise

b- Vit D , ca supplement , …….

c- …………….

d- ……………
The correct answer is b

………………………………………………………………………………………………………
……………………………………………..
826) What is the most common non-traumatic Fx caused by
osteoporosis :

a- Colle's Fx

b- Femoral Fx

c- Vertebral compression Fx

The correct answer is c , because most common non traumatic .

827) 4 or 5 ( not sure ) brought by his parents with weight >


95th percentile , height < 5th percentile & bowing of both
legs what is the appropriate management :

a- Liver & thyroid function tests

b- Lower limb X-ray

c- Pelvis X-ray

d- Thyroid or ( liver not sure ) function test

The correct answer is a

……………………………………………………………………………………………………
………………………………………………..
828) At which stage sapration of chromatoids occur :

a- Metaphase

b- Telophase

c- ………….

d- ……………

The correct answer is a


………………………………………………………………………………………………………
……………………………………………..
829) Elderly pt with DM2 , HTN , smoker , sedentary lifestyle ,
inestigation low HDL, high LDL , ( ‫ ) المهم أن كل المآئسي فيه‬which of
what will be effective in this pt :

a- Level of HDL , smoking , obesity

b- Triglyceride level , ……………………

c- ………………………………………………….

d- ………………………………………………….( all were long sorry )

MCQs are not complete.

………………………………………………………………………………………………………
……………………………………………..
830) pt known case of ulcerative colitis with erythematous
rash in lower limb what is most likely DX:

a- erythema nodusum

831) Pt known to have ulcerative colitis coming with skin


lesion around Tibia which is with irregular margins what is
most likely Dx:

a- Pyoderma gangirenosum
……………………………………………………………………………………………………
………………………………………………..
832) Young pt victim of RTA coming with SOB ( or not
breathing not sure ) absent of breath sound & chest
movement in RT side but resonance in percussion what is
your next step :

a- Do stat chest X-ray

b- Emergency surgery

c- …………………………….

d- ………………………………

Needle aspiration then chest tube

………………………………………………………………………………………………………
……………………………………………..
833) Psychiatric pt non-compliance to his medication what is
the ttt :

a- …………. Given IM

b- ………………….. given IV

c- ………………..oral

d- …………………..oral

Q is not complete

………………………………………………………………………………………………………
……………………………………………..
834) Which psychiatric disease is treated with
electroconvulsive therapy :
a- Paranoia

b- ………………………..

The correct answer is : Major depression

835) Pt with bilateral eye discharge , watery , red eyes,


corneal ulceration what is the most common cause :

a- Dust & pollen

b- Hypertension & ……………

c- Ultra-violet light & stress

d- ………………………………….

The correct answer is a

………………………………………………………………………………………………………
……………………………………………..
836) 4 y/o baby felt down his mother pulled him by his arm &
since then he kept his arm in pronation position what is
your management:

a- Splint ………….

b- Do x-ray for the arm before any intervention

c- Orthopedic surgery

d- ……………………………….

The correct answer is b

………………………………………………………………………………………………………
..……………………………………………
838) diabetic pt on medication found un conscious his blood
suger was 60 what is the most common to cause this
problem:

a- Sufonylurease

b- Bigunides
c- ……………………

The correct answer is a

………………………………………………………………………………………………………
……………………………………………..

839) what is most sensitive indicator for factitious fever :

a- Pulse rate

b- …………………….

c- ……………………….

Factitious fever: Fever produced artificially by a patient. This is


done by artificially heating the thermometer or by self-
administered pyrogenic substances. An artificial fever may be
suspected if the pulse rate is much less than expected for the
degree of fever noted. This diagnosis should be considered in all
patients in whom there is no other plausible explanation for the
fever. Patients who pretend to have fevers may have serious
psychiatric problems.

840) young male ( I think he was a student ) with headache


involving the whole head & bilateral eye glob that increase
with stress what is most likely DX:

a- Tension headache

b- Migraine

c- …………………….

d- ……………………….

Q not cmplete

………………………………………………………………………………………………………
……………………………………………..
841) pt turns to be erratic , for 4 month he said that’s people
in TV knows what people are thinking about , in last 2
month he claim that he has special power that no one has
what is the most likely DX :
a- Uni-polar ….. depression

b- Bipolar …. Mania

c- SCZ

d- ……………………

The correct answer is c

………………………………………………………………………………………………………
……………………………………………..
842) common site of anal fissure is :

a- Anterior

b- Posterior

c- Lateral

d- …………………….

The correct answer is : b

………………………………………………………………………………………………………
..……………………………………………
843) common site of hernia sac in relation to the cord content :

a- Antro-lateral

b- Medial

c- Anywhere

d- Posterior

The correct answer is a

844) what a 4 years child can do :


a- Draw square & triangle

b- Say complete sentence

c- Tie his shoes

d- ……………………………..

The correct answer is a

………………………………………………………………………………………………………
………………………………………
845) You received a call from a father how has a son diagnosed
recently with DM-I for six months, he said that he found his
son lying down unconscious in his bedroom, What you will
tell him if he is seeking for advise:

a. Bring him as soon as possible to ER

b. Call the ambulance

c. Give him his usual dose of insulin

d. Give him IM Glucagone

e. Give him Sugar in Fluid per oral

The correct answer is e

………………………………………………………………………………………………………
……………………………………………..
846) factor which determine recurrence of breast cancer :

a- Site & size of breast mass

b- No of lymph nodes

c- Positive estrogen receptor

d- Positive progesterone receptor

The correct answer is b

………………………………………………………………………………………………………
……………………………………………..
847) teacher in school presented with 3 days Hx of jaundice
( they wrote it ictric as if it will a differacne) & abdominal
pain , N+V , 4 of school student had the same illness in lab
what is true regarding this pt :

a- Positive for hepatitis A IgG

b- Positive hepatitis A IgM

c- Positive hepatitis B core

d- Positive hepatitis B c anti-body

The correct answer is b

848) regarding weight gain in pregnancy what is true :

a- Pregnant woman should consume an average calorie 300-


500 per day

b- Regardless her BMI or body weight she should gain from


1.5 – 3 lb which represent the baby's growth

c- …………………………………………………………..

d- ……………………………………………………………….

Q is not complete

Weight gain during pregnancy :

- 100 – 300 Kcal / day , 500 Kcal / day in breastfeeding

- Wt. gain : 1 – 1.5 kg / month , 11 – 16 kg gain during


pregnancy.

( for change from Ib to Kg : divided by 2.2)

………………………………………………………………………………………………………
……………………………………………..
849) what condition is an absolute contraindication of lactation
:
a- Mother with open pulmonary TB for 3 month

b- Herpes zoster in T10 drematome

c- Asymptomatic HIV

d- …………………………………………….

The correct answer is c

………………………………………………………………………………………………………
……………………………………………..
850) what is most sensitive for DX of duodenal ulcer :

a- Epigasteric pain starting 30-60 min after the meal

b- Epigasteric pain staring immediately after a meal

c- Increasing of pain when lying supine

d- Pain radiating to the back

The correct answer is d

………………………………………………………………………………………………………
……………………………………………..
851) female at 40 yr with BMI >28 what is your management :

a- Reduce calorie in take to 800 /day

b- In general reduce calorie intake

c- ………………………………….

d- ………………………………………..

Q is not complete, but with these MCQs , the correct answer is b

852) 70 y/o female say that she play puzzle but for a short
period she can't play because as she develop headache
when playing what u will exam for
a. astigmatism
b. glaucoma

the correct answer is a


………………………………………………………………………………………………………
……………………………………………..

852) prophylaxis for meningitis ttt contact


a. cimitidine
b. rifampicin

the correct answer is b


………………………………………………………………………………………………………
……………………………………………..

853)rheumatic fever pt has streptococcal pharingitis risk to


develop another attack
a. trimesmore than normal
b. 100%
c. 50%

the correct answer is c

………………………………………………………………………………………………………
……………………………………………..

854)child with unilateral nasal obstruct with bad odor


a. unilateral adenoid hypertrophy
b. FB

the correct answer is b

………………………………………………………………………………………………………
……………………………………………..

855)PT. do colectomy for colon cancer routine follow up every


a. 6 months
b. 3 months
c. 9 months
d. 1 years

the correct answer is d


856) for patient with osteoarthritis , what is your initial action :

a. Intraarticular corticosteroid

b. Quadriceps strengthening exercise

c. Climb stairs exercise

The correct answer is b

………………………………………………………………………………………………………
……………………………………………..

857) 70-y old female patient with osteoporosis , what is her T


score :

a. (-2.5)

b. (-1)

c. (1)

d. (2)

The correct answer is a

………………………………………………………………………………………………………
……………………………………………..
858 ) diabetic patient , diagnosed 2 weeks back came to your
clinic at scheduled appointment supposed to be at 10:00
AM but because you were having another complicated
case , he had to wait for more than an hour , and he was
extremely angry , what u will do :

a. be empathetic as this anger is mostly because of the new


morbidity diagnosed at this patient

b. you start your talk with him by saying “I was having a


hard case “

c. Don’t say anything regarding being late unless he brings


it up
d. you star YOUR TALK WITH HIM BY SAYINH “you seem
furious”

………………………………………………………………………………………………………
……………………………………………..
859) 1st step in epidemic study is :

a. verifying diagnosis

Q is not complete

860) Randomized control trials become stronger if :

a. you follow more than 50% of those in the study

b. systematic assignment predictability by participants

Q is not complete , but with these MCQs , the correct answer is


b

………………………………………………………………………………………………………
……………………………………………..

861) child with multiple painful swellings on the dorsum of


hands , feet , fingers and toes ,his CBC showed Hb =7,RBC’s
on peripheral smear are crescent shaped , what is your
long-term care :

a. corticosteroids

b. penicillin V

c. antihistaminic

the correct answer is b , this pt. is sickle cell anemia

………………………………………………………………………………………………………
……………………………………………..
862) 54 y old patient , farmer , coming complaining of dry eye ,
he is smoker for 20 years and smokes 2 packs/ day , your
recommendation :

a. advise him to exercise

b. stop smoking

c. wear sunscreen

The correct answer is b , smoking increase risk of dry eye


(international study )

………………………………………………………………………………………………………
……………………………………………..
863) patient is wearing contact lenses for vision correction
since ten years , now coming c/o excessive tearing when
exposed to bright light , what will be your advice to him :

a. wear hat

b. wear sunglasses

c. remove the lenses at night

d. saline eye dros 4 times / day

The correct answer is d ( I'm not sure )

864) management of mild epistaxis :

a. pinching the fleshy part of nose while telting head


foreword

………………………………………………………………………………………………………
……………………………………………..

865) most common site of malignancy in paranasal sinuses :

a. Maxillary sinus ( correct )

……………………………………………………………………………………………………
………………………………………………..
866) patient with hypertension , what is the best non-
pharmacological method to lower the elevated blood
pressure :

a. weight reduction ( correct )

………………………………………………………………………………………………………
……………………………………………..

867) newly diagnosed patient with hyper tension having


Na=147 , K=3 , what is the most likely cause of his
secondary hypertension :

a. hyperaldosteronism ( correct )

………………………………………………………………………………………………………
……………………………………………

868) 50 year old patient , diagnosed with hypertension , he is


used to drink one glass of wine every day , he is also used
to get high Na and high K intake , his BMI is 30kg/m , what
is the strongest risk factor for having hypertension in this
patient :

a. wine

b. high Na intake

c. high K intake

d. BMI=30

the correct answer is d , ( I'm not sure )

869) all of the following tests are necessary to be done before


initiating lithium except:

a. liver function tests

………………………………………………………………………………………………………
……………………………………………..

870) in dementia , best drug to use :


a. haloperidol

b. galantamine

the correct answer is b

treatment of dementia is cholinesterase inhibitor ( galantamine,


donepezil, rivastigmine, and tracing )

………………………………………………………………………………………………………
……………………………………………..

871) most important point to predict a prognosis of SLE patient


: ??

a. degree of renal involvement

b. sex of the patient

c. leucocyte count

the correct answer is a

……………………………………………………………………………………………………
………………………………………………..

872) what is the antibiotic that you will give if you will do
appendectomy :

a. doxycycline

b. ceftriaxone

c. metronidazole

d. cefuroxime

The correct answer is d

873) most common site for ectopic pregnancy :

a. fallopian tubes

……………………………………………………………………………………………………
………………………………………………..
874)child with low grade fever , rash , pharingitis , and tender
splenomegally , +ve EBV , Dx :

a. infectious mononeucleosis

……………………………………………………………………………………………………
………………………………………………..

875) best early sign to detect tension pneumothorax :

a. tracheal shift

b. ditended neck veins

c. hypotensionv

the correct answer is b , I'm not sure , I think tracheal shift is


the late sign .

………………………………………………………………………………………………………
……………………………………………..

876) First sign of magnisium sulfate toxicity is :

a. Loss of deep tendon reflex

………………………………………………………………………………………………………
……………………………………………..

877) Patient with mashroom toxicity will present with

a. Constipation

b. Hallucination

c. Anhydrosis

the correct answer is b


878) Child ate overdose of iron , best immediate management

a. Gastric lavage

b. Induce vomiting manually

c. Emetic drugs

d. Ipecac

the correct answer is a

………………………………………………………………………………………………………
……………………………………………..

879) Status epilepticus is :

a. Continuous sizure activity more than 30 min without


regaining consciousness

……………………………………………………………………………………………………
………………………………………………..

880) First sign of increased ICP is :

a. contralateral papillary dilatation

b. Ipsilateral papillary dilatation

c. Decrease level of consciousness

the correct answer is c

………………………………………………………………………………………………………
……………………………………………..

880) Patient with history of head trauma unable to move the


fork to his mouth , where is the lesion:

a. cerebellum
b. frontal lobe

c. temporal

the correct answer is a

………………………………………………………………………………………………………
……………………………………………..

881) Patient with lumbar canal stenosis , I forget the


presentation , what is the ttt:

a. Physiotherapy

b. Lumbar dicsectomy

Q IS NOT COMPLETE

Mild to moderate : NSAID + physiotherapy

Advance : epidural corticosteroid

Refractory : surgical laminectomy

882) Most common cause of otorrhia :

a. Csf leak

b. Acute otitis media

c. Cerumen leak

The correct answer is b

………………………………………………………………………………………………………
……………………………………………..

883) Most common symptoem of acute otitis media :

a. Pain

b. Discharge
c. Tinnitus

d. Vertigo

The correct answer is a

………………………………………………………………………………………………………
……………………………………………..

884) 12 yo boy is mocked at school because he is obese , ate a


lot of pill to sleep and never wake up again , best
management is :

a. Refer to mental professional

b. Tell him that most kid grow out before they grow up

c. Advice healthy food

………………………………………………………………………………………………………
……………………………………………..

885) 12 yo boy brought by his parent for routine evaluation ,


his is obese but otherwise healthy , his parents want to
measure his cholesterol level , what is the best indicator of
measuring this child cholesterol :

a. His parent desire

b. Family hx of early cva

c. High bmi

The correct answer is b


886) Diabetic pt. have neovasclarization and vetrous
hemorrhage , next step :

a. Refer to ophthalmologist

………………………………………………………………………………………………………
……………………………………………..

887) Histopatholgy of resected inflamed appendix will show :

a. Neutrophil in muscular wall

b. Lymphoid hyperplasia with multineucleated gaint cel

The correct answer is a

………………………………………………………………………………………………………
……………………………………………..

888) A child had bee bite presented after 18 hour with left
arm erythema and itching and…. Long scenario , what to
do:

a. Antihistaminic

b. Oral steroid

c. Subcutaneous epinephrine

The correct answer is a

………………………………………………………………………………………………………
……………………………………………..
889) Old man with urinary incontinence , plapable bladder
after voiding , urgency and sense of incomplete voiding dx;

a. Stress incontenece

b. Overflow

c. Reflex

The correct answer is b

………………………………………………………………………………………………………
……………………………………………..

890) Ttt of scabis :

a. Permethrin

891) Picture , hyperkeratotic , scaly lesion over the extensor


surface of knee and elbow , what to do to avoid
exacerbation :

a. Steroid

b. Avoid sun exposure

c. Avoid trauma

The correct answer is a

………………………………………………………………………………………………………
……………………………………………..

892) Which is true about allopuranol :

a. Good if given during acute gout

b. Uricisoric
c. Reduce the chance of uric acid stone

d. Can be antagonize by salysilate

The correct answer is c

………………………………………………………………………………………………………
……………………………………………..

893) man walking in street and saying bad words to stranger ,


he is not aware of his conditiond , what is the description :

a. flight of idea

b. insertion of idea

c. loosening of association

………………………………………………………………………………………………………
……………………………………………..

894) which of the following suggest that thyroid nodule is


bengin rather that malignant:

a. history of childhood head and neck radation

b. hard consistency

c. lemphadenopathy

d. presense of multiple nodules

the correct answer is d

thyroid cancer : ( cold and solid nodule )


895) child presented after sting bite with abd pain ,
arthalgia , lower extremity and buttock rash , normal RFT ,
+ve fecal occult stool , dx :

a. HSP

b. Lyme diases

The correct answer is a

………………………………………………………………………………………………………
……………………………………………..

896) best indicator for labor progress is :

a. frequency of contractions

b. strength of contractions

c. descent of the presenting part

the correct answer is c

………………………………………………………………………………………………………
……………………………………………..

897) lactating mother is taking Phenobarbital for sizure


prophylaxsis , what to advice her :

a. stop breast feeding immediately

b. breast feed after taking the pill by 8 hour

c. start weaning

d. does not matter

the correct answer is a

………………………………………………………………………………………………………
……………………………………………..

898) best single way to reduce high blood pressure is :


a. smoke cessation

b. decrease lipid level

c. reduce weight

the correct answer is c

………………………………………………………………………………………………………
……………………………………………..

899) drug of choice for supraventricular tachycardia is :

a.adenosine

900) patient with blood group A had blood transfusion group B


, the best statement that describe the result is :

a. type IV hypersensitivity

b. inflammatory reaction ….

c. ……….

The correct answer is : It is type 2 hypersensitivity

………………………………………………………………………………………………………
……………………………………………..

901) a nurse gave blood transfusion through CVP , 2 hours


later the patient is comatose and unresponsive , dx :

a. Septic shock

b. Blood group mismatch

The correct answer is a ( I'm not sure )

………………………………………………………………………………………………………
……………………………………………..
902) 100% O2 given for prolonged periods can cause all
except:

a. Retrosternal Pain

b. Seizures

c. Depression

d. Ocular Toxicity

e. ???

The correct answer is c ( I'm not sure because choice e is not


mention )

903) Organophosphorus poisoning, what is the antidote?

a. Atropine

b. Physostigmine

c. Neostigmine

d. Pilocarpine

e. Endrophonium
The correct answer is a

………………………………………………………………………………………………………
……………………………………………..

904) Which of the following pulse character goes with the


disease?

a. Collapsing pulse  Severe anemia

b. Pulsus Alternans  Premature ventricular complexes

c. Slow rising Pulse  Mitral stenosis

d. Pulsus Bisferens  Mitral Regurgitation

e. Pulsus paradoxus  I think Aortic stenosis or


regurgitation

The correct answer is a

PLUSE DEFINITION CAUSES

Collapsing Pulse with fast upstroke and 1- Sever anemia


pulse fast downstroke 2- Aortic regurge
3- Thyrotoxicosis

Pulse alternans Altrnans weak and strong pulse 1- LV dysfunction

Pulsus Decline systolic more than 10 1- Cardiac


paradoxus mm Hg during inspiration tamponade
2- Asthma & COPD
Pulsus parvus Weak and delayed pulse 1- Aortic stenosis
et tardus

Slow rising Slow upstroke pulse 1- Aortic stenosis


pulse

Pulsus pulse characterized by two 1- Aortic reguirge


bisferiens strong systolic peaks separated 2- AR + AS
by a midsystolic dip
905) Entamoeba histolytica cysts are destroyed best by:

a. Boiling

b. Iodine added to water

c. Chlorine added to water

d. Freezing

The correct answer is a

………………………………………………………………………………………………………
……………………………………………..
906) A scenario about an old male with symptoms suggesting
parkinsonism such as difficulty walking, resting tremors
and rigidity in addition to hypotension. Then he asks about
what is the most common presenting symptom of this
disease

a. Rigidity

b. Tremors

c. Unsteady Gait

d. Hypotension

e. ???

The correct answer is b

………………………………………………………………………………………………………
……………………………………………..
907) Regarding postpartum Psychosis:

a. Recurrences are common in subsequent pregnancies

b. It often progresses to frank schizophrenia

c. It has good prognosis

d. It has insidious onset

e. It usually develops around the 3rd week postpartum


The correct answer is a

………………………………………………………………………………………………………
……………………………………………..
908) A pregnant female develops lesions on the vulva and
vagina and she was diagnosed as genital herpes, what
should be included in her future health care?

a. Cesarian section should be done if the lesions did not


disappear before 2 weeks of delivery date

b. Oral acyclovir to treat herpes

c. Termination of pregnancy because of the risk of fetal


malformations

d. Avoidance of sexual intercourse for 1 month after the


healing of the lesions

e. ?????

The correct answer is b

HSV in pregnant treated by : oral acyclovir 400 mg TID for 5-7


days . if HSV was present at time of labor : c – section

909) A picture of 2 warts in the index finger.

Diagnosis is  Viral Warts

………………………………………………………………………………………………………
……………………………………………..

910) What is the most common sequalae of acute


pancreatitis ?

a. Abscess

b. Pseudocyst

c. Bowel obstruction
The correct answer is b

Most common complication of acute pancreatitis:


pancreatic pseudocyt and fistula formation

………………………………………………………………………………………………………
……………………………………………..

911) An old patient with history of cerebrovascular disease &


Ischemic heart disease, presents with a pattern of
breathing described as: A period of apnea followed by slow
breathing which accelerates & becomes rapid with
hyperpnea & tachycardia then apnea again. What is this
type of breathing?

a. Hippocrates

b. Chyene-stokes breathing

c. Kussmaul breathing

d. One type beginning with O letter and contains 3 letters


only

e. ????

The correct answer is b

1- Chyene-stokes respiration : rapid deep breathing phase


followed by period of apnea , present with heart failure , stroke,
brain trauma , also can be with sleep or high altitude

2-kusmmaul's breathing : rapid and deep breathing . present


with metabolic acidosis particularly in diabetic ketoacidosis
912) A patient with mixed 1st & 2nd degree burns in head &
neck region, what is the most appropriate management?

a. Apply silver sulfadiazine and cream to all burned


areas,cover them and admit to hospital

b. Apply cream to 2nd degree burns and cover them, give IV


fluids

c. Debridement of 2nd degree burns and ….

d. Apply silver sulfadiazine then Vaseline ointment to all


areas then discharge the patient

The correct answer is a

1sr degree : epidermis + painful ( no blister )

2nd degree : epidermis + partial dermis + painful _ blister

3rd degree : epidermis + full thickness dermis + painless

Rule of nine to estimate BSA :

1- Head and neck : 9%

2- Each upper arm : 9%

3- Each loer limb : 18%

4- Each post. & ant. Trunk : 18%

5- Perineum & genitalia : 1%

-Silver sulfadiazine and mafenide used for all types of burn

-supportive : tetanus and I.V nacrotic can be used for all types of
burn
- I.V fluid use only in ( > 10% of BSA) in 2 nd & 3rd degree of burn
according parkland formula : ( fluid in first 24h = 4 * wt. in kg *
BSA . 50% in first 8 hours and another in next 16 hours )

In this patient 1st deree and 2nd degree < 10% ( head and neck
9%) : so, not use fluid

………………………………………………………………………………………………………
……………………………………………..

913) 2 years old child with ear pain & bulging tympanic
membrane, what is the diagnosis?

a. Otitis media

b. Otitis externa

c. Otomycosis

d. Bullous myringitis

The correct answer is a

914) Besides IV fluids, what is the most important drug to be


given in anaphylaxis?

a. Epinephrine

b. Steroids

c. ??? Other choices

The correct answer is a

………………………………………………………………………………………………………
……………………………………………..

915) An adult patient in 20s or 30s of age presents by history


of 1 month of fever, 5 days of headache & 2 days of altered
sensorium. On examination there is nuchal rigidity, then
there is a table showing investigations which include

Hb: 10 g/dl

Blood WBC: 18,000

CSF Examination: WBCs elevated: 77% lymphocytes, 33%


Neutrophils. Protein ???. Glucose??

What is the diagnosis?

a. Viral meningioencephalitis

b. Tuberculous meningitis

c. Ph…. Meningitis

d. ???

Q is not complete , but with this scenario and MCQs , The


correct answer is b

916) Regarding chronic fatigue syndrome, which is true?

a. Antibiotics may reduce the symptoms

b. Antidepressants may reduce the symptoms

c. Rest may reduce the symptoms


d. ???

e. ????

Q IS not complete , but with these MCQs , the correct answer is


b.

chronic fatigue syndrome:

characterizes by profound mental and physical exhaustion. In


associated with multiple system and neurospsychiatric
symptoms that last at least 6 mounth. Must be new ( not life
long ) , must not be relived by rest, and must result in greater
than 50% reduction in previous activity. Presentation with 4 or
more of the following : poor memory / concentration, myalgia,
arthalagia, sore throat, tender lymph node, recent onset
headach, unrefreshing sleep, excessive tiredness with exercise.
Treatment by : cognitive and excercise therapy .also, diet,
physiotherapy, dietary supplements, antidepressants.

………………………………………………………………………………………………………
……………………………………………..

917) A patient complains of dry eyes, a moisturizing eye drops


were prescribed to him 4 times daily. What is the most
appropriate method of application of these eye drops?

a. 1 drop in the lower fornix

b. 2 drops in the lower fornix

c. 1 drop in the upper fornix

d. 2 drops in the upper fornix

………………………………………………………………………………………………………
……………………………………………..

918) A patient with penetrating abdominal stab wound. Vitals


are: HR 98, BP 140/80, RR 18. A part of omentum was
protruding through the wound. What is the most
appropriate next step:

a. FAST Ultrasound
b. DPL (Diagnostic peritoneal lavage)

c. Explore the wound

d. Arrange for a CT Scan

e. Exploratory laparotomy

The correct answer is e

919) A patient presents with loin pain radiating to the groin.


Renal stones are suspected. What is the test that has the
most specificity & sensitivity in diagnosing this condition?

a. Noncontrast spiral CT scan of the abdomen

b. Ultrasound

c. KUB

d. Intravenous pyelography (IVP)

e. Nuclear Scan

The correct answer is a

………………………………………………………………………………………………………
……………………………………………..

920) An old patient presents with history dizziness & falling


down 1 day ago accompanied by history of Epigastric
discomfort. He has very high tachycardia I think around
130-140 and BP 100/60. What is the diagnosis?

a. Peptic ulter

b. GERD

c. Leaking aortic aneurysm

d. ???

The correct answer is c

………………………………………………………………………………………………………
……………………………………………..
921) A child is about to be given flu vaccine, what allergy
should be excluded before giving the vaccine?

a. Chicken

b. egg

c. Fish

d. ???

The correct answer is b

………………………………………………………………………………………………………
……………………………………………..

922) Which of the following is proven to reduce the incidence


of cancer?

a. Salt

b. Mineral water

c. Vitamin D

d. Fiber

The correct answer is c

923) A female patient with history of cyclic abdominal pain,


inability to conceive, heavy menses, and examination
showed tenderness & nodularity in uterosacral ligaments.
What is the diagnosis?

a. Endometriosis

………………………………………………………………………………………………………
……………………………………………..

924) A female patient on the 3rd week postpartum. She says to


the physician that the frequently visualizes snakes crawling
to her baby’s bed. She knows that it is impossible but she
cannot remove the idea from her head. She says she wakes
up around 50 times at night to check her baby. This
problem prevents her from getting good sleep and it
started to affect her marriage. What is this problem she is
experiencing?

a. An obsession

b. A hallucination

c. A postpartum psychosis

d. A Delusion

The correct answer is a

Obsession : persistent, unwanted, and intrusive ideas ,


thoughts, impulses or images

………………………………………………………………………………………………………
……………………………………………..

925) Regarding screening for cancer, which of the following is


true?

a. Screening for cervical cancer had decreased in recent years

b. Screening for breast cancer had decreased in recent years

c. Screening for Colorectal cancer is inadequate for the high-risk


groups

d. Screening for lung cancer has reduced the mortality rate of


lung cancer

e. Screening for tobacco use is now adequately done by health


professionals

………………………………………………………………………………………………………
……………………………………………..

926) Regarding postpartum depression, what is the most


appropriate intervention to reduce the symptoms?
a. Include family in the therapy

b. Isolation therapy

c. Add very low doses of impiramine

d. Encourage breastfeeding

The correct answer is a

927) Which of the following is a side effect of Buprione, a drug


used to help smoking cessation:

a. Arrhythmia

b. Xerostomia

c. Headache

d. Seizure

The correct amswer is d

………………………………………………………………………………………………………
……………………………………………..
928) A 60 years old patient with history of heart attack 6
weeks ago, complaining of not getting enough sleep.
Psychiatric evaluation is unremarkable for depression or
anxiety, what should be given to this patient?

a. Amytriptiline

b. Buspirone

c. Buprione

d. Zolbidim

The correct answer is d

………………………………………………………………………………………………………
……………………………………………..
929) A patient is asked to face the wall, bend his waist, and
let his hands hang down without support. This test is used
as a screening tool for which of the following?

a. Lower limb asymmetry

b. Rectal prolapsed

c. Scoliosis

d. ????

The correct answer is c

This test is called for (Adam's Forward Bend Test )

930) A patient presents with sore throat, cervical


lymphadenopathy, mild splenomegaly, EBV antibodies are
+ve. What is the diagnosis?

a. Infectious Mononucleosis (EBV )

………………………………………………………………………………………………………
……………………………………………..

931) An old patient presents with knee pain, limitation of


movement and quadriceps muscle atrophy. On examination
there is limited range on movement, pain on movement &
Crepitus. What is the diagnosis?

a. Osteoarthritis

b. Rheumatoid arthritis

c. Ankylosing Spondylitis

The correct answer is a

………………………………………………………………………………………………………
……………………………………………..

932) A patient presents with long time history of knee pain


suggestive of osteoarthritis. Now he complains of unilateral
lower limb swelling and on examination there is +ve pedal
& tibial pitting edema. What is the next appropriate
investigation?

a. CXR

b. ECG

c. Echocardiography

d. Duplex ultrasound of lower limb

The correct answer is d

Osteoarthritis relief by rest . So, immobility pt. can


lead to DVT

………………………………………………………………………………………………………
……………………………………………..

933) Alopecia is a side effect of which antiepileptic?

a. Phenytoin

b. Carbamazepine

c. Na Valproate

d. ???
The correct answer is c

1-phenytoin : gingival hyperplasia, hirsuteism, ataxia

2-carbamazepine : agranulocytosis, hepatotoxicity, aplastic


anemia

3- Na Valproate : transient hair loss.

934) Which of the following is an indication for tonsillectomy?

a. Sleep apnea

b. Asymptomatic large tonsils

c. Peripharygeal abscess

d. Retropharyngeal abscess

The correct answer is a

………………………………………………………………………………………………………
……………………………………………..

935) Which of the following is a gram –ve rods that grow on


charcoal yeast agar?

a. Legionella

………………………………………………………………………………………………………
……………………………………………..

936) A 7 years old child had history of chest infection which


was treated with antibiotics. The patient presented 6 weeks
after cessation of antibiotics with abdominal pain, fever
and profuse watery diarrhea for the past month. Which of
the following organisms is responsible for the patient’s
condition?

a. Giardia Lamblia
b. Clostridium Difficile

c. Escherichia coli

d. Clostridium Perfringens

The correct answer is b

………………………………………………………………………………………………………
……………………………………………..

937) Which of the following features of ulcerative colitis


distinguishes it from crohn’s disease

a. Possible malignant transformation

b. Fistula formation

c. Absence of granulomas

d. Colon involvement

The correct answer is a

938) A very very long scenario about a female patient with


vaginal discharge “malodorous watery in character” with
pH of 6 & +ve clue cells but there is no branching
pseudohyphe. (He is telling you the diagnosis is vaginosis &
there is no fungal infection) Then he asks about which of
the following drug regimens should NOT be used in this
paitent:

a. Metronidazole (PO 500 gm for 7 days)

b. Metronidazole (PO 2 large dose tablets for 1 or 2 days)

c. Metronidazole (IV or IM …..)

d. Miconazole ( PO …..)
e. Clindamycin (PO …..)

The correct answer is d

This case is " bacteria vaginosis" and treatment by :


metradinazole or clindamycine . avoid miconazole because it is
antifungal

………………………………………………………………………………………………………
……………………………………………..
939) What is the condition that produces malodorous watery
vaginal discharge with +ve clue cells in wet mount
preparation slides:

a. Bacterial vaginosis

b. Vaginal Candidiasis

c. Trichomonas vaginalis

d. Gonorrhea

The correct answer is a

………………………………………………………………………………………………………
……………………………………………..
940) Which of the following is the best treatment for
Giardiasis:

a. Metronidazole

………………………………………………………………………………………………………
……………………………………………..
941) A woman with 9weeks History of elevated erythematous
wheals overall her body , she also has lip swelling. There is
no History of recent travel or food allergy or drug ingestion,
Diagnosis:

a. Chronic Angioedema & Urticaria

b. Contact Dermatitis

c. Solar Urticaria

d. Cholinergic Urticaria

e. Cold Urticaria
The correct answer is a

Chronic urticaria : if more than 6 months

Solar urticaria : due to sunlight

Cholinergic urticariae : due to brief increase in body


temperature.

Cold urtiaria : due to exposure to cold

942) A 17 years old male with history of mild intermittent


asthma. Attacks occur once or twice weekly in the morning
and no attacks at night. What should be the initial drug to
give?

a. Inhaled short acting B2 agonist as needed

b. Inhaled high dose corticosteroid as needed

The correct answer is a

………………………………………………………………………………………………………
……………………………………………..
943) A child presented with sore throat. Culture from the
throat revealed +ve meningiococci. The patient is now
asymptomatic. Which of the following should be done?

a. Reassurance

b. Rifampicin oral for 7 days

c. IM ceftriaxone 1 dose

d. Ceftriaxone oral

e. ??????

The correct answer is c ( I'm not sure )

2-day regimen of oral rifampin or a single dose of IM ceftriaxone


or a single dose of oral ciprofloxacin).
………………………………………………………………………………………………………
……………………………………………..
944) Which of the following increases the quality of the
randomized controlled study & make it stronger:

a. Systemic Assignment predictability by participants

b. Open Allocation

c. Including only the participants who received the full


intervention

d. Following at least 50 % of the participants

e. Giving similar intervention to similar groups

The correct answer is a

………………………………………………………………………………………………………
……………………………………………..
945) A scenario about an infant who presented with difficulty
breathing and sweating, examination shows hyperactive
precordium with loud S2 and pansystolic murmur grade 3/6
best heard in the left 3rd parasternal region. What is the
Diagnosis?

a. VSD

b. Mitral Regurgitation

c. PDA

d. Large ASD

The correct answer is a

946)Post IHD , your advise :


a. Yoga
b. isotonic exercise

???????????

………………………………………………………………………………………………………
……………………………………………..
947)very obese girl , what's the first step to reduce weight :
a. increase water intake
b. decrease calorie intake

the correct answer is b


………………………………………………………………………………………………………
……………………………………………..

948) 40 y/o male , presented with large hands , hepatomegaly


….. diagnosis :
a. acromegaly
b. gigantism

the correct answer is a


……………………………………………………………………………………………………
………………………………………………..
949) cousre of facial nerve when comes out from stylomastoid …
a. Deep to parotid gland & lateral to external carotid &
retromandibular vein

………………………………………………………………………………………………………
……………………………………………..
950) old female with hemorrhoids for 10 years , no
complication , your action ?
a. observe
b. surgery
c. increase fiber diet

the correct answer is c

………………………………………………………………………………………………………
……………………………………………..

951) femal about 30y c/o abdominal pain related to menses


(scenario going with endometriosis)…………next step in dx:

a. Laparoscopy

b. U/S

c. CT

The correct answer is a

Confirm diagnosis by laparoscopy , US & MRI can use but if


normal can not exclude endometriosis .
952) female about 30y with breast cancer (given cbc –chem. And
reavel low hb and hematocrite….) what is the next step in
mangment:

a. Staging

b. Lumpectomy

c. Mastectomy

d. Chemotherapy

………………………………………………………………………………………………………
………………………………………….....

953) pt with open angle glaucoma and k/c of COPD and DM ttt:

a. Timelol

b. betaxolol

c. Acetazolamide

the correct answer is c

………………………………………………………………………………………………………
………………………………………….....

954) infant with runny nose and fever after that develop
dysnea ,wheezing with working accessory muscles and
chest retractions with cyanosis, temp 38 dx:

a. Viral pnumonia

b. Bacterial pneumonia

c. Bronchiolitis
The correct answer is c

955) 34y female with HIV pap smear negative, about cervical
cancer screening :

a. After 3m if negative repeat after 6m

b. After 6m …………………………… annually

c. After 1y………………………………….annually

The correct answer is b (I'm not sure ), may be c .


• Screening should begin no later than age 21.
• Screening should begin earlier than age 21 if the patient is sexually active. In this
case, it should start 3 years after initiation of vaginal intercourse.

• Once initiated, screening should be performed annually if a traditional, glass-


slide-based technique is used. If liquid medium is used, Pap smear screening may
be performed every other year.

• After age 30, for women who have had 3 consecutive, normal Pap smears,
screening frequency may be reduced to every two to three years.

• Women who are HIV positive, immunocompromised due to disease or


medication, or are DES daughters, should continue annual screening.

• Screening may stop following a total hysterectomy (including the cervix), if the
the patient is at low risk, and has had three consecutive normal Pap smears within
the last 10 years.

• High risk patients, including those with a history of cervical cancer, DES
exposure in-utero, HIV positive, immunocompromised from medication, and
those tested positive for HPV, should continue to be screened indefinitely.

• Screening may stop after age 70, if the patient is low risk, and has had three
normal Pap smears over the last 10 years.

• Screening may be omitted in the case of women with life-threatening or other


serious illness

………………………………………………………………………………………………………
………………………………………….....

956) which drug increase incidence of reflux oesophagitis:

a. Theophylline

b. Amoxicilline

c. Metoclopramide

d. Rantidine

e. Lansoprazole

The correct answer is a


957) most effective ttt of cluster headach:

a. Ergotamine nebulizer

b. S/C Sumatriptan

c. 100% O2

d. IV Verapamil

The correct answer is c

………………………………………………………………………………………………………
………………………………………….....

958) old pt. with HTN and migraine ttt:

a. B blockers

b. ACE I

c. Ca blockers

The correct answer is a

………………………………………………………………………………………………………
………………………………………….....

959) lactating women 10 days after delivery developed


fever ,malaise, chills tender Lt breast with hotness and
small nodule in upper outer quadrant with axillary LN
.Leukocyte count was 14 *10/L dx:

a. Inflammatory breast cancer


b. Breast abscess

c. Fibrocystic disease

d. ????

???????

………………………………………………………………………………………………………
………………………………………….....

960) newborn with fracture mid clavicle what is true:

a. Most cases cause serious complication

b. Arm sling or figure 8 sling used

c. Most patient heal without complications

The correct answer is c

-most clavicles fracture in newborn no need to treatment


apart from careful handling. If the fracture is displaced and
baby in pain, simple sling is require.

961) 70y male with osteoporosis the T score of bone


densometry would be :

a. 3.5

b. -2.5

c. -1

d. 2
e. 3.5

The correct answer is b

- The T score is the number of standard deviation (SD ) a


patient's BMD deviated from the mean.

- T score > (- 1) : normal BMD

- T score (-1 to -2.5) : ostopenia

- T score < ( -2.5 ) : osteoprosis

………………………………………………………………………………………………………
………………………………………….....

962) what is the most complication after hesterectomy:

a. Ureteral injury

b. Pulmonary embolism

c. Haemorrhage

The correct answer is c

………………………………………………………………………………………………………
………………………………………….....

963) young female with BMI 18 , fine hair allover body ,


feeling of she is fat ,doesn’t eat well with excessive
exercise………………..

a. Anorexia nervosa?

b. Body dysmorphic disorder

c. Bulimia nervosa

The correct answer is a

………………………………………………………………………………………………………
………………………………………….....
964) what's advantage of mature human milk over cow's milk:

a. More protein

b. More Iron content

c. More calories

d. More fat

The correct answer is b

………………………………………………………………………………………………………
………………………………………….....

965) young male c/o of deformity of jaw .past h/o of


nasoplasty and blepharoplasty O/E nothing abnormal
………..dx:

a. Body dysmorphic syndrome

966) the most common cause of non-traumatic subarachnoid


haemorrhage:

a. Rupture aneurysm

b. Vessels abnormality

c. Hypertension

The correct answer is a

-Most common causes of subarachnoid hemorrhage are trauma


and berry aneurysm

………………………………………………………………………………………………………
………………………………………….....

967) Pt. with chest pain and s.o.b decreased by leaning forword
. O/E friction rub and increased JVP >>>> (a case of
pericardial effusion) ECG will show:

a. ST changes

b. Low voltage

c. Increase PR interval
ECG changes during pericarditis : diffuse ST elevation , PR
depression followed by t-wave intervention , and low voltage.

………………………………………………………………………………………………………
………………………………………….....

968) The causative organism of psudomembranous colitis is:

a. Clostridium difficile

………………………………………………………………………………………………………
………………………………………….....

969) scenario about arthritis (I couldn't remember the details )


but Joint aspirate reveal Gram negative diplococci …..dx:

a. Nisseria gonorrhea

N. gonorrhoeae : gram negative diplococci and can also cause conjunctivitis,


pharyngitis, proctitis or urethritis, prostatitis and orchitis.

………………………………………………………………………………………………………
……………………………………….....

970) Vertigo & …….. is caused by which of the following drug ?

a. Aphotercine B

b. Pnicilline reaction

c. INH

………………………………………………………………………………………………………
……………………………………….....

971) Baby with red macule & dilated capillary on the rt side of
the face

Sturge-Weber Syndrome or Nevus Flammeus (one of


them will come)
Don’t choose milia or cavernous haemangioma

972) A 25 year old secondary school teacher that every time


enters the class starts sweating and having palpitation, she
is a fired to give wrong information and be unparsed. What
is the diagnosis:

a. Specific Phobia

b. Social Phobia

The correct answer is b

………………………………………………………………………………………………………
……………………………………….....

973) At which chromosome is the cystic fibrosis gene:

a. Long arm chromosome 7

b. Short arm chromosome 7

c. Long arm chromosome 8

d. Short arm chromosome 8

e. Long arm chromosome 17

The correct answer is a

………………………………………………………………………………………………………
……………………………………….....

974) Regarding hepatocellular carcinoma (Hepatoma) Which is


true:

a. More common in females

b. ..

c. ..

d. The most common cancer in Africa and Asia

e. Increase risk in chronic liver disease


The correct answer is e , chronic disease especially Hepatitis B

………………………………………………………………………………………………………
……………………………………….....

975) A patient is having a 2 year history of low interest in live,


he doesn’t sleep well and can’t find joy in life, What is the
most likely diagnosis:

a. Dysthymia

b. Major depressive disorder

c. Bipolar disorder

d. ..

e. ..

The correct answer is a

976) What is the mechanism of OCD drugs:

a. Increase availability of Serotonin

b. Decrease production of Serotonin

c. Increase production of Serotonin

d. .. Serotonin

e. .. Serotonin
The correct anser is a

………………………………………………………………………………………………………
……………………………………….....

977) What is the MOST accurate test to detect early


pregnancy:

a. ..Serum BHCG

b. ..

c. Urine pregnancy test

d. Ultrasound

e. ..

The correct answer is a

………………………………………………………………………………………………………
……………………………………….....

978) A case of a patient with thin cervix and little amount of


cervical mucus, how would you treat her: ?????

a. Estrogen injections

…………………………………………………………………………………
…………………………………………………………….....

979) The most common cause on chronic interrupted rectal


bleeding is:

a. Diverticulosis

b. Hemorrhoids
The correct answer is b

………………………………………………………………………………………………………
……………………………………….....

980) A child is having a croup early morning, the most


common cause is:

a. Post nasal drip

981) A burn patient is treated with Silver Sulfadiazine, the


toxicity of this drug can cause:

a. Lycosytosis

b. Neutropenia

c. Electrolyte disbalance

d. Hypokalemia

e. ..

The correct answer is b , side effect of sulfadiazine

………………………………………………………………………………………………………
……………………………………….....
982) A case scenario about a patient who had appendectomy,
after that he has abdominal pain and constipation and
absent bowel sound, the most likely cause is:

a. Ilus paraticus

Q is not complete

………………………………………………………………………………………………………
……………………………………….....

983) A case scenario describing a patient who had a right


chest trauma than developed tension pneumothorax, the
immediate management is:

a. Insert a needle in the second intercostal space

b. Insert a needle in the fifth intercostal space

c. Give IV antibiotics

d. ..

e. Insert a tube in the fifth intercostal space

The correct answer is a

………………………………………………………………………………………………………
……………………………………….....

984) A patient that have a penile ulcer that healed after that
he developed a palm and sole rash, the most likely cause is:

a. Syphilis

Q is not complete
985) A case scenario about a patient who has on and off
episodes of abdominal pain and was found to have multiple
gallstones, the largest is 1 cm and they are not blocking
the duct, What will you do:

a. Give pain killers medication

b. Remove gallbladder by surgery

Q is not complete , but with these MCQs , the correct answer is b

…………………………………………………………………………………
…………………………………………………………….....

986) Most common symptoms of soft tissue sarcoma :

a. Paralysis

b. On growing mass

c. Pain

d. ..

The correct answer is b

………………………………………………………………………………………………………
……………………………………….....

987) Patient use illegal drug abuse and the blood show RNA
virus. Which hepatitis

a. A

b. B (DNA)
c. C

d. E

The correct answer is c , RNA virus transmitted by blood id HCV

………………………………………………………………………………………………………
……………………………………….....

988) A case scenario about bronchial carcinoma, which is true:


????

a. The most common cancer in females

b. Squamous cell carcinoma spreads faster

c. Adenocarcinoma is usually in the upper part

d. Elevation of the diaphragm on the x-ray means that the


carcinoma has metastasize outside the chest

e. Bronchoscopy should be done

The correct answer is c

989) old female with pubic itching with bloody discharge, then
she developed pea shaped swelling in her labia, most likely:

a. Bartholin cyst

b. Bartholin gland carcinoma

c. Bartholin abscess

d. ..
The correct answer is c

………………………………………………………………………………………………………
……………………………………….....

990) Patient has decrease visual acuity bilateral , but more in


rt side , visual field is not affected , in fundus there is
irregular pigmentations and early cataract formation . what
you will do

a. Refer to ophthalmologist for laser therapy

b. Refer to ophthalmologist for cataract surgery

c. ..

The correct answer is a

………………………………………………………………………………………………………
……………………………………….....

991) Inflammatory bowel disease is idiopathic but one of


following is possible underlying cause:

a. Immunological

…………………………………………………………………………………
…………………………………………………………….....

992) pregant lady deliverd Anencephalytillbirth occurance of


neural tube defect in next pregnancy
a. 8%
b. 2%
c. 10%
d. 20%
The correct answer is b , ( 2 – 4 % )
………………………………………………………………………………………………………
…………………………………………

993) typist theaner atrophy ,+ve tephlen sign


a. Median nerve
b. Ulnar
c. Brachial
The correct answer is a
994) painful pile
a. Exison daringe
b. Sitz path and steriod supp
c. Ab
d. Fiber food and analgesics
The correct answer is a
………………………………………………………………………………………………………
…………………………………………

995) 39 years old HIV patient with TB recive 4 drugs of tt after


one month
a. Continue 4 drugs for 1 years
b. Cintinue isoniazide for 9months
c. Contiue isonizide for 1 year

The correct answer is a


………………………………………………………………………………………………………
…………………………………………

996) female not married with normal investigation except FBS=142.


RBS196. so ttt
a. give insulin subcutaneous
b. advice not become married
c. barrier contraceptive is good
d. BMI control

The correct answer is d

………………………………………………………………………………………………………
…………………………………………

997) Entamoeba histolytica cysts are destroyed best by:


a. Boiling

b. Iodine added to water

c. Chlorine added to water

d. Freezing

The correct answer is a

998)A scenario about an old male with symptoms suggesting


parkinsonism such as difficulty walking, resting tremors and
rigidity in addition to hypotension. Then he asks about what is
the most common presenting symptom of this disease

a. Rigidity

b. Tremors

c. Unsteady Gait

d. Hypotension

E. ???

The correct answer is b

………………………………………………………………………………………………………
……………………………………………..
998) Regarding postpartum Psychosis:

a. Recurrences are common in subsequent pregnancies


b. It often progresses to frank schizophrenia

c. It has good prognosis

d. It has insidious onset

e. It usually develops around the 3rd week postpartum

The correct answer is a

………………………………………………………………………………………………………
……………………………………………..
999) Gastrictomy post-op 1 day. He have temperature 38.8 &
pulse 112. What is the most common cause ?

a. wound infection.

b. inflammatory mediator in the circulation.

c. UTI

d. normal

the correct answer is a or b

most common causes of post-operation fever : ( 6 w )

1- W ound

2- Wine = pneumonia or atlectasia

3- Water = UTI

4- What happen = transfusion

5- Walk = DVT

6- Wonder drug = asprin,erythromycin , isonizide

Immediately ( 24 h ) : post-op inflammation , wound infection ,


transfusion , streptococcus

Acute ( 3 – 7 days ) : pneumonia

1000) Malaria ?
- the most common cases is caused by Plasmodium
falciparum.

……………………………………………………………………………………………………
………………………………………………..

1001 ) Polyartheralgia rhumatica. What is the thing that


suggest it rather than ↑ ESR & C-reactive protein:

a. proximal muscle weakness

b. proximal muscle tenderness

the correct answer is a

………………………………………………………………………………………………………
……………………………………………..
1002) The cause of insulin resistance in obese is:

a. ↑ insulin receptors kinase activity

b. ↑ number of insulin receptor

c. ↑ circulation of anti-insulin

d. ↓ insulin production from the pancreas

e. ↓ post-receptor action

the correct answer is e (I'm not sure )

1003) Pt came with total bilirubin: 5.8 & direct bilirubin: 0.4

a. Guillain–Barré syndrome

b. Dubin-junson syndrome
c. Biliary sclerosis

d. Crigler-Najjar syndrome

e. Gilbert Syndrome

the correct answer is e

………………………………………………………………………………………………………
……………………………………………

1004) Pt complaining of hypotension & bradycardia.


Electrolytes show: ↓Na, ↑K, ↑Cl, ↑Urea. So the cause of this
is:

a. hyponatremia

b. hyperkalemia

c. hyperchloremia

d. uremia

the correct answer is a ( not sure )

‫تمنياتي للجميع بالتوفيق‬

‫ محمد الحربي‬:‫أخوكم‬

Orthopaedic – Qassim University

You might also like